NURS 223: Exam 4

¡Supera tus tareas y exámenes ahora con Quizwiz!

A patient is ordered desmopressin [DDAVP] 0.3 mcg/kg. The patient weighs 150 pounds. What dose should the nurse administer? Record your answer using two decimal places. _____ mcg

20.45 mcg 150 pounds divided by 2.2 = 68.18 kg; 68.18 x 0.3 mcg = 20.45 mcg dose

A patient is prescribed aminocaproic acid. Which statement is true concerning this therapy? 1 This medication is only given orally. 2 This medication is only used in adults. 3 This medication is for serious bleeding. 4 This medication is only given intravenously.

3) This medication is for serious bleeding. Aminocaproic acid is given for serious bleeding. It can be given via IV and also orally. It is used in both adults and children.

The nurse is assessing a patient with hemophilia for complications of clotting factor administration. Which assessment findings would indicate an adverse effect of this treatment? Select all that apply. A Hives and urticaria B Bethesda titer of 40 units C Creatinine level of 2.5 mg/dL D Potassium level of 3.2 mEq/L E Alkaline phosphatase of 303 units/L

A) Hives and urticaria B) Bethesda titer of 40 units Complications of clotting factor administration include allergic reactions, which can range from mild to anaphylactic, and the development of antibodies against the factor (known as inhibitors). Hives and urticaria indicate an allergic reaction. The Bethesda assay titers are used to detect the presence of inhibitors. Clotting factor administration is not associated with hypokalemia, renal dysfunction (elevated creatinine), or liver dysfunction (elevated alkaline phosphatase).

A nurse is teaching the staff about Starling's law. Which image should the nurse use to depict this concept in the teaching session? 1 2 3 4

1 [graph w/ Y = Contractile force (SV) & X = Fiber Length (Ventricular Diameter) The graph showing fiber length and contractile force would best illustrate Starling's law. Starling's law states that the force of ventricular contraction is proportional to muscle fiber length (up to a point). Accordingly, as fiber length (ventricular diameter) increases, there is a corresponding increase in contractile force. The other images depict venous valves, the auxiliary venous pump, and the forces that promote and impede the flow of blood.

When teaching the nursing student about coronary heart disease, which statement will the nurse include in the session? 1 "Coronary heart disease occurs due to the formation of atherosclerotic plaques." 2 "Coronary heart disease cannot be prevented by changing lifestyle modifications." 3 "Coronary heart disease is due to decreases in the cholesterol levels to below 200 mg/dL." 4 "Coronary heart disease is nonlethal and cannot be prevented by antilipemic medications."

1) "Coronary heart disease occurs due to the formation of atherosclerotic plaques." Coronary heart disease (CHD) occurs due to the formation of atherosclerotic plaques inside the walls of the coronary arteries. CHD is lethal and can be prevented by antilipemic medications that lower the serum cholesterol levels, if diagnosed early. Antilipemic therapy is used as primary prevention in patients with known risk factors for CHD. Cholesterol levels above 200 mg/dL increase the risk for developing coronary artery disease. The risk of CHD also increases with an unhealthy lifestyle, which should be modified.

Which statement indicates that a pregnant patient has understood the nurse's discharge teaching regarding folic acid? 1 "Folic acid deficit can cause spina bifida." 2 "Anemia is the only risk from no folic acid." 3 "Folic acid does not absorb without vitamin B12." 4 "I will double my dose if I forget to take it the day before."

1) "Folic acid deficit can cause spina bifida." The patient should realize that loss of folic acid can cause spina bifida in the developing fetus as well as other neural tube defects. The patient should continue dosing as scheduled and should not double the dose if a dose is missed. Anemia is not the only risk, as there are various risks to the fetus.

The nurse provides discharge instructions to a patient prescribed verapamil [Calan] SR 120 mg PO daily for essential hypertension. Which statement by the patient indicates understanding of the medication? 1 "I must make sure I swallow the pill whole." 2 "I'll need to reduce the amount of fiber in my diet." 3 "I will take the medication with grapefruit juice each morning." 4 "I should expect occasional loose stools from this medication."

1) "I must make sure I swallow the pill whole." "SR" indicates that the drug is sustained release; therefore, the patient must swallow the pill intact, without chewing or crushing, which would result in a bolus effect. Grapefruit juice should be avoided, because it can inhibit intestinal and hepatic metabolism of the drug, thereby raising the drug level. Constipation, not loose stools, is a common side effect of verapamil [Calan]; increasing fluids and dietary fiber can help prevent this adverse effect.

A patient receiving atorvastatin [Lipitor] therapy to reduce high cholesterol levels calls the clinic and reports, "I am experiencing severe pain in both my legs." What is the nurse's best response? 1 "Stop taking the drug and visit the clinic immediately." 2 "Continue taking the drug; leg pain is a common side effect." 3 "Stop taking the drug if the symptoms persist for another week." 4 "Continue taking the drug along with niacin [Niaspan] and a pain killer."

1) "Stop taking the drug and visit the clinic immediately. The patient may have pain in both legs due to myopathy, an adverse effect of atorvastatin [Lipitor]. It progresses to a life-threatening condition called rhabdomyolysis, which involves the breakdown of muscle proteins leading to renal failure and death. The nurse should instruct the patient to stop taking the drug and immediately visit the clinic. The nurse will not instruct the patient to continue the drug as it is a life-threatening condition and requires immediate medical attention. The nurse will instruct the patient to watch for the symptoms; rhabdomyolysis further worsens the patient's condition, leading to renal failure. Administration of niacin [Niaspan], along with atorvastatin [Lipitor], further increases the breakdown of muscle proteins and causes rhabdomyolysis. The nurse should ask the patient to stop taking the medication until confirming the cause of the leg pain.

A patient received desmopressin preoperatively to maintain hemostasis during a surgical procedure. Which postoperative assessment finding indicates the patient has experienced an adverse effect of the drug? 1 +2 edema of the extremities 2 Blood glucose level of 220 mg/dL 3 Serum sodium level of 150 mEq/L 4 Serum potassium level of 5.5 mEq/L

1) +2 edema of the extremities Principal adverse effects of desmopressin are fluid retention and hyponatremia. Edema of the extremities is a sign of fluid retention. A sodium level below 135 mEq/L is considered low.

To be effective for treating chickenpox, acyclovir [Zovirax] should be initiated within how many hours of the appearance of the rash? 1) 24 2) 36 3) 48 4) 72

1) 24 The oral acyclovir [Zovirax] is used to treat varicella infections (chickenpox) in immunocompetent children if used within 24 hours of the appearance of the rash.

Which patient assessment would assist the nurse in evaluating the therapeutic effects of a calcium channel blocker? 1 Absence of chest pain 2 Patient denies dizziness 3 Patient states that she feels stronger 4 Decreased swelling in the ankles and feet

1) Absence of chest pain The workload in the heart should be decreased with the vasodilation from the calcium channel blocker. With less strain, the patient should have fewer incidences of angina as afterload is decreased. Diuretics are used to decrease edema in the ankles and feet. Dizziness may be a side effect of the medication. Calcium channel blockers may cause fatigue.

Before the nurse administers isosorbide mononitrate [Imdur], what is a priority nursing assessment? 1 Assess blood pressure. 2 Assess serum electrolytes. 3 Monitor level of consciousness. 4 Measure blood urea nitrogen and creatinine.

1) Assess blood pressure. Isosorbide mononitrate [Imdur] is a vasodilator and thus can cause hypotension. It is important to assess blood pressure before administering.

A patient is diagnosed with cytomegalovirus (CMV) retinitis. Which drug would the nurse expect to administer to this patient? 1 Cidofovir [Vistide] 2 Ribavirin [Rebetol] 3 Acyclovir [Zovirax] 4 Amantadine [Symmetrel]

1) Cidofovir [Vistide] Cidofovir [Vistide] is an intravenous (IV) drug with just one indication: cytomegalovirus (CMV) retinitis in patients with AIDS who have failed on ganciclovir or foscarnet. Acyclovir [Zovirax] is effective against herpes simplex virus types 1 and 2, herpes zoster, and chickenpox. Ribavirin [Rebetol] is used to treat respiratory syncytial virus infection. Amantadine [Symmetrel] is an antiviral drug developed to treat influenza A but is no longer recommended for this purpose and is now used to treat Parkinson's disease.

A patient is receiving transdermal clonidine [Catapres]. What information does the nurse need to include in the teaching plan? 1 Do not abruptly discontinue the medication. 2 Take the medication on an empty stomach. 3 Stay in bed for at least 1 hour when receiving a new patch. 4 Keep the new patch on for 30 minutes before removing the old patch.

1) Do not abruptly discontinue the medication. Abrupt discontinuation of clonidine can cause rebound hypertension.

A patient is receiving weekly subcutaneous injections of peginterferon-alfa-2a [Pegasys] for chronic hepatitis C. A nurse teaches the patient that which adverse effect will diminish with continued therapy? 1 Flu-like symptoms 2 Dyspnea and wheezing 3 Black, sticky, tarry stools 4 Lower leg muscle weakness

1) Flu-like symptoms The most common adverse side effect of peginterferon-alfa-2a, a long-acting interferon used to treat chronic hepatitis C, is flu-like symptoms characterized by fever, fatigue, myalgia, headache, and chills. Symptoms are likely to diminish with continued therapy over time. Black, tarry stools; lower leg muscle weakness; dyspnea; and wheezing are not adverse effects of peg-interferon-alfa-2a.

A nurse discovers that a patient has a cardiac output of 5 L/min. What else should the nurse assess to determine cardiac output besides stroke volume? 1 Heart rate 2 Blood pressure 3 Cardiac preload 4 Cardiac afterload

1) Heart rate The basic equation for cardiac output is CO = HR × SV, where CO is cardiac output, HR is heart rate, and SV is stroke volume. Cardiac preload and afterload help determine stroke volume. Blood pressure is affected by cardiac output.

Before administering amiodarone, what is the most important assessment for the nurse to complete? 1 Heart rate 2 Temperature 3 Skin integrity 4 Respiratory rate

1) Heart rate The nurse should assess heart rate before administering amiodarone. Altered skin integrity, elevated temperature, and decreased or increased respiratory rate are not contraindications. The other absolute contraindications include severe sinus bradycardia or second- or third- degree heart block.

The nurse is caring for a patient who has coronary heart disease (CHD). The nurse tells the patient, "Your cholesterol levels are abnormal; you are at a high risk of having a heart attack." What did the nurse discover regarding the lipoprotein levels in the patient's blood report? 1 High levels of low-density lipoproteins (LDL) 2 High levels of high-density lipoproteins (HDL) 3 Low levels of very-low-density lipoproteins (VLDL) 4 Low levels of intermediate-density lipoproteins (IDL)

1) High levels of low-density lipoproteins (LDL) High level of low-density lipoproteins (LDL) refers to high cholesterol levels in the blood, as LDL is almost entirely composed of cholesterol. This cholesterol is bad cholesterol, which promotes the formation of atherosclerotic plaque resulting in CHD. High-density lipoproteins (HDL) are good cholesterol, which has a cardioprotective action. Low levels of very-low-density lipoproteins (VLDL) are due to a low fat diet; however, it does not cause high cholesterol levels. Low levels of intermediate-density lipoproteins (IDL) do not increase the risk of CHD; they are useful for the production of bile acids.

The nurse is reviewing the chart of a patient with heart failure. Which information would lead the nurse to notify the provider? 1 Hypokalemia 2 Pulse rate 62 3 Large amount clear yellow urine 4 Carvedilol 12.5 mg twice/day (bid)

1) Hypokalemia The most common cause of dysrhythmias in patients receiving digoxin is hypokalemia secondary to the use of diuretics (in this case furosemide). Pulse rate 62 is normal, and the provider should not be notified. Carvedilol is a beta blocker given for heart failure, and the provider should not be notified. The provider should not be notified for the large amount of clear yellow urine because digoxin increases urine production as does furosemide, a loop diuretic.

When will the nurse administer hydroxymethylglutaryl-coenzyme A (HMG-CoA) reductase inhibitors (statins)? 1 In the evening 2 With breakfast 3 With an antacid 4 On an empty stomach

1) In the evening The liver produces the majority of cholesterol during the night. Thus, it is best to give HMG-CoA reductase inhibitors (statins), which work to decrease this synthesis, during the evening so that blood levels are highest coinciding with this production. Since this drug has a tendency to elevate the liver enzyme level, it may not be advisable to take the drug on an empty stomach. Since the liver produces the majority of cholesterol during the night, it is not ideal to give the drug during breakfast. An antacid is generally given to prevent stomach upset.

The long half-life of amiodarone [Cordarone] contributes to which complication? 1 Liver toxicity 2 Decreased dosing 3 Short onset of action 4 Enhanced therapeutic effects

1) Liver toxicity Amiodarone [Cordarone] has a long half-life, is highly lipid soluble, and accumulates in many tissues. Therefore, liver toxicity is of concern.

Hemophilia is a genetically based bleeding disorder seen almost exclusively in which patients? 1 Males 2 Females 3 Hispanics 4 Caucasians

1) Males Hemophilia is a genetically based bleeding disorder seen almost exclusively in males. Because males have only one X chromosome, a male with a defective gene has hemophilia. In contrast, a female with a defective gene on just one X chromosome is an asymptomatic carrier.

A patient with cardiovascular disease is taking rosuvastatin [Crestor]. Which finding would indicate a potential adverse effect of this drug? 1 Muscle pain and tenderness 2 Platelet count of 100 × 103/mm3 3 Blood pressure of 140/90 mm Hg 4 Wheezing and shortness of breath

1) Muscle pain and tenderness The statins, such as rosuvastatin [Crestor], typically are well tolerated; however, in rare cases they can cause the serious adverse effect of myopathy and rhabdomyolysis. If unexplained muscle pain and tenderness develop, the prescriber should be notified. The other effects would not likely be caused by rosuvastatin [Crestor].

A nurse is caring for a patient who is taking an angiotensin-converting enzyme (ACE) inhibitor and develops a dry, nonproductive cough. What is the nurse's priority action? 1 Notify the healthcare provider. 2 Tell the patient that the cough will subside in a few days. 3 Assess the patient for other symptoms of upper respiratory infection. 4 Instruct the patient to take antitussive medication until the symptoms subside.

1) Notify the healthcare provider. Angiotensin-converting enzyme inhibitors prevent the breakdown of bradykinin, frequently causing a nonproductive cough. Angiotensin-receptor blocking agents do not block this breakdown, thus minimizing this annoying side effect. The healthcare provide should switch the patient to a different medication if the side effect cannot be tolerated.

The nurse is caring for patient who is being treated with amiodarone [Cordarone]. The nurse notes that the patient is experiencing a hacking cough. What is the nurse's priority action? 1 Notify the provider regarding this symptom. 2 Document the findings in the patient's chart. 3 Administer medication to help the patient rest. 4 Administer the cough syrup that has been ordered.

1) Notify the provider regarding this symptom. The most serious adverse effect of amiodarone [Cordarone] is pulmonary toxicity, which involves a clinical syndrome of progressive dyspnea and cough accompanied by damage to the alveoli.

Which form of nitroglycerin is likely to have a large first-pass effect? 1 Oral 2 Sublingual 3 Transdermal 4 Intravenous

1) Oral Oral nitroglycerin goes to the liver and is metabolized before it can become active in the body. As a result, a large amount of nitroglycerin is removed from circulation. This is known as a large first-pass effect. Sublingual nitroglycerin has an onset of action of 2 to 3 minutes and is absorbed quickly because the area under the tongue is highly vascular. Transdermal nitroglycerin has an onset of action of 30 to 60 minutes. It is used for long-term management of angina pectoris because it allows for the continuous, slow delivery of nitroglycerin. Intravenous nitroglycerin is quickly absorbed in the blood; it has an onset of action of 1 to 2 minutes.

Why does the nurse anticipate administering metoprolol [Lopressor] rather than propranolol [Inderal] for diabetic patients who need a beta-blocking agent? 1 Propranolol causes both beta1 and beta2 blockade. 2 Metoprolol is less likely to cause diabetic nephropathy. 3 Propranolol is associated with a higher incidence of foot ulcers. 4 Metoprolol helps prevent retinopathy in individuals with diabetes.

1) Propranolol causes both beta1 and beta2 blockade. Metoprolol is a second-generation beta blocker and as such is more selective for beta1 receptors. At therapeutic doses, it causes less bronchoconstriction and suppression of glycogenolysis, which can cause problems in diabetic patients. Propranolol blocks both beta1 and beta2 receptors.

Which is the most important assessment to complete before the nurse administers an antidysrhythmic medication? 1 Pulse 2 Temperature 3 Respiratory rate 4 Auscultation of lungs

1) Pulse When antidysrhythmic drugs are administered, the nurse should monitor vital signs, especially pulse rate and blood pressure. If the pulse is lower than 60 beats/min, the nurse should notify the prescriber.

The nurse is caring for a patient receiving amiodarone [Cordarone]. Which body system should the nurse assess for serious adverse effects of this medication? 1 Respiratory 2 Integumentary 3 Gastrointestinal 4 Musculoskeletal

1) Respiratory Pulmonary toxicity is the most serious potential adverse effect of amiodarone [Cordarone]. It may manifest as pneumonitis or pulmonary fibrosis, with symptoms such as dyspnea, cough, and chest pain.

Which drug is used to manage viral pneumonia in a patient with respiratory syncytial virus? 1 Ribavirin [Virazole] 2 Flucytosine [Ancobon] 3 Griseofulvin [Grifulvin V] 4 Amantadine [Symmetrel]

1) Ribavirin [Virazole] Ribavirin [Virazole] is indicated for serious viral pneumonia caused by the respiratory syncytial virus. Amantadine [Symmetrel] is an antiinfluenza drug used in the management of Parkinson's disease). Flucytosine [Ancobon] and griseofulvin [Grifulvin V] are antifungal drugs.

Laboratory reports indicate that a patient's serum cholesterol concentration is 250 mg/dL and triglyceride levels are 450 mg/dL. The healthcare provider prescribes gemfibrozil [Lopid] to lower cholesterol levels. What is the desired effect of the drug on the patient's lipoprotein levels? 1 There would be a decrease in low-density lipoprotein (LDL) levels. 2 There would be an increase in low-density lipoprotein (LDL) levels. 3 There would be a decrease in high-density lipoprotein (HDL) levels. 4 There would be an increase in very-low-density lipoprotein (VLDL) levels.

1) There would be a decrease in low-density lipoprotein (LDL) levels. The serum cholesterol level of 250 mg/dL and the triglyceride levels of 450 mg/dL indicate that the patient has type IV hyperlipidemia. These patients have elevated VLDL levels. Gemfibrozil [Lopid] activates the enzyme lipoprotein lipase, an enzyme responsible for the breakdown of cholesterol. This drug decreases the very-low-density lipoproteins and increases the levels of LDL. A decrease in LDL levels occurs upon further continuation of the drug. Gemfibrozil [Lopid] increases HDL levels by converting LDL to HDL. However, in this case HDL remains unchanged at the time of the serum test. The serum levels of VLDL are decreased by gemfibrozil [Lopid] due to the activation of the enzyme lipoprotein lipase that converts VLDL to LDL.

The nurse is caring for a patient receiving propranolol [Inderal]. Which clinical finding is most indicative of an adverse effect of this drug? 1 Wheezing 2 Urinary urgency 3 A heart rate of 100 beats/min 4 A glucose level of 180 mg/dL

1) Wheezing Beta blockers, such as propranolol, are known to cause bronchoconstriction, which could manifest as wheezing. Other adverse effects could include bradycardia, atrioventricular heart block, heart failure, rebound cardiac excitation, inhibition of glycogenolysis, and potential central nervous system effects.

The nurse knows that systolic hypertension is characterized by systolic pressure of ____mm Hg or higher.

140 Systolic hypertension is a consistent systolic pressure of 140 mm Hg or higher with near normal diastolic pressure.

A patient with hemophilia is frightened about contracting human immunodeficiency virus (HIV) from therapy. How should the nurse respond to the patient? 1 "This is a risk." 2 "All blood is screened for viruses." 3 " Why do you think this is possible?" 4 " I will call the healthcare provider, so you can discuss this."

2) "All blood is screened for viruses." The nurse should tell the patient that all blood is screened for viruses and the risk of this type of transmission is very low to impossible. It has not been a risk for many years. The nurse should reassure the patient.

The nurse determines that the patient has understood the discharge teaching regarding warfarin [Coumadin] based on which of the patient's statements? 1 "I should keep taking ibuprofen for my arthritis." 2 "I should use a soft toothbrush for dental hygiene." 3 "I should decrease the dose if I start bruising easily." 4 "I will double my dose if I forget to take it the day before."

2) "I should use a soft toothbrush for dental hygiene." Using a soft-bristled toothbrush for dental hygiene will reduce the risk of bleeding. The patient should not double the dose of warfarin. Ibuprofen will potentiate bleeding. The patient should call the healthcare provider if experiencing excessive bruising.

Which statement made by a patient about nitroglycerin indicates understanding? 1 "I will refill my prescription once a year to maintain potency." 2 "I will keep the nitroglycerin stored in the bottle it comes in." 3 "I will take a nitroglycerin tablet 2 hours before I engage in physical activity." 4 "I will take a nitroglycerin tablet every 15 minutes until my chest pain is gone."

2) "I will keep the nitroglycerin stored in the bottle it comes in." Nitroglycerin needs to be stored away from heat, humidity, and light, all of which can decrease its potency. Keeping it in the brown light-resistant bottle helps achieve this. It should be replaced every 3 to 6 months to maintain potency. Nitroglycerin is used every 5 minutes for pain relief or just prior to engaging in activity that is known to cause chest pain.

The nurse is assessing a patient who has been prescribed atorvastatin [Lipitor]. What instruction should the nurse provide for the patient to ensure proper administration of the medication? 1 "Take the drug after breakfast." 2 "Take the drug in the evening." 3 "Take the drug with an antacid." 4 "Take the drug on an empty stomach."

2) "Take the drug in the evening." Cholesterol production by the liver usually occurs at night; thus, statin drugs such as atorvastatin [Lipitor] work by decreasing the cholesterol synthesis and are generally administered in the evening to reduce cholesterol production. All statins should be taken once daily during the evening meal or at bedtime. The drug need not be administered after breakfast because cholesterol level production is lesser in the mornings. Antacids may not be administered along with the drug as it doesn't cause gastric irritation. The desired therapeutic effects may not be produced if the drug is administered on an empty stomach.

A middle-aged male patient started antihypertensive drug therapy 3 months earlier and is in the healthcare provider's office for a follow-up visit. While the nurse is taking his blood pressure, he informs the nurse that he has had some problems with erectile dysfunction. Which would be the most appropriate response by the nurse? 1 "Don't worry. Eventually, tolerance will develop." 2 "The physician can work with you on changing the dose and/or medications." 3 "Sexual dysfunction happens with this therapy, and you will learn to accept it." 4 "This is an unusual occurrence, but it is important to stay on your medications."

2) "The physician can work with you on changing the dose and/or medications." Beta blockers and adrenergic medications used in the treatment of hypertension can result in the common adverse effect of erectile dysfunction in males. Consideration should be given to switching to an alternate medication for blood pressure control. The patient's complaint should not be dismissed, and factual information should be provided.

The nurse is caring for a patient who is receiving enteral feedings because of dysphagia. The healthcare provider prescribes isosorbide mononitrate [Imdur] 60 mg SR daily via the enteral tube. What is the most appropriate action by the nurse? 1 Have the patient swallow the pill, because it cannot be crushed. 2 Call the healthcare provider about an alternate form of nitrate for administration. 3 Place the nitrate under the patient's tongue and let it dissolve, because the person has dysphagia. 4 Crush the isosorbide mononitrate [Imdur] into a fine powder, dilute it with water, and administer it via the enteral tube.

2) Call the healthcare provider about an alternate form of nitrate for administration. The nurse should contact the prescriber for an alternate form of nitrate, such as a non-sustained-release pill, transdermal patch, topical ointment, or immediate release. Because isosorbide mononitrate [Imdur] is a sustained-release pill, it cannot be crushed. Because the patient has dysphagia, attempting to administer the pill orally could result in aspiration. Sustained-release tablets must be administered whole.

The patient is scheduled to receive argatroban. Which is the correct route by which to administer the drug? 1 Intradermal 2 Intravenous 3 Intramuscular 4 Subcutaneous

2) Intravenous Argatroban is given only by the intravenous route.

A patient has taken a medication that causes the arterioles to dilate. What will likely happen to the cardiac afterload? 1 It will rise. 2 It will decrease. 3 It will stay the same. 4 It will fluctuate wildly.

2) It will decrease. When arterioles dilate, peripheral resistance falls, causing arterial pressure (afterload) to decline. When arterioles constrict, peripheral resistance rises, also causing arterial pressure (afterload) to rise. It will not stay the same nor will it fluctuate wildly.

The patient who takes insulin has just been prescribed a beta blocker drug. The nurse recognizes that the interaction of the beta blocker and the insulin may have which result? 1 Enhanced activity of the beta blocker drug 2 Masked signs of hypoglycemia 3 Decreased activity of the beta blocker drug 4 Masked signs of hyperglycemia

2) Masked signs of hypoglycemia The interaction of insulin and a beta blocker drug is known to result in masking of the signs of hypoglycemia.

The nurse has just administered the initial dose of enalapril [Vasotec] to a newly admitted patient with hypertension. What is the priority nursing intervention over the next several hours? 1 Check the heart rate. 2 Monitor blood pressure. 3 Auscultate lung sounds. 4 Draw a potassium level.

2) Monitor blood pressure. First-dose hypotension is a serious potential adverse effect of angiotensin-converting enzyme (ACE) inhibitors such as enalapril [Vasotec]. Monitoring the blood pressure is the priority nursing intervention. If hypotension develops, the nurse will place the patient in the supine position and possibly increase intravenous fluids. The other interventions may be appropriate for this patient; however, in the hours immediately after the first dose of an ACE inhibitor, monitoring of the blood pressure is most important.

The nurse is administering intravenous lidocaine [Xylocaine] to a patient with a ventricular dysrhythmia. Which is the priority nursing intervention to prevent a potential complication with this drug? 1 Obtain a complete blood count. 2 Monitor the electrocardiogram (ECG). 3 Keep naloxone [Narcan] at the bedside. 4 Instruct the patient to report any chest pain.

2) Monitor the electrocardiogram (ECG). Continuous electrocardiogram (ECG) monitoring is required during lidocaine [Xylocaine] infusions to evaluate cardiac response and adjust dosage accordingly and detect toxicity. Blood counts are not necessary, because lidocaine [Xylocaine] is not linked to blood dyscrasias. Preferably Lidocaine [Xylocaine] is given intravenously (IV) rather than intramuscularly (IM). Sudden onset of chest pain is a sign of arterial embolism, which is a possible adverse effect of quinidine, not lidocaine [Xylocaine]. Naloxone [Narcan] is a reversal agent for opioids and is not used with lidocaine [Xylocaine].

The laboratory calls to report a drop in the platelet count to 90,000/mm3 for a patient receiving heparin for the treatment of postoperative deep vein thrombosis (DVT). Which action by the nurse is the most appropriate? 1 Obtain vitamin K and prepare to administer it by intramuscular (IM) injection. 2 Notify the healthcare provider to discuss the reduction or withdrawal of heparin. 3 Observe the patient and monitor the activated partial thromboplastin time (aPTT) as indicated. 4 Call the healthcare provider to discuss increasing the heparin dose to achieve a therapeutic level.

2) Notify the healthcare provider to discuss the reduction or withdrawal of heparin. The nurse should notify the healthcare provider to discuss the reduction or withdrawal of heparin. Heparin-induced thrombocytopenia (HIT) is a potential immune-mediated adverse effect of heparin infusions that can prove fatal. HIT is suspected when the platelet counts fall significantly. A platelet count below 100,000/mm3 would warrant discontinuation of the heparin.

A patient who is receiving heparin therapy has bruises covering the abdomen as well as red-colored urine. What does the nurse need to assess? 1 Urine culture 2 Platelet level 3 Over-the-counter medications 4 Ingestion of acetaminophen [Tylenol]

2) Platelet level The patient's platelet level should be assessed when he or she receives heparin and then develops bruises and blood in the urine. The patient may have type II heparin-induced thrombocytopenia (HIT). The patient's medication would need to be changed. Although over-the-counter medications may potentiate bleeding, they are not likely to cause these symptoms. Acetaminophen and urinary tract infections should not cause these symptoms.

The nurse notes significant edema surrounding and proximal to the peripheral intravenous (IV) site where epinephrine is being infused. Which action would the nurse anticipate first? 1 Ensure that naloxone [Narcan] is available. 2 Prepare to administer phentolamine [Regitine]. 3 Institute the protocol for congestive heart failure (CHF). 4 Monitor the blood urea nitrogen, creatinine, and potassium levels.

2) Prepare to administer phentolamine [Regitine]. Phentolamine, an adrenergic antagonist, is used to prevent tissue necrosis after extravasation of a vasoconstrictor, such as epinephrine. The edema surrounding the peripheral IV suggests infiltration of the IV, not CHF. Naloxone is an opioid antagonist.

A patient is given medication to make the blood vessels constrict. The nurse understands that what occurs with this medication? 1 Blood flow increases. 2 Resistance increases. 3 Vessel diameter enlarges. 4 Blood pressure decreases.

2) Resistance increases. When vessels constrict, resistance rises, causing blood flow to decline. To maintain adequate flow when resistance rises, blood pressure must rise as well. When vessels dilate, the diameter enlarges, while constriction causes the diameter to become smaller.

Which drug should the nurse avoid administering to a pregnant woman? 1 Cidofovir [Vistide] 2 Ribavirin [Virazole] 3 Acyclovir [Zovirax] 4 Foscarnet [Foscavir]

2) Ribavirin [Virazole] Respiratory syncytial virus (RSV) infection is a viral infection that is treated with ribavirin [Virazole]. It is Pregnancy Risk Category X and thus is strictly contraindicated in pregnant woman and their male partners, as the drug has teratogenic effects. Herpesvirus infections are treated with acyclovir [Zovirax], which may be used to treat viral infections during pregnancy. Non-nasal iInfluenza virus is not contraindicated in pregnancy; pregnant patients are vaccinated for influenza via injection. Cytomegalovirus infections are treated with cidofovir [Vistide] and foscarnet [Foscavir], which may be administered during pregnancy, though it is not recommended unless benefits outweigh risks.

During IV administration of norepinephrine [Levophed], the nurse notes extravasation at the IV insertion site on the patient's right arm. What is the nurse's priority action? 1 Notify the healthcare provider. 2 Stop the norepinephrine infusion. 3 Start a second IV line in the left hand. 4 Elevate the right arm above heart level.

2) Stop the norepinephrine infusion. Extravasation is a sign of norepinephrine infiltration. The infusion should be stopped immediately to prevent further harm to the patient. After stopping the IV infusion, the healthcare provider can be notified. Another IV line will likely need to be started. However, the norepinephrine must be stopped first. After the norepinephrine is stopped, the arm will need to be elevated to reduce swelling and an alpha-adrenergic antagonist, phentolamine, will need to be infiltrated into the region to minimize injury.

In the failing heart, arterial pressure falls, stimulating the baroreceptor reflex to increase sympathetic nervous system activity. The nurse understands increased sympathetic activity will produce which response? 1 Bradypnea 2 Tachycardia 3 Hypotension 4 Hypoglycemia

2) Tachycardia Increased sympathetic activity results in an increased heart rate (tachycardia), increased contractility, increased venous tone, and increased arteriolar tone (elevated blood pressure). Sympathetic stimulation also causes bronchodilation (not bradypnea) and possibly hyperglycemia.

A patient who is prescribed sublingual nitroglycerin [Nitrostat] reports after 6 months that she left the bottle in her car for a few weeks. The nurse instructs the patient to obtain a new prescription from the primary healthcare provider. What is the purpose of obtaining a new prescription? 1 The drug can cause toxicity. 2 The drug has lost its potency. 3 The patient needs a lower drug dose. 4 The patient has developed drug tolerance.

2) The drug has lost its potency. If nitroglycerin [Nitrostat] is not stored properly, the drug loses its potency, and thus a new prescription is needed for therapeutic benefits. The nurse should advise the patient to store the drug in its original container in a cool and dry place so that it does not lose potency. Drug tolerance is indicated if the potent drugs do not have the desired effect on the patient. A lower dose is prescribed if the patient has any adverse effects. Toxic levels are more of a concern if the patient's hepatic function is impaired.

The nurse is caring for several patients. Which patient will need teaching about treatment of hypertension? 1 The patient with a diastolic pressure of 70 mm Hg and above 2 The patient with a systolic pressure consistently above 140 mm Hg 3 The patient with a diastolic blood pressure 80 mm Hg after exercise 4 The patient with a blood pressure of 140/90 mm Hg after walking up two flights of stairs

2) The patient with a systolic pressure consistently above 140 mm Hg Hypertension is defined as a persistent systolic pressure of greater than 140 mm Hg and or a diastolic pressure greater than 90 mm Hg. A patient is not diagnosed with hypertension after one episode of an elevated pressure.

A large number of megaloblasts are seen in the patient's lab results. What should the nurse teach the patient? 1 This is a normal finding. 2 This is a result of B12 deficiency. 3 Megaloblasts are oversized white cells. 4 This could be caused by chronic kidney disease.

2) This is a result of B12 deficiency. The most conspicuous consequence of B12 deficiency is anemia, in which large numbers of megaloblasts (oversized erythroblasts) appear in the bone marrow and macrocytes (oversized erythrocytes) appear in the blood.

The nurse teaches a patient about therapy for hemophilia. What should be included in the plan? 1 Treatment can cure the disease. 2 Treatment with factor VIII or IX is preferred. 3 Antifibrinolytic drugs are the mainstay of therapy. 4 Replacement therapy should only occur during acute episodes.

2) Treatment with factor VIII or IX is preferred. Treatment with factor replacement is the preferred treatment for patients with hemophilia. Replacement therapy is prescribed prophylactically. Antifibrinolytic drugs are used as adjunct treatment. Medication therapy does not cure the patient.

A patient who has been taking warfarin [Coumadin] is admitted with coffee-ground emesis. What can the nurse anticipate being prescribed for this patient? 1 Vitamin E 2 Vitamin K 3 Protamine sulfate 4 Calcium gluconate

2) Vitamin K Vitamin K is the antagonist for warfarin. If a heparin overdose occurs, the antidote is protamine sulfate. Vitamin E and calcium gluconate are not used to counteract the effects of warfarin.

A patient has received a medication that constricts the blood vessels. What effect will this have on the blood pressure? 1 Will decrease 2 Will be elevated 3 Will stay the same 4 Will fluctuate wildly

2) Will be elevated To maintain adequate flow when resistance (constriction) rises, blood pressure must rise as well. Dilation will cause the blood pressure to decrease. The blood pressure will not stay the same nor will it fluctuate wildly.

Which needle length and gauge should the nurse choose to administer subcutaneous heparin? 1 1½ inch; 20 gauge 2 ⅝ inch; 25 gauge 3 1½ inch; 18 gauge 4 1 inch; 26 gauge

2) ⅝ inch; 25 gauge Heparin should be administered subcutaneously into the fatty layer of the abdomen with a ½- to ⅝-inch needle, 25 or 26 gauge.

A patient asks the nurse for information about cytomegalovirus. Which is the most accurate response by the nurse? 1 "About 60% of people have the disease." 2 "Cytomegalovirus retinitis is the principal cause of loss of vision." 3 "After the initial infection, the virus stays dormant in the body for life." 4 "The mode of transmission for cytomegalovirus is airborne transmission."

3) "After the initial infection, the virus stays dormant in the body for life." After the initial infection, which usually has mild symptoms in healthy people, the virus remains dormant in cells for life without causing any problems. Between 50% and 85% of adults in America 40 years and older harbor the virus. Cytomegalovirus (CMV) retinitis is a reason for loss of vision in persons with AIDS and CMV. The mode of transmission for CMV is contact with contaminated body fluids, including blood, urine, and breast milk.

The nurse is teaching a patient whose dysrhythmia is being treated with medication therapy. Which fact should the nurse include in the teaching plan? 1 "Medication is only taken during the time the dysrhythmia is occurring." 2 "Medication therapy often does not work for dysrhythmias." 3 "Almost all drugs used to treat dysrhythmias can cause dysrhythmias." 4 "Drug therapy is only used for bradydysrhythmias."

3) "Almost all drugs used to treat dysrhythmias can cause dysrhythmias." The patient should understand that almost all drugs used to treat dysrhythmias can cause dysrhythmias. The nurse should teach the patient the effects of the drug as well as how to assess their pulse rate.

A nurse is teaching a group of patients regarding flu season in the United States. Which statement should the nurse include in the teaching? 1 "To ensure full protection, the best time to vaccinate is September." 2 "In the United States, flu season usually peaks in October or November." 3 "For people who missed the best time, vaccinating as late as April may be of help." 4 "The influenza vaccine may not be administered at the same time as the pneumococcal vaccine."

3) "For people who missed the best time, vaccinating as late as April may be of help." Peak flu season in the United States is usually January or February. To ensure full protection, the best time to vaccinate is October or November. For people who missed the best time, vaccinating as late as April may be helpful. The influenza vaccine may be given at the same time as other vaccines, including the pneumococcal vaccine.

The healthcare provider has prescribed amiodarone for a patient with ventricular dysrhythmias. Upon reviewing the patient's medical history, which statement by the patient would lead the nurse to notify the healthcare provider? 1 "I get faint when I see needles." 2 "I wear eyeglasses to read print up close." 3 "I take a blood thinner so I don't get clots." 4 "I break out in a rash when I take antibiotics."

3) "I take a blood thinner so I don't get clots." Amiodarone has two very significant drug interactions, namely with digoxin and warfarin. Patients often refer to anticoagulants as "blood thinners." The nurse should verify which medication the patient is taking, then notify the healthcare provider. When amiodarone is prescribed for patients taking warfarin, the recommendation is to reduce the dose of warfarin by 50% at the start of amiodarone therapy. Amiodarone adverse effects include visual halos, photophobia, and dry eyes, but wearing eyeglasses is not a contraindication. Amiodarone is an antidysrhythmic, and there should be no cross sensitivities with antibiotics. The patient may get faint at the site of needles; however, the priority of care is to eliminate dysrhythmias. To help reduce the patient's fears, the nurse can provide emotional reassurance and compassionately administer the amiodarone.

Which statement made by the patient indicates to the nurse that understanding of the discharge instructions on antihyperlipidemic medications has occurred? 1 "I will stop taking the medication if it causes nausea and vomiting." 2 "It is important to double my dose if I miss one in order to maintain therapeutic blood levels." 3 "I will continue to modify my diet and keep exercising to help increase my high-density lipoprotein serum levels." 4 "Antihyperlipidemic medications will replace the other interventions I have been doing to try to decrease my cholesterol."

3) "I will continue to modify my diet and keep exercising to help increase my high-density lipoprotein serum levels." Antihyperlipidemic medications are an addition to, not a replacement for, the therapeutic regimen of diet modification combined with exercise that is used to decrease serum cholesterol levels. The dose should never be doubled if one is missed nor stopped due to side effects. If the medication causes nausea or vomiting, the healthcare provider should be notified.

A patient is receiving warfarin [Coumadin] for a chronic condition. Which patient statement requires immediate action by the nurse? 1 "I will avoid contact sports." 2 "I will take my medication at the same time each day." 3 "I will increase dark green, leafy vegetables in my diet." 4 "I will contact my healthcare provider if I develop excessive bruising."

3) "I will increase dark green, leafy vegetables in my diet." Dark green, leafy vegetables are rich in vitamin K, which would antagonize the effects of warfarin. Rather than increase the intake of these vegetables, it is more important to maintain a consistent daily intake of vitamin K. The patient should monitor his or her incidence of bruising carefully. The medication will usually be ordered to be taken at the same time each day.

In which patient would a low-dose aspirin be contraindicated? 1 A patient with thrombosis 2 A patient with a heart problem 3 A patient with a hemorrhagic stroke 4 A patient with a deep vein thrombosis

3) A patient with a hemorrhagic stroke The patient contraindicated to take a low-dose aspirin is the patient with a hemorrhagic stroke. The patient with a thrombosis, deep vein thrombosis, and a heart problem would benefit from a low-dose aspirin.

Which patient will benefit most from prophylactic iron therapy? 1 A teenage girl 2 An elderly patient 3 A pregnant woman 4 An adolescent boy

3) A pregnant woman Pregnant women are the principal candidates for prophylactic iron therapy as iron is needed for fetal growth. Other candidates are infants, children, and women experiencing menorrhagia.

The nurse will most likely administer a combination drug of hydralazine and isosorbide dinitrate [BiDil] to patients belonging to which ethnic group? 1 Asian American 2 Native American 3 African American 4 Mexican American

3) African American Hydralazine is also available in fixed-dose combination with isosorbide dinitrate (a vasodilator) sold as BiDil. BiDil is the first drug product approved for treating a specific ethnic group—namely, African Americans.

The nurse is instructing a patient about potential adverse effects of a prescribed angiotensin-converting enzyme (ACE) inhibitor. For which adverse effect should the patient seek immediate medical attention? 1 Fatigue 2 Diarrhea 3 Angioedema 4 Dry, nonproductive cough

3) Angioedema Angioedema is a potentially fatal reaction that develops in up to 1% of patients. A dry, nonproductive cough is an adverse reaction but is not life-threatening. If fatigue does occur, it is not life-threatening. Diarrhea is not an adverse effect.

The renin-angiotensin-aldosterone system plays an important role in maintaining blood pressure. Which compound in this system is most powerful at raising blood pressure? 1 Renin 2 Angiotensin I 3 Angiotensin II 4 Angiotensin III

3) Angiotensin II Angiotensin II is a potent vasoconstrictor. It participates in all the pathways regulated by the renin-angiotensin-aldosterone system. Angiotensin I is a precursor to angiotensin II; angiotensin III is formed by degradation of angiotensin II and is less potent. Renin catalyzes the conversion of angiotensinogen to angiotensin I.

The nurse is teaching a patient about therapy with anticoagulants. What is essential information to include in the teaching plan? 1 Anticoagulants dissolve clots. 2 Anticoagulants alter platelet function. 3 Anticoagulants prevent clots from forming. 4 Anticoagulants interfere with drug metabolism.

3) Anticoagulants prevent clots from forming. The patient needs to understand that anticoagulants will prevent new clots from forming but will not dissolve clots that are already formed. Anticoagulants inhibit clotting by acting on clotting factors and do not alter platelets or drug metabolism.

Which drug is the most effective for lowering low-density lipoprotein (LDL) cholesterol? 1 Gemfibrozil 2 Ezetimibe [Zetia] 3 Atorvastatin [Lipitor] 4 Cholestyramine [Questran]

3) Atorvastatin [Lipitor] The statin drugs, such as atorvastatin [Lipitor], are the most effective drugs for lowering LDL cholesterol. They are better tolerated, have fewer adverse effects, and produce better clinical outcomes than any other agents available for lowering LDL. Cholestyramine [Questran] , Gemfibrozil, and Ezetimibe [Zetia] are also cholesterol-lowering agents but are not classified as statins.

The nurse is administering captopril to a patient with heart failure. Which best describes the effects of this drug? 1 Constricts arterioles and veins 2 Increases release of aldosterone 3 Blocks production of angiotensin II 4 Increases production of angiotensin II

3) Blocks production of angiotensin II Captopril an angiotensin-converting enzyme (ACE) inhibitor, blocks production of angiotensin II, decreases release of aldosterone, and suppresses degradation of kinins. By suppressing production of angiotensin II, ACE inhibitors cause dilation of arterioles and veins.

Which assessment finding is most important for the nurse to obtain before administering hydralazine [Apresoline]? 1 Capillary refill 2 Homans' sign 3 Blood pressure 4 Peripheral pulses

3) Blood pressure Hydralazine [Apresoline] is a vasodilator that causes arteriolar dilation, decreased resistance, and decreased blood pressure. Monitoring of the blood pressure and heart rate is the highest assessment priority.

A staff member asks how calcium channel blockers (CCBs) work. Which response should the nurse provide? 1 CCBs increase arterial pressure. 2 CCBs block reabsorption of sodium. 3 CCBs relax vascular smooth muscle. 4 CCBs have a positive inotropic effect.

3) CCBs relax vascular smooth muscle By blocking calcium channels, CCBs inhibit muscle contraction in the coronary arteries and peripheral arteries, resulting in relaxed vascular smooth muscle (vasodilation). CCBs have a negative inotropic effect by blocking calcium entry, which decreases the force of contractions. CCBs have no direct role in sodium activity. Vasodilation from CCBs decrease arterial pressure.

Which assessment finding could indicate a myocardial infarction (MI) in a patient who is taking nitroglycerin [Nitrostat] for angina? 1 The patient has a headache after taking the drug. 2 There is sudden and severe hypotension in the patient. 3 Chest pain is not relieved by administering nitroglycerin. 4 The patient complains of feeling dizzy when trying to move.

3) Chest pain is not relieved by administering nitroglycerin. If chest pain is not relieved after taking nitroglycerin [Nitrostat], it indicates possible MI in the patient. In this case, the patient needs to report to the emergency department immediately to seek prompt treatment. Headache is a common side effect of nitroglycerin. It diminishes soon after the therapy is started. The patient feels dizzy due to the drug-induced hypotension. However, the patient can be asked to restrict movement or turn slowly to avoid dizziness. Sudden and severe hypotension is seen when a patient is administered intravenous nitroglycerin [Nitro-Bid] rapidly.

A patient who was admitted with deep vein thrombophlebitis is complaining of difficulty breathing and chest pain. What is the most likely cause of these symptoms? 1 Anxiety 2 Medication reaction 3 Embolus to the lungs 4 Fatigue from the extra work of walking with pain

3) Embolus to the lungs A thrombus can become an embolus and travel to the lungs. This pulmonary embolus can cause chest pain and difficulty breathing. It is not likely that fatigue, anxiety, or a medication reaction would cause chest pain and dyspnea in this case.

What is the mechanism of action of ezetimibe [Zetia]? 1 It inhibits the biosynthesis of cholesterol in the liver. 2 It decreases the adhesion of cholesterol on the arterial walls. 3 It inhibits absorption of dietary and biliary cholesterol in the small intestine. 4 It inhibits the absorption of bile thus causing the liver to produce bile from cholesterol.

3) It inhibits absorption of dietary and biliary cholesterol in the small intestine. Ezetimibe [Zetia] works by selectively inhibiting the absorption of cholesterol and related sterols in the small intestine. The mechanism of action of ezetimibe [Zetia] does not involve the liver or arterial walls.

A patient's blood pressure is 200/120 mm Hg, and the provider starts the patient on sodium nitroprusside. What is the appropriate action by the nurse? 1 Measure hourly output. 2 Administer with a full glass of water. 3 Monitor blood pressure continuously. 4 Make certain the patient does not crush the pill.

3) Monitor blood pressure continuously. During nitroprusside infusion, blood pressure should be monitored continuously with either an arterial line or an electronic monitoring device. Hourly outputs are not required. The drug is administered intravenously, not orally.

A patient is taking pravastatin sodium [Pravachol]. Which assessment finding requires immediate action by the nurse? 1 Fatigue 2 Headache 3 Muscle pain 4 Slight nausea

3) Muscle pain Patients who experience severe muscle pain while taking pravastatin sodium [Pravachol] need to report the findings right away as this may be indicative of rhabdomyolysis, a muscle disintegration that can become fatal. Headaches, slight nausea, and fatigue are not high priority symptoms and may not be caused by the medication. Muscle pain is the only one that is most likely caused by the medication.

The patient has been placed on aspirin as an antiplatelet drug. Which side effect is the patient most likely to experience? 1) Itching 2) Edema 3) Nausea 4) Chest pain

3) Nausea Nausea accompanied by vomiting is an expected side effect of treatment with aspirin. Chest pain, edema, and itching are side effects that are more likely to be seen with the use of clopidogrel.

What does the nurse administer when a patient has a sudden attack of angina? 1 Atenolol 2 Nifedipine 3 Nitroglycerin [Nitrostat] 4 Transdermal nitroglycerin patch

3) Nitroglycerin [Nitrostat] Sublingual nitroglycerin works quickly to relieve pain and cause vasodilation during an angina attack. A nitroglycerin transdermal patch has an onset of action of 30 to 60 minutes and may be ineffective in relieving a sudden attack of angina. The calcium channel blocker is used for cerebral artery spasms associated with aneurysm rupture. Atenolol [Tenormin] is available only in the oral form and is more effective in patients after myocardial infarction.

The nurse is caring for a patient who is scheduled to begin treatment with carvedilol [Coreg]. While updating the history, the patient tells the nurse that he experiences frequent attacks of asthma. What is the nurse's highest priority action? 1 Expect a decreased effect from the medication. 2 Expect an increased effect from the medication. 3 Notify the healthcare provider of this information. 4 Monitor the patient for a toxic reaction to the drug.

3) Notify the healthcare provider of this information. Carvedilol [Coreg] should be used with caution in patients with a history of asthma. The priority for the nurse is to notify the healthcare provider of this information.

The nurse reviews the prescribed medication for a patient with hemophilia who complains of bleeding-related pain. Which medication should the nurse question if ordered for this patient? 1 Codeine 2 Tylenol (acetaminophen) 3 Percodan (oxycodone/aspirin) 4 MS Contin (time released morphine sulfate)

3) Percodan (oxycodone/aspirin) The nurse should question the order for Percodan because this drug contains 325 mg of aspirin. Aspirin should be avoided in bleeding-related pain management because it causes irreversible inhibition of platelet aggregation and thus increases the risk of bleeding. Severe pain can be treated with opioids such as codeine and MS Contin. Mild pain can be treated with Tylenol.

Which drug is classified as an adrenergic antagonist? 1 Propofol [Diprivan] 2 Milrinone [Primacor] 3 Propranolol [Inderal] 4 Epinephrine [Adrenaline]

3) Propranolol [Inderal] Propranolol is an adrenergic antagonist. Propofol comes under the class of anesthetic agents. Milrinone is classified as an inotropic phosphodiesterase type 3 enzyme inhibitor. Epinephrine comes under the class of adrenergic agonists.

Which structure in the heart is referred to as the pacemaker? 1 Bundle of His 2 Purkinje fibers 3 Sinoatrial (SA) node 4 Atrioventricular (AV) node

3) Sinoatrial (SA) node The sinoatrial (SA) node is the pacemaker because it can spontaneously depolarize easier and faster than the other area

The nurse is caring for a patient who is scheduled to receive a first dose of an alpha adrenergic antagonist drug. The nurse is prepared to monitor the patient for which effect? 1 Rash 2 Anorexia 3 Syncope 4 Vomiting

3) Syncope (temporary loss of consciousness caused by a fall in blood pressure) First-dose syncope is associated with alpha adrenergic antagonists.

A 79-year-old patient who received a beta-adrenergic agonist complains of chest pain. What is the nurse's priority action? 1 Take the patient's temperature. 2 Count the patient's respiratory rate. 3 Take the patient's blood pressure and pulse. 4 Obtain a 12-lead electrocardiogram (ECG) at the bedside.

3) Take the patient's blood pressure and pulse. Severe physiologic changes occur in the cardiovascular system of the older adult. Therefore close monitoring is required when administering adrenergic medications. The nurse should first assess the chest pain and take vital signs, especially blood pressure and pulse. The patient's temperature and respiratory rate are less affected by adrenergic medications than blood pressure and pulse. After obtaining vital signs and assessing the pain, the nurse should notify the healthcare provider for further orders such as obtaining an ECG.

A patient with hypercholesterolemia is prescribed lovastatin [Mevacor]. After reviewing the patient's medical history, the nurse discovers that the medication is not safe to prescribe for the patient and reports this finding to the healthcare provider. What did the nurse find in the patient's medical history? 1 The patient has leukemia. 2 The patient has renal disease. 3 The patient has hepatic disease. 4 The patient has chronic pulmonary disease.

3) The patient has hepatic disease Lovastatin [Mevacor] can cause an increase in liver enzymes and should not be prescribed to patients with preexisting liver disease. Statins induce cell death in malignant cells. Cell death occurs via apoptosis and lovastatin [Mevacor] concentrations are used in the treatment of leukemia. Statins slow down the progress of chronic kidney disease by reducing kidney inflammation or improving the function of kidney tissues. Statins reduce chronic obstructive pulmonary disease (COPD).Lovastatin [Mevacor] can be prescribed to the patient with leukemia, renal disease and COPD.

Which patient finding would cause the nurse to hold the 8 AM dose of verapamil [Calan]? 1 The patient has atrial flutter. 2 The patient has tachycardia. 3 The patient has hypotension. 4 The patient has angina pectoris.

3) The patient has hypotension. A patient who is scheduled to receive verapamil [Calan], a calcium channel blocker (CCB), should have blood pressure and pulse assessed prior to administration. Hypotension would be a reason to hold the drug. Verapamil [Calan] is given for atrial flutter. Patients with angina may receive CCBs for their vasodilating effects. Tachycardia is not a reason to hold the drug; CCBs decrease the heart rate.

The nurse is caring for a patient with iron deficiency anemia. What should the nurse teach this patient? 1 This is a very rare anemia. 2 There are no risks from this deficit. 3 This disease is caused by a nutritional deficit. 4 There is no treatment needed for this condition.

3) This disease is caused by a nutritional deficit. Iron deficiency anemia is the most common nutritional deficiency and the most common cause of nutrition-related anemia. It is extremely common. The risks from this anemia are just as serious as any type of anemia.

The heart undergoes cardiac remodeling during the initial phase of heart failure. Which structural change occurs with heart failure? 1 Ventricular atrophy 2 Ventricular constriction 3 Ventricular wall thickening 4 Ventricles become more cylindric

3) Ventricular wall thickening An increase in ventricular wall thickness, also called ventricular hypertrophy, is characteristic of the remodeling process during the initial phase of heart failure. The ventricles also dilate and become more spherical (less cylindric). This change in cardiac shape typically occurs after cardiac injury under the influence of the neurohormonal systems, such as the sympathetic nervous system and renin-angiotensin-aldosterone system.

Which instructions should the nurse include in the plan of care for a patient receiving ferrous sulfate therapy? 1 "Iron does not absorb." 2 "Iron should only be taken at night." 3 "Iron compounds are not taken orally." 4 "Antacids should not be taken with iron."

4) "Antacids should not be taken with iron." Antacids can decrease the absorption of iron. Iron is typically taken in the morning. It is absorbed readily and is taken orally.

Which statement made to the patient will demonstrate the nurse's knowledge of calcium-channel blockers (CCBs) for management of hypertension? 1 "CCBs promote excretion of water." 2 "CCBs block reabsorption of sodium." 3 "CCBs increase blood return to heart." 4 "CCBs relax vascular smooth muscle."

4) "CCBs relax vascular smooth muscle." By blocking calcium channels, CCBs inhibit muscle contraction in the coronary arteries and peripheral arteries resulting in vasodilation, improved blood flow to the myocardium, and decreased blood pressure. CCBs have no direct role in sodium or water activity. Since CCBs decrease blood return to the heart, preload is reduced.

The patient asks the nurse to explain the difference between dalteparin [Fragmin] and heparin. Which response by the nurse is accurate? 1 "There is no real difference. Dalteparin is interchangeable with heparin." 2 "The only difference is that heparin dosing is based on the patient's weight." 3 "I'm not sure why some healthcare providers choose dalteparin and some heparin. You should ask your doctor." 4 "Dalteparin is a low-molecular-weight heparin that is more predictable in its effect and has a lower risk of bleeding."

4) "Dalteparin is a low-molecular-weight heparin that is more predictable in its effect and has a lower risk of bleeding." A low-molecular-weight heparin is more predictable in its effect than regular heparin. Dalteparin [Fragmin] is not interchangeable with heparin and is dosed based on the patient's weight. It is not appropriate to provide the patient with no instruction other than to simply refer the patient to the healthcare provider.

Which comment by the patient indicates understanding about the use of enalapril [Vasotec] for treatment of hypertension? 1 "I cannot go out in the sun while on this therapy." 2 "I should stop the drug if I have ringing in my ears." 3 "If I feel tired, I should call the healthcare provider." 4 "If I develop a chronic cough, I need to notify my healthcare provider."

4) "If I develop a chronic cough, I need to notify my healthcare provider." The patient on therapy with an angiotensin-converting enzyme inhibitor such as enalapril needs to report a nonproductive chronic cough as this is a potential side effect. There is no treatment other than to change the medication therapy. The other statements are not correct.

Which comment by a patient indicates correct understanding about the use of enalapril? 1 "If I feel tired, I should double the dose." 2 "I cannot go out in the sun while on this therapy." 3 "I should stop the drug if I have ringing in my ears." 4 "If I develop a chronic cough, I need to notify my provider."

4) "If I develop a chronic cough, I need to notify my provider." A patient on therapy with an angiotensin-converting enzyme (ACE) inhibitor such as enalapril should report a nonproductive chronic cough, as this is a potential side effect. There is no treatment other than to change the medication therapy. The patient should not double the dose of an antihypertensive. Ringing in the ears in not a concern for ACE inhibitors and the patient need not avoid the sun.

What is the priority instruction a nurse gives to a male patient who is scheduled to receive ribavirin [Rebetol] combined with peginterferon-alfa-2a for treatment of hepatitis C? 1 "Combining these two medications will greatly increase response rates." 2 "It is very important that you have blood counts checked every 2 weeks." 3 "An antidepressant can be prescribed to alleviate symptoms of depression." 4 "If you are sexually active, use two reliable forms of birth control to prevent pregnancy."

4) "If you are sexually active, use two reliable forms of birth control to prevent pregnancy." Cidofovir [Vistide] is an intravenous (IV) drug with just one indication: cytomegalovirus (CMV) retinitis in patients with AIDS who have failed on ganciclovir or foscarnet. Acyclovir [Zovirax] is effective against herpes simplex virus types 1 and 2, herpes zoster, and chickenpox. Ribavirin [Rebetol] is used to treat respiratory syncytial virus infection. Amantadine [Symmetrel] is an antiviral drug developed to treat influenza A but is no longer recommended for this purpose and is now used to treat Parkinson's disease.

The nurse teaches the patient about his essential hypertension. What statement should be included in the teaching plan? 1 "Once you have surgery, this will go away." 2 "This type of hypertension has a definitive cause." 3 "This type of hypertension is not able to be treated." 4 "The cause of your hypertension is unknown but treatable."

4) "The cause of your hypertension is unknown but treatable." The specific cause of essential (primary) hypertension is unknown. Surgical treatment may cure secondary hypertension but not idiopathic hypertension.

The nurse is teaching the patient about the reason for the administration of calcium channel blockers. What statement will be included in the teaching plan? 1 "This medication will help you to get rid of sodium." 2 "This medication will work to cause you to get rid of fluid." 3 "This medication will help you to lose weight to lower your blood pressure." 4 "This medication will enable vasodilation of your blood vessels to lower your blood pressure."

4) "This medication will enable vasodilation of your blood vessels to lower your blood pressure." This medication causes vasodilation and is used in hypertension to lower blood pressure. It causes direct vasodilation by blocking calcium influx in smooth muscles in the blood vessels. This medication class does not help to rid fluids, decrease sodium, or help the patient to lose weight.

For his asthma, a patient is prescribed albuterol in the form of an inhalant. The patient tells the nurse that the medication causes shakes and hand tremors. What is the nurse's best response? 1 "I will contact your provider about your problem." 2 "The tremors are probably not related to the medication." 3 "I will contact the respiratory therapist about your problem." 4 "Tremors are the most common side effect of this inhalant and will usually fade over time."

4) "Tremors are the most common side effect of this inhalant and will usually fade over time." "Tremors are the most common side effect of this inhalant and will usually fade over time" is the nurse's best response. Initiating therapy at lower doses can also help alleviate tremors. It is important to contact the respiratory therapist and physician if the patient is having any adverse effects that do not subside.

A patient cannot take oral cyanocobalamin. Which is an appropriate intervention? 1 Administer an antidote. 2 Administer an antihistimine. 3 Prepare the patient for surgery. 4 Administer intranasal cyanocobalamin.

4) Administer intranasal cyanocobalamin. Intranasal administration is an alternative to to oral, intramuscular, or even subcutaneous injection for people who cannot take cyanocobalamin by mouth. There is no need for surgery, an antihistamine, or antidote.

Which treatment should be included in a plan for a patient with mild joint pain related to hemophilia? 1 Administration of aspirin 2 Administration of narcotics 3 Administration of ibuprofen [Motrin] 4 Administration of acetaminophen [Tylenol]

4) Administration of acetaminophen [Tylenol] For mild pain, acetaminophen [Tylenol] is the drug of choice. For severe pain, an opioid analgesic may be needed. Regardless of pain severity, aspirin should be avoided as it causes irreversible inhibition of platelet aggregation and can thus increase bleeding risk. Aspirin can also induce gastrointestinal ulceration and bleeding, an obvious problem.

A patient is receiving isosorbide dinitrate [Isordil] 20 mg 3 times per day for management of newly diagnosed stable angina. Which assessment finding would require an immediate nursing intervention? 1 Report of increased frequency of urination 2 A change in blood pressure from 122/70 to 108/66 mm Hg 3 A headache the patient rates as a 5 on a pain scale of 0 to 10 4 An increase in the resting heart rate to 110 beats/min from baseline rates of 72 beats/min

4) An increase in the resting heart rate to 110 beats/min from baseline rates of 72 beats/min Because nitrates lower blood pressure, isosorbide dinitrate [Isordil] can activate the baroreceptor reflex, causing sympathetic stimulation of the heart; this negates the benefits of treatment with nitrates, because it increases the cardiac oxygen demand. For these reasons, addressing the tachycardia becomes the nurse's immediate priority. A decrease in blood pressure would be expected, and there is no indication the patient is experiencing side effects of decreased cardiac output. Headache is an adverse effect and should be treated (eg, with acetaminophen [Tylenol]), but it does not require immediate intervention. Increased frequency of urination requires follow-up, but it is not the immediate concern.

The nurse is preparing to administer a daily dose of digoxin [Lanoxin]. What is the priority nursing intervention? 1 Check blood pressure. 2 Palpate the pedal pulses. 3 Assess for Homans' sign. 4 Analyze heart rate and rhythm.

4) Analyze heart rate and rhythm. Before giving digoxin [Lanoxin], the nurse should assess heart rate and rhythm. The dosage will be held and the prescriber notified if the heart rate is below 60 beats/min or if the cardiac rhythm has changed. Digoxin [Lanoxin] can cause bradycardia and electrical changes in the heart.

The home health nurse observes the patient for proper use of topical nitroglycerin ointment. Which action by the patient indicates the need for further teaching? 1 Rotates the application sites to minimize skin irritation 2 Uses the applicator paper to measure the prescribed dose 3 Removes ointment from a previous dose before applying the next dose 4 Applies the prescribed ribbon of ointment to the applicator paper and always places it over the heart

4) Applies the prescribed ribbon of ointment to the applicator paper and always places it over the heart Once the prescribed ribbon of ointment has been squeezed onto the applicator paper provided, the paper is used to spread the ointment over an area at least 2.5 by 3.5 inches. It is then covered with plastic wrap to prevent the ointment from being absorbed into the clothing. There is no mechanistic advantage to applying the ointment over the heart. The ointment can be applied to the back, abdomen, or anterior thigh, as well as the chest. Sites should be rotated to minimize skin irritation.

In what part of the conduction pathway in a healthy heart is the electrical impulse delayed to provide time for the blood to fill the ventricles? 1 Bundle of His 2 Purkinje system 3 Internodal pathways 4 Atrioventricular node

4) Atrioventricular node Impulses originate in the sinoatrial (SA) node and then travel through the atrioventricular (AV) node to reach the ventricles. The impulse is delayed at the AV node to provide time for the ventricles to fill before they contract.

A patient has received three times the ordered dose of propafenone [Rythmol]. The charge nurse is called into the room and observes that the patient is unresponsive with no pulse. What is the nurse's priority action? 1 Notify the provider. 2 Place the patient on cardiac monitoring. 3 Start an IV line with a large-bore catheter. 4 Begin cardiopulmonary resuscitation efforts.

4) Begin cardiopulmonary resuscitation efforts. The main effects of the antidysrhythmics involve the heart, circulation, and central nervous system (CNS). Management of an overdose involves maintaining adequate circulation and respiration using general support measures such as cardiopulmonary resuscitation (CPR) and providing any required symptomatic treatment. Early chest compressions and defibrillation, as indicated, are recommended as soon as possible by the American Heart Association 2010 guidelines.

The nurse administers candesartan [Atacand] to a patient. Which patient assessment finding should the nurse use as a clinical indicator of therapeutic effectiveness of the medication? 1 Cool, dry, pale extremities 2 Serum sodium 140 mEq/L 3 Serum potassium 3.8 mEq/L 4 Blood pressure 120/72 mm Hg

4) Blood pressure 120/72 mm Hg The nurse uses the patient's blood pressure, which is within normal limits, to determine that an angiotensin II receptor blocker (ARB) is effective because they decrease blood pressure by causing dilation of arterioles and veins. Because ARBs promote vasodilation, the nurse expects the patient's extremities to be warm and pink. The patient's sodium level is normal; because ARBs block the secretion of aldosterone, the patient's sodium level is likely to be unaffected by ARB administration. The patient's potassium level is normal; ARBs are less likely than ACE inhibitors to affect the potassium level.

The nurse administers clonidine [Catapres]. What finding indicates the medication is therapeutic? 1 There is increased urinary output. 2 There is an absence of chest pain. 3 The heart rate decreases from 100 beats/min to 40 beats/min. 4 Blood pressure decreases from 150/100 mm Hg to 110/70 mm Hg.

4) Blood pressure decreases from 150/100 mm Hg to 110/70 mm Hg. Clonidine is used primarily for its ability to decrease blood pressure. It is not used for angina or to decrease the heart rate. It is not a diuretic and will not increase urinary output.

The nurse is administering an antihypertensive medication. What assessment finding requires immediate action? 1 Calcium level of 8 mEq/dL 2 Potassium level of 5 mEq/dL 3 Apical pulse of 100 beats/min 4 Blood pressure of 80/60 mm Hg

4) Blood pressure of 80/60 mm Hg Blood pressure that goes below 100 mm Hg should immediately be reported to the healthcare provider, and the medication should be held. The other assessment findings are within normal limits and do not require immediate action.

The nurse is caring for a patient taking foscarnet [Foscavir]. What should the nurse monitor to identify potential side effects of this medication? 1 Platelets 2 Stool guaiac 3 Hemoglobin and hematocrit 4 Blood urea nitrogen and creatinine

4) Blood urea nitrogen and creatinine Foscarnet [Foscavir] can cause kidney damage, so blood urea nitrogen and creatinine should be closely monitored. Hemoglobin, hematocrit, and platelets have to do with blood levels. The stool guaiac test is to check for blood in the stool. These tests do not have anything to do with Foscarnet [Foscavir].

A patient who has received iron dextran is complaining of chest pain. What is the nurse's best action? 1 Call a code. 2 Administer an antacid. 3 Administer a respiratory treatment. 4 Consult with the healthcare provider to assess the ECG.

4) Consult with the healthcare provider to assess the ECG. Circulatory failure and cardiac arrest can occur in the administrationration of iron dextran. The nurse should consult with the healthcare provider to assess the electrocardiogram (ECG) to look for a myocardial infarction. The nurse should also assess the patient's pain intensity.

When teaching a patient about amiodarone, the nurse should advise the patient to avoid which food or drink? 1 Gluten 2 Poultry 3 Whole milk 4 Grapefruit juice

4) Grapefruit juice Grapefruit juice can inhibit the metabolism of several antidysrhythmics, such as amiodarone, disopyramide, and quinidine. It is not necessary to restrict gluten, poultry, and whole milk from the patient's diet.

Which antifungal drug is contraindicated in a patient with porphyrias? 1 Fluconazole [Diflucan] 2 Ketoconazole [Nizoral, Extina] 3 Terbinafine [Lamisil, Lamisil AT] 4 Griseofulvin [Grifulvin V, Gris-PEG]

4) Griseofulvin [Grifulvin V, Gris-PEG] Griseofulvin [Grifulvin V, Gris-PEG] is administered orally to treat superficial mycoses. The drug is inactive against organisms that cause systemic mycoses. The use of griseofulvin [Grifulvin V, Gris-PEG] is contraindicated in patients with porphyrias, which are disorders affecting the skin caused by a deficiency in a liver enzyme involved in the heme biosynthetic pathway. Griseofulvin [Grifulvin V, Gris-PEG] administration in patients with these disorders has been found to precipitate an acute worsening of the condition. Terbinafine [Lamisil, Lamisil AT], fluconazole [Diflucan], and ketoconazole [Nizoral, Extina] have not been known to worsen this condition; therefore, they are not contraindicated in porphyrias.

What should the nurse recommend in relation to immunizations for a child with hemophilia? 1 Start immunizations as an adult. 2 Administer only inhaled immunizations. 3 Avoid immunization due to the risk of bleeding. 4 Have immunizations administered intramuscularly.

4) Have immunizations administered intramuscularly. Children with hemophilia should undergo the normal immunization schedule. Some clinicians inject vaccines subQ, rather than IM, to avoid muscle hemorrhage. However, since the efficacy of subQ vaccination is not certain, and since most patients tolerate IM injections without bleeding, IM vaccination is generally preferred. The risk of bleeding after IM injection can be reduced by prolonged application of pressure.

A patient is taking verapamil [Calan]. Which instruction should the nurse give the patient to help prevent a common adverse effect of this drug? 1 Use sunscreen. 2 Increase calorie intake. 3 Use a skin barrier cream. 4 Increase fluid and fiber intake.

4) Increase fluid and fiber intake. Constipation may occur with verapamil [Calan]. Advise patients that constipation can be minimized by increasing dietary fluid and fiber. Verapamil [Calan] does not cause photosensitivity, so sunscreen is not needed. Verapamil [Calan] does not cause weight loss, so an increase in calories is not needed. Verapamil [Calan] does not cause diarrhea, so a skin barrier cream is not needed.

Which outcome would be most appropriate to establish for patients taking beta1 agonists? 1 Relaxation of uterine smooth muscle 2 Relaxation of bronchial smooth muscle 3 Increased resistance of peripheral vessels 4 Increased contractility of the myocardium

4) Increased contractility of the myocardium Stimulation of beta1 receptors in the "fight-or-flight" response results in stronger ventricular contraction owing to increased contractility of the myocardium. Beta2 agonist stimulation results in vasodilation and decreased resistance of peripheral vessels. Beta1 agonists act most specifically on the heart; relaxation of uterine smooth muscle is a result of taking alpha1 antagonists. Bronchial smooth muscle is affected by beta2 agonists.

The nurse is monitoring a patient receiving ferrous sulfate for treatment of iron deficiency anemia. Which finding indicates that the therapy is effective? 1 Black stools 2 Decrease in energy 3 Absence of emesis 4 Increased hemoglobin

4) Increased hemoglobin A patient who has increased hemoglobin would be evaluated as having a positive response to ferrous sulfate therapy. Increase in energy would also be therapeutic. Black stools are a side effect of therapy. Emesis is not a measure of therapeutic effect.

Which approach should a nurse take to administer intravenous acyclovir [Zovirax] to an immunocompromised patient? 1 Ask the provider to change the route to subcutaneous injection. 2 Only infuse the drug if the white blood cell (WBC) count is above 2500/mm3. 3 Administer intravenous acyclovir diluted in 20 mL normal saline over 10 minutes. 4 Infuse intravenous fluids during administration of the dose and for 2 hours afterward.

4) Infuse intravenous fluids during administration of the dose and for 2 hours afterward. Acyclovir can cause renal damage, and hydration of the patient during the infusion and for 2 hours afterward minimizes this risk. Acyclovir is not given by intravenous (IV) bolus or by intramuscular (IM) or subcutaneous (subQ) injection. It is used to treat infections in immunocompromised patients, but the dose is not changed or affected by the WBC count.

Which adverse effect is the greatest concern with thrombolytic therapy? 1 Hemophilia 2 Anaphylaxis 3 Cerebral edema 4 Intracranial hemorrhage

4) Intracranial hemorrhage The major complication of thrombolytic therapy is bleeding. Intracranial hemorrhage (ICH) is the greatest concern.

Which is the most appropriate antifungal agent for treating a patient with an aspergillosis lung infection? 1 Flucytosine [Ancobon] 2 Ketoconazole [Nizoral] 3 Griseofulvin [Gris-PEG] 4 Itraconazole [Sporanox]

4) Itraconazole [Sporanox] Itraconazole [Sporanox] is an effective treatment for an aspergillosis lung infection because it is widely dispersed in body tissue and is active against this fungus. Flucytosine [Ancobon], griseofulvin [Gris-PEG], and ketoconazole [Nizoral] are not effective drugs for aspergillosis infections because the fungi are resistant to these drugs.

The patient is an older adult who has been placed on a beta-adrenergic agonist. The patient complains to the nurse of experiencing angina after being on the drug for 12 hours. What is the nurse's highest priority action? 1 Question the patient to see if angina has been experienced in the past. 2 Reassure the patient that this is an expected side effect of the medication. 3 Ask the patient what over-the-counter medications are preferred to treat pain. 4 Notify the prescribing healthcare provider of the angina experienced by the patient.

4) Notify the prescribing healthcare provider of the angina experienced by the patient. Although it is important to gain as much information as possible from the patient, such as use of over-the-counter medications and details about previous chest pain, because of the likelihood that beta-adrenergic agonists will affect blood pressure and pulse rate, the prescribing healthcare provider should be notified of the incidence of chest pain or palpitations.

What is the most accurate outcome criterion for the nurse to use when determining if a patient is responding appropriately to the use of an adrenergic drug? 1 Patient is able to correctly state three side effects of the medication. 2 Patient is alert to time, person, and place and makes coherent conversation. 3 Patient awakens easily during nurse's morning rounds and states, "I feel fine." 4 Patient's respiratory rate is 16 breaths/minute and blood pressure is 130/72 mm Hg.

4) Patient's respiratory rate is 16 breaths/minute and blood pressure is 130/72 mm Hg. The blood pressure and respiratory readings will give the most accurate information for outcome criteria; the other information is vague and subjective.

A patient is receiving cyanocobalamin for the treatment of pernicious anemia. Which electrolyte should the nurse monitor as a result of this treatment? 1 Sodium 2 Calcium 3 Chloride 4 Potassium

4) Potassium Potassium depletion (hypokalemia) may occur as a natural consequence of erythrocyte production. Because erythrocytes incorporate significant amounts of potassium and a large number of erythrocytes are being produced, potassium levels may fall. The other electrolytes are not affected.

A patient is prescribed digoxin to treat heart failure. Which biochemical parameter should be assessed by the nurse to ensure safe drug administration? 1 Liver enzyme concentration 2 Blood glucose concentration 3 Serum calcium concentration 4 Serum potassium concentration

4) Serum potassium concentration Hypokalemia, usually diuretic induced, is the most frequent underlying cause of dysrhythmias. The nurse should monitor serum potassium concentrations. Because potassium competes with digoxin, when potassium levels are low, binding of digoxin to Na+, K+-ATPase (sodium, potassium-ATPase) increases. This increase can produce excessive inhibition of Na+, K+ -ATPase with resultant toxicity. Digoxin does not have any effect on liver enzymes, blood glucose, or serum calcium. Therefore, assessment of these parameters is not necessary before administering digoxin.

A patient being treated for pernicious anemia with cyanocobalamin, the patient reports new onset of muscle weakness and states, "My heart is skipping beats." Which laboratory value most likely is contributing to these new symptoms? 1 Serum chloride level of 98 mEq/L 2 Serum sodium level of 133 mEq/L 3 Serum glucose level of 185 mg/dL 4 Serum potassium level of 2.3 mEq/L

4) Serum potassium level of 2.3 mEq/L Potassium depletion (hypokalemia) may occur as a natural consequence of erythrocyte production. Because erythrocytes incorporate significant amounts of potassium and a large number of erythrocytes are being produced, potassium levels may fall. A serum potassium level of 2.3 mEq/L indicates hypokalemia. The sodium, chloride, and blood glucose levels are not affected.

A patient's arterial pressure drops upon standing. How should the nurse report this finding to the oncoming shift? 1 The patient has water retention. 2 The patient has reflex tachycardia. 3 The patient has steady-state control. 4 The patient has orthostatic hypotension.

4) The patient has orthostatic hypotension. Postural hypotension, also known as orthostatic hypotension, is a reduction in arterial pressure that can occur when we move from a supine or seated position to an upright position. Water retention is mediated in part by aldosterone through retention of sodium; it is water that is not being excreted but being retained in the body. Reflex tachycardia occurs if a vasodilating drug is administered, activating the baroreceptor reflex. Steady-state sympathetic tone provides a moderate level of vasoconstriction.

The nurse is teaching a patient about the reason for the administration of calcium channel blockers (CCBs). Which information should be included in the teaching plan? 1 This medication will help you to get rid of sodium. 2 This medication will work to cause you to get rid of fluid. 3 This medication will lower your blood pressure by promoting weight loss. 4 This medication will enlarge the blood vessels to lower your blood pressure.

4) This medication will enlarge the blood vessels to lower your blood pressure. This medication causes vasodilation (widening) and is used in hypertension to lower blood pressure. It causes direct vasodilation by blocking calcium influx in smooth muscles in the blood vessels. This medication class does not help to rid fluids, decrease sodium, or lose weight.

What is the primary effect of calcium channel blockers (CCBs)? 1 To treat acute myocardial infarction (MI) 2 To reduce exercise-induced elevations in heart rate 3 To prolong the QT interval on the electrocardiogram (ECG) 4 To decrease the afterload and reduce the workload of the heart

4) To decrease the afterload and reduce the workload of the heart The calcium channel blockers decrease the afterload and reduce the workload of the heart by preventing muscle contraction and promoting muscle relaxation. The beta blockers are more effective in reducing exercise-induced heart rate elevations as they promptly decrease blood pressure and heart rate. CCBs are contraindicated in patients with acute MI.

A nurse is caring for a patient with an average heart rate of 56 beats/min. The patient has no adverse symptoms associated with this heart rate and is receiving no treatment. Which of the following activity modifications should the nurse suggest to avoid further slowing of the heart rate? a) "Avoid bearing down while having a bowel movement." b) "Avoid stress or overexcitement." c) "Avoid strenuous aerobic exercise." d) "Limit your intake of caffeinated drinks."

a) "Avoid bearing down while having a bowel movement." Bearing down during a bowel movement stimulates the vagus nerve and results in a slowing of the heart rate. Vagal stimulation as well as some medications decreases the firing rate of the sinoatrial node and conduction through the atrioventricular node to cause a decrease in heart rate.

A patient in the nursing home wonders why he is having these signs and symptoms of left-sided failure. Which of the following explanations will the nurse give the patient? a) "The left ventricle is having problems pumping blood forward, and this is causing blood to back up into your lungs." b) "The left ventricle is pumping excessive amounts of blood through the carotid arteries causing headache." c) "The left ventricle is not adequately perfusing your gastrointestinal tract, leading to diarrhea and vomiting." d) "The left ventricle is having problems pumping blood forward so blood is backing up systemically causing edema in your feet."

a) "The left ventricle is having problems pumping blood forward, and this is causing blood to back up into your lungs." In left-sided failure, the left ventricle pumps inefficiently, blood backs up in the pulmonary circulation, and less, rather than more, blood enters the carotid arteries. Right-sided rather that left-sided failure causes blood to back up in the venous circulation resulting in edema. Congestion of the gastrointestinal tract is usually characteristic of advanced heart failure.

What should the nurse teach a client who is diagnosed with chronic hepatitis B infection about treatment? a) "The treatment is effective if your liver enzymes return to normal." b) "Steroids are often used to control hepatitis B infection." c) "You will need evaluation for a liver transplant in a few months." d) "Antiretroviral medications work for HIV, but not for hepatitis B."

a) "The treatment is effective if your liver enzymes return to normal." Chronic hepatitis B infection is treated with diet to avoid stressing the liver (no alcohol; moderate fat intake) and medications. Drugs include interferons and the nucleotide and nucleotide analog antiretroviral agents (lamivudine, entacavir, and tenofovir). Liver transplantation is considered for end-stage liver failure, but is not as successful for chronic hepatitis B as for chronic hepatitis C. Steroids are used to treat autoimmune hepatitis

A 29-year-old woman who considers herself active and health conscious is surprised to have been diagnosed with preeclampsia in her second trimester. What should her care provider teach her about this change in her health status? a) "We don't really understand why some women get high blood pressure when they're pregnant." b) "Even though you're a healthy person, it could be that you have an underlying heart condition." c) "This is likely a result of your nervous system getting overstimulated by pregnancy." d) "Hypertension is a common result of all the hormonal changes that happen during pregna

a) "We don't really understand why some women get high blood pressure when they're pregnant." The cause of pregnancy-induced hypertension is largely unknown. During pregnancy, levels of estrogen, progesterone, renin, angiotensin II and aldosterone are elevated, and women who develop preeclampsia may be especially sensitive to their vasoconstrictive and sodium and water retaining properties.

Normal red blood cells live an average of about how many days? a) 120 b) 3 c) 90 d) 12

a) 120 Normal red blood cells live 120 days.

A client has been placed on bed rest. Select the percentage of the blood volume that will be distributed to the client's pulmonary circulation. a) 25% b) 10% c) 20% d) 15%

a) 25% Body position affects the distribution of blood volume. In the recumbent position, approximately 25% to 30% of the total blood volume is in the pulmonary circulation. On standing, gravity causes a rapid displacement of this blood to the lower part of the body. Because the volume of the systemic circulation is approximately seven times that of the pulmonary circulation, a shift of blood from one system to the other has a much greater effect in the pulmonary than in the systemic.

The physician's order states, "Calculate the pulse pressure of the client's B/P." The blood pressure reading is as follows: systolic pressure of 146 mm Hg and a diastolic pressure of 82 mm Hg. The pulse pressure would be: a) 64 mm Hg b) 228 mm Hg c) 41mm Hg d) 73 mm Hg

a) 64 mm Hg The difference between the systolic and diastolic pressure is called the pulse pressure (approximately 40 mm Hg). The pulse pressure is calculated by deducting the diastolic pressure (82) from the systolic pressure (146). The difference is the pulse pressure.

From the following clients, who are at high risk for developing heart failure as a result of diastolic dysfunction? Select all that apply. a) A 48-year-old client with uncontrolled hypertension b) A marathon runner with history of chronic bradycardia whose pulse rate is 46 c) A 70-year-old with enlarged left ventricle due to myocardial hypertrophy d) A 57-year-old client with history of ischemic heart disease

a) A 48-year-old client with uncontrolled hypertension c) A 70-year-old with enlarged left ventricle due to myocardial hypertrophy Conditions that reduce the heart's ability to adequately fill during diastole, such as myocardial hypertrophy and tachycardia, can lead to heart failure. Hypertension remains the leading cause of diastolic dysfunction. Ischemic heart disease is associated with systolic heart failure, or impaired contractile performance. It is normal for athletes, like marathon runners, to have slow pulses.

Which of the following patients is at greatest risk for orthostatic hypotension? a) A 66-year-old post-surgery patient on bed rest b) A 20-year-old pregnant patient at 36 weeks gestation c) A 42-year-old male patient with history of pulmonary embolism d) A 70-year-old female patient who has taken the same antihypertensive medication for 10 years

a) A 66-year-old post-surgery patient on bed rest Post-surgery patients who have been immobile are at greatest risk for developing othostatic hypotension. The 70-year-old female may also be at some risk: age is a risk factor, as is administration of some antihypertensive medications.

ST-elevated myocardial infarction is accompanied by severe, crushing pain. Morphine is the drug of choice used to treat the pain of STEMI when the pain cannot be relieved with oxygen and nitrates. Why is morphine considered the drug of choice in STEMI? a) Action decreases metabolic demands of the heart. b) Action increases autonomic nervous system activity. c) Action increases anxiety increasing metabolic demands of the heart. d) Action relieves pain and gives sense of depression.

a) Action decreases metabolic demands of the heart. Although a number of analgesic agents have been used to treat the pain of STEMI, morphine is usually the drug of choice. It usually is indicated if chest pain is unrelieved with oxygen and nitrates. The reduction in anxiety that accompanies the administration of morphine contributes to a decrease in restlessness and autonomic nervous system activity, with a subsequent decrease in the metabolic demands of the heart. Morphine does not cause a feeling of depression to the client.

A patient comes to the clinic with a runny nose and scratchy throat. The nurse is evaluating the white blood cell count, which shows an elevated number of white blood cells and an increased percentage of eosinophils. Which of the following is the most likely cause of the symptoms? a) Allergic reaction b) Bacterial infection c) Viral infection d) Fungal infection

a) Allergic reaction Eosinophils increase in number during allergic reactions and parasitic infections.

Anticoagulant drugs prevent thromboembolic disorders. How does warfarin, one of the anticoagulant drugs, act on the body? a) Alters vitamin K, reducing its ability to participate in the coagulation of the blood b) Increases vitamin K-dependent factors in the liver c) Increases prothrombin d) Increases procoagulation factors

a) Alters vitamin K, reducing its ability to participate in the coagulation of the blood The anticoagulant drugs warfarin and heparin are used to prevent thromboembolic disorders, such as deep vein thrombosis and pulmonary embolism. Warfarin acts by decreasing prothrombin and other procoagulation factors. It alters vitamin K in a manner that reduces its ability to participate in synthesis of the vitamin K-dependent coagulation factors in the liver.

A client is rushed to the emergency department with symptoms of urticaria, wheezing, chest tightness, and difficulty in breathing. The client is most likely experiencing which type of shock? a) Anaphylactic b) Cardiogenic c) Neurogenic d) Septic

a) Anaphylactic Signs and symptoms associated with impending anaphylactic shock include abdominal cramps; apprehension; warm or burning sensation of the skin, itching, and urticaria (i.e., hives) and coughing, choking, wheezing, chest tightness, and difficulty in breathing. The other options do not have these manifestations.

The nurse is preparing to auscultate for a mitral valve stenosis murmur Where is the best location to place the stethoscope? a) At the apex of the heart b) Over the sternum c) At the carotid arteries d) Over the aorta

a) At the apex of the heart The murmur of mitral valve stenosis is heard during diastole when blood is flowing through the constricted valve; it is characteristically a low pitched, rumbling murmur best heard at the apex of the heart. The other locations would not be effective.

A client's echocardiogram identified a narrowed valve that has resulted in a decreased blood flow between the left atria and left ventricle. The nurse would interpret this as the: a) Bicuspid valve b) Tricuspid valve c) Aortic valve d) Pulmonic valve

a) Bicuspid valve The bicuspid valve, (also called the mitral) controls the flow of blood between the left atria and left ventricle. The aortic valve controls flow between the left ventricle and aorta. The tricuspid controls the flow between the right atria and ventricle. The pulmonic valve controls flow between the right ventricle and pulmonary artery.

A client is admitted to the intensive care unit suspected of having infective endocarditis. Which of the following tests is the most definitive diagnostic procedure that is done and used to guide treatment for this type of client? a) Blood culture b) CBC c) Echocardiogram d) ECG

a) Blood culture The blood culture remains the most definitive diagnostic procedure and is essential to guide treatment. An echo, ECG, clinical findings and lab information are also used to help in the diagnosis.

A client was involved in an auto accident and suffered massive internal injuries that resulted in a large blood loss. Select the type of anemia the client is at greatest risk to develop. a) Blood loss b) Iron deficiency c) Aplastic d) Hemolytic

a) Blood loss The clinical manifestations and red cell changes associated with blood loss anemia depend on the rate of hemorrhage and whether the bleeding loss is internal or external. The effects of acute blood loss are mainly due to loss of intravascular volume, which can lead to cardiovascular collapse and shock. Hemolytic anemia is characterized by the premature destruction of red cells. The usual reason for iron deficiency in adults in the Western world is chronic blood loss. Aplastic anemia describes a disorder of pluripotential bone marrow stem cells.

Select the option that best describes the production of T lymphocytes. a) Bone marrow - thymus - lymph nodes b) Bone marrow - plasma cells - lymph nodes c) Plasma cells - lymph nodes - arteries d) Myocardial tissue - bone marrow - lymph nodes

a) Bone marrow - thymus - lymph nodes T lymphocytes leave the bone marrow as precursor T lymphocytes and travel to the thymus, where they differentiate into CD4 helper T cells and CD8 cytotoxic T cells. T lymphocytes also travel to the lymph nodes. The first option describes the production of B lymphocytes. The other options do not describe B- or T-lymphocyte production

A client presents with epigastric pain, a mild fever, nausea, and vomiting. His history shows a previous episode with similar symptoms that reverted in 24 hours. This time the pain is not subsiding. What diagnosis is the most likely? a) Calculous cholecystitis b) Hepatitis C virus (HCV) c) Pancreatic cancer d) Cirrhosis

a) Calculous cholecystitis Persons with calculous cholecystitis usually, but not always, have experienced previous episodes of biliary pain. Hepatitis, pancreatic cancer, and cirrhosis have very different symptoms.

Select the correct sequence of blood return to the heart. a) Capillaries, venules, veins, right atrium b) Capillaries, arterioles, veins, left atrium c) Capillaries, arterioles, arteries, right atrium d) Capillaries, venules, veins, left atrium

a) Capillaries, venules, veins, right atrium The correct pathway for blood returning back to the heart is the capillaries, venules, veins, and right atrium. The other options do not support normal blood flow.

An elderly patient is brought to the emergency department with garbled speech, unilateral facial drooping, and weakness. One hour after admission, the patient dies. An autopsy reveals the presence of polycythemia. Which of the following was the most likely cause of the patient's death? a) Cerebral thrombosis b) Infection and sepsis c) Acute leukemia d) Anaphylactic reaction

a) Cerebral thrombosis Unregulated overproduction of the red cell mass is termed polycythemia, which causes a thickening of the blood and an increased risk of blood clots.

All diseases have risk factors. What is the most significant environmental risk factor for pancreatic cancer? a) Cigarette smoking b) Water pollution c) Air pollution d) Heavy metal toxicity

a) Cigarette smoking In pancreatic cancer, the most significant and reproducible environmental risk factor is cigarette smoking. The other answers are incorrect.

A nurse is planning a community education program on lifestyle modification to manage hypertension. Which of the following topics should be included in the teaching plan? Select all that apply. a) Consume a diet rich in fruits, vegetables, and low-fat dairy products. b) Reduce dietary sodium intake. c) Avoid aerobic physical activity. d) Stop smoking. e) Limit alcohol consumption.

a) Consume a diet rich in fruits, vegetables, and low-fat dairy products. b) Reduce dietary sodium intake. d) Stop smoking. e) Limit alcohol consumption. Hypertension lifestyle modification included the maintenance of normal body weight (BMI, 18.5-24.9 kg/m2) 5-20 mm Hg/10 kg weight loss. Adoption of the Dietary Approaches to Stop Hypertension (DASH) eating plan including consuming a diet rich in fruits, vegetables, and low-fat dairy products with a reduced content of saturated and total fat 8-14 mm Hg. Dietary sodium reduction to no more than 100 mmol/day (2.4 g sodium or 6 g sodium chloride) 2-8 mm Hg. Engage in regular aerobic physical activity such as brisk walking (at least 30 minutes per day. Moderation of alcohol consumption; limiting consumption to no more than 2 drinks (1 oz or 30 mL ethanol) (e.g., 24 oz beer, 10 oz wine, or 3 oz 80-proof whiskey) per day in most men and 1 drink per day in women and lighter-weight persons.

A patient has developed secondary hypertension. The nurse knows that causes of secondary hypertension include which of the following? Select all that apply. a) Cushing's syndrome b) Pheochromocytoma c) Aortic coarctation d) Oral contraceptives e) Advanced age

a) Cushing's syndrome b) Pheochromocytoma c) Aortic coarctation d) Oral contraceptives In secondary hypertension, an underlying cause can be identified. Excess glucocorticoid secretion as in Cushing's syndrome enhances sodium and water retention which elevates blood pressure. Pheochromocytoma is a tumor which secretes epinephrine and norepinephrine which produce vasoconstriction and increased rate and force of myocardial contraction, increasing blood pressure. Coarctation is a narrowing of the aorta which decreases blood flow to the lower body and increases flow and systolic pressure in the upper body. Oral contraceptive drugs cause sodium and water retention, increasing blood volume and pressure. Advanced age is a risk factor for essential or primary hypertension.

Although the usual site for a bone marrow test is the posterior iliac crest, other sites include the anterior iliac crest and the sternum. What are the dangers of using the sternum for a bone marrow test in children? a) Danger of perforating the mediastinum and heart b) Potential for hemorrhage c) Potential for infection in the chest cavity d) Danger of perforating the lungs

a) Danger of perforating the mediastinum and heart Usually, the posterior iliac crest is used in all persons older than 12 to 18 months of age. Other sites include the anterior iliac crest, sternum, and spinous processes T10 through L4. The sternum is not commonly used in children because the cavity is too shallow, and there is danger of mediastinal and cardiac perforation.

A patient brought to the emergency department has preeclampsia. The nurse knows that this disorder can progress to include which of the following complications? Select all that apply. a) Disseminated intravascular coagulation (DIC) b) Renal failure c) Cerebral hemorrhage d) Hepatic failure e) Elevated platelet count

a) Disseminated intravascular coagulation (DIC) b) Renal failure c) Cerebral hemorrhage d) Hepatic failure Severe preeclampsia has risk for cerebral hemorrhage, hepatic and renal failure, and DIC. Platelet counts fall, with less than 100,000/mm2 diagnostic of serious disease.

An elderly patient is diagnosed with orthostatic hypotension. For which of the following clinical manifestations should the nurse assess? a) Dizziness and fainting b) Headache and double vision c) Chest pain and palpitations d) Nausea and vomiting

a) Dizziness and fainting Orthostatic or postural hypotension is an abnormal drop in blood pressure on assumption of the standing position The presence of orthostatic symptoms (e.g., dizziness, syncope) is more relevant to the diagnosis that the actual blood pressure readings.

A nurse in a nursing home is concerned that a resident may be developing left heart failure. Manifestations of left heart failure would include which of the following symptoms? a) Dyspnea, cough b) Weakness, peripheral edema c) Diarrhea, nausea, vomiting d) Fatigue, headache

a) Dyspnea, cough In left failure, dyspnea and cough, especially at night, result from blood backing up in the pulmonary circulation. The other manifestations listed are not characteristic of left-sided failure.

A client who previously has suffered a myocardial infarction comes to clinic for a routine checkup. He tells the nurse that he has started to experience more episodes of angina pectoris than he had before. The nurse should tell this client that which of the following can precipitate his angina pectoris? Select all that apply. a) Exposure to cold b) Physical exertion c) Emotional stress d) Eating too fast e) Reading suspense books

a) Exposure to cold b) Physical exertion c) Emotional stress Angina pectoris usually is precipitated by situations that increase the work demands of the heart, such as physical exertion, exposure to cold and emotional stress.

The nurse is assessing a female patient with a hemoglobin of 6.8g/dL. Which of the following symptoms would the nurse expect to find? Select all that apply. a) Faintness b) Pink skin and mucous membranes c) Bradycardia d) Headache e) Dyspnea

a) Faintness d) Headache e) Dyspnea In anemia, the oxygen-carrying capacity of hemoglobin is reduced, causing tissue hypoxia. Tissue hypoxia can give rise to fatigue, weakness, dyspnea and sometimes angina. Hypoxia of brain tissue results in headache, faintness and dim vision. Redistribution of blood from cutaneous tissues or a lack of hemoglobin causes pallor of the skin and mucous membranes. Tachycardia and palpitation may occur as the body tries to compensate with an increase in cardiac output.

A nurse is evaluating hypertension risk factors with an African American male who is a lawyer in a busy legal firm. He reports that he eats fairly well, usually having red meat and potatoes daily. His father and older brother have hypertension. His paternal grandfather had a stroke. The lawyer drinks about four beers and eats salted popcorn while watching television in the evening and has gained 15 pounds in the past year. Which of the following are nonmodifiable risk factors associated with this diagnosis? Select all that apply. a) Family history b) Obesity c) Race d) Excessive alcohol consumption e) Stress

a) Family history c) Race Nonmodifiable risk factors include a family history of hypertension, age-related increases in blood pressure, and race. Modifiable risk factors are lifestyle factors can contribute to the development of hypertension by interacting with the constitutional risk factors. These lifestyle factors include high salt intake, excessive calorie intake and obesity, excessive alcohol consumption, and low intake of potassium. Although stress can raise blood pressure acutely, there is less evidence linking it to chronic elevations in blood pressure. Smoking and a diet high in saturated fats and cholesterol, although not identified as primary risk factors for hypertension, are independent risk factors for coronary heart disease and should be avoided.

A nursing professor is teaching nursing students about the immune system. The instructor knows that teaching has been successful when a student correctly identifies which of the following as being roles of the monocyte? Select all that apply. a) Functions as a macrophage in the tissues b) Development of immune granulomas c) Function as an antigen-presenting cell d) Releases immunoglobulins e) Engulf foreign material

a) Functions as a macrophage in the tissues b) Development of immune granulomas c) Function as an antigen-presenting cell e) Engulf foreign material Monocytes, which are precursors of the mononuclear phagocyte system, are often referred to as macrophages when they enter the tissues. The monocytes engulf larger and greater quantities of foreign material than the neutrophils. These leukocytes play an important role in chronic inflammation and are also involved in the immune response by activating lymphocytes and by presenting antigen to T cells. When the monocyte leaves the vascular system and enters the tissues, it functions as a macrophage with specific activity. Granulomatous inflammation is a distinctive pattern of chronic inflammation in which the macrophages form a capsule around insoluble materials that cannot be digested

Nearly everyone with pericarditis has chest pain. With acute pericarditis, the pain is abrupt in onset, sharp, and radiates to the neck, back, abdomen, or sides. What can be done to ease the pain of acute pericarditis? a) Have the client sit up and lean forward. b) Have the client breathe deeply. c) Have the client change positions to unaffected side. d) Have the client swallow slowly and frequently.

a) Have the client sit up and lean forward. With acute pericarditis, the pain typically is pleuritic (aggravated by inspiration and swallowing) and positional (decreases with sitting and leaning forward; increases with moving to the side) because of changes in venous return and cardiac filling.

A client with malignant hypertension is at risk for a hypertensive crisis, including the cerebral vascular system often causing cerebral edema. The nurse would assess this client for which signs and symptoms? a) Headache and confusion b) Restlessness and nervousness c) Papilledema and lethargy d) Stupor and hyperreflexia

a) Headache and confusion Cerebral vasoconstriction probably is an exaggerated homeostatic response designed to protect the brain from excesses of blood pressure and flow. The regulatory mechanisms often are insufficient to protect the capillaries, and cerebral edema frequently develops. As it advances, papilledema (i.e., swelling of the optic nerve at its point of entrance into the eye) ensues, giving evidence of the effects of pressure on the optic nerve and retinal vessels. The client may have headache, restlessness, confusion, stupor, motor and sensory deficits, and visual disturbances. In severe cases, convulsions and coma follow. Lethargy, nervousness, and hyperreflexia are not signs or symptoms of cerebral edema in malignant hypertension.

A client tells the nurse that he is concerned about developing hepatitis after being exposed to contaminated feces, saliva, and food. The nurse is aware that the client is at risk for: a) Hepatitis A b) Hepatitis D c) Hepatitis B d) Hepatitis C

a) Hepatitis A Hepatitis A is normally transmitted through the fecal-oral route by drinking contaminated milk or water and eating shellfish from infected waters. Hepatitis B is transmitted through infected blood or serum, hepatitis C is transmitted by recreational injection drug use, and hepatitis D occurs largely to persons at high risk for HBV infection.

The nurse is caring for a client with right upper quadrant pain secondary to acute choledocholithiasis. The nurse anticipates if the common bile duct becomes obstructed, manifestations will also include which of these? a) Hyperbilirubinemia b) Vomiting c) Ascites d) Hemorrhage

a) Hyperbilirubinemia Choledocholithiasis, stones in the common duct, usually originate in the gallbladder, but can form spontaneously in the common duct. Bilirubinuria and an elevated serum bilirubin (hyperbilirubinemia) are present if the common duct is obstructed. With acute cholecystitis, approximately 75% of patients have vomiting. Ascites is common with late-stage liver failure rather than duct obstructions. Bleeding is associated with liver failure due to deficiency of clotting factors and acute pancreatitis due to activated enzymes causing fat necrosis and hemorrhage from the necrotic vessels.

The nurse is assisting a patient who had a myocardial infarction 2 days ago during a bath. The patient suddenly lost consciousness and the nurse was unable to feel a pulse. Cardiopulmonary resuscitation was begun and the patient was connected to the monitor with a gross disorganization without identifiable waveforms or intervals observed. Which of the following is a priority intervention at this time? a) Immediate defibrillation b) Synchronized cardioversion c) Administration of atropine d) Applying a transcutaneous pacemaker

a) Immediate defibrillation The classic electrocardiographic pattern of ventricular fibrillation is that of gross distortion without identifiable waveforms or or intervals. When the ventricles do not contract, there is no cardiac output, and there are no palpable or audible pulses. Immediate defibrillation using a nonsynchronized, direct-current electrical shock is mandatory for ventricular fibrillation and for ventricular flutter that has caused loss of consciousness.

A client has just been told that he has an infection of the inner surface of the heart. He is also told that the bacteria has invaded his heart valves. What term is used for this disease process? a) Infective endocarditis b) Myocardial infarction c) Pericarditis d) Cardiomyopathy

a) Infective endocarditis Infective endocarditis is a serious and potentially life threatening infection of the inner surface of the heart. Pericarditis involves an inflammatory response of the pericardium. Myocardial infarction is a heart attack while cardiomyopathy is a heart disorder that is confined to the myocardium and can sometimes represent myocardial changes that occur with a variety of systemic disorders.

Following a lecture on hemostasis, a nursing student accidently cuts her hand while preparing supper for her family. She watches the laceration very closely. Sure enough, the first thing she notes is: a) Initially, it takes a few seconds for blood to appear as a result of vessel spasm. b) Small hair-like strands form a blood (fibrin) clot. c) Limited bleeding initially as a response to the small vessel walls being sealed by a platelet plug. d) Clot retraction by pushing serum out of the clot and joining the edges of the broken vessel.

a) Initially, it takes a few seconds for blood to appear as a result of vessel spasm. There are five stages in hemostasis, with the first step being transient vessel vasospasm. Vessel spasm is initiated by endothelial injury and caused by local and humoral mechanisms. It is a transient event, usually lasting less than 1 minute. For smaller vessels, release of the vasoconstrictor TXA2 is responsible for much of the vessel spasm. Formation of the platelet plug, development of an insoluble fibrin clot, and clot retraction happen after vessel spasm.

A client undergoing emergent open heart surgery receives an autologous transfusion that has been collected from the operative site. This type of collection is known as which of the following? a) Intraoperative salvage b) Phlebotomy c) Hemodilution d) Predeposit

a) Intraoperative salvage Intraoperative blood salvage is the collection of blood shed from the operative site for reinfusion into the person. A person who is anticipating elective surgery may predeposit blood. Hemodilution involves phlebotomy before surgery with transfusion of the person's blood at the completion of surgery.

Respiratory sinus arrhythmia is considered a more optimal rhythm than a rhythm where all RR intervals are equal. In respiratory sinus arrhythmia, what is the variation in cardiac cycles related to? a) Intrathoracic pressure changes that occur with respiration b) Intraabdominal pressure changes that occur with respiration c) Respiratory sinus arrhythmia is considered a more optimal rhythm than a rhythm where all RR intervals are equal. In respiratory sinus arrhythmia, what is the variation in cardiac cycles related to? d) Intraabdominal pressure changes due to vagal nerve stimulus e) Intrathoracic pressure changes due to inadequate oxygenation

a) Intrathoracic pressure changes that occur with respiration Today, it is accepted that a more optimal rhythm is respiratory sinus arrhythmia. Respiratory sinus arrhythmia is a cardiac rhythm characterized by gradual lengthening and shortening of RR intervals. This variation in cardiac cycles is related to intrathoracic pressure changes that occur with respiration and resultant alterations in autonomic control of the sinoatrial node. The other answers do not cause the variation in cardiac cycles related to respiratory sinus arrhythmia.

Manifestations of heart failure are due to the decreased pumping ability of the heart. The nurse will monitor for which of the following signs and symptoms of heart failure? Select all that apply. a) Malnutrition b) Exercise intolerance c) Hyperglycemia d) Cyanosis e) Fatigue

a) Malnutrition b) Exercise intolerance d) Cyanosis e) Fatigue Manifestations of heart failure can include fatigue and exercise intolerance, as reduced cardiac output does not provide adequate oxygen and nutrients to support physical activity. Cyanosis develops in heart failure due to poor arterial blood oxygenation. Gastrointestinal changes discourage food consumption, leading to poor nutrition. Hyperglycemia is not a sign of heart failure.

A client arrives at the doctor's office complaining of severe indigestion that has been intermittent; however, the pain is now constant and feels like a vise. The nurse does an ECG and recognizes that the situation is possibly emergent due to ST-segment elevation, which could indicate which of the following? a) Myocardial infarction b) Advanced hypokalemia c) Decreased intracranial pressure d) Benign late repolarization

a) Myocardial infarction The ST-segment elevation with myocardial infarction could be caused by a decrease in blood supply to that area of cardiac muscle. However, the mechanism of ST-segment elevation is not clearly understood. The other options with this question are the opposite of other causes of ST-segment elevation.

A major factor in the development of hepatic encephalopathy is: a) Neurotoxin accumulation b) Hypersplenism c) High sodium level d) Steroid hormone deficiency

a) Neurotoxin accumulation Although the cause of hepatic encephalopathy is unknown, the accumulation of neurotoxins, which appear in the blood because the liver has lost its detoxifying capacity, is believed to be a factor. The liver metabolizes the steroid hormones; therefore, these hormones are often elevated in persons with liver failure and cause feminization (rather than encephalopathy) of male clients. Hypersplenism associated with liver failure is a factor in the development of anemia, thrombocytopenia, and leukopenia. Although the mechanisms responsible for the development of ascites are not completely understood, several factors seem to contribute to fluid accumulation, including salt and water retention by the kidney and increase in capillary pressure due to portal hypertension and obstruction of venous flow through the liver.

A client who developed a deep vein thrombosis during a prolonged period of bed rest has deteriorated as the clot has dislodged and resulted in a pulmonary embolism. Which of the following types of shock is this client at risk of experiencing? a) Obstructive shock b) Cardiogenic shock c) Hypovolemic shock d) Distributive shock

a) Obstructive shock Obstructive shock results from mechanical obstruction of the flow of blood through the central circulation, such as the blockage that characterizes a pulmonary embolism.

A client is seen in the emergency room with complaints of sharp chest pain that started abruptly. He says it has radiated to his neck and abdomen. He also states that it is worse when he takes a deep breath or swallows. He tells the nurse that when he sits up and leans forward the pain is better. Upon examination the nurse notes a pericardial friction rub and some EKG changes. Which disease should the nurse suspect this client to have? a) Pericarditis b) Pneumonia c) Abdominal aortic aneurysm d) Myocardial infarction

a) Pericarditis This patient is demonstrating signs and symptoms of pericarditis, which includes a triad of chest pain, pericardial friction rub, and EKG changes. Other signs are that the pain is usually abrupt in onset, occurs in the pericardial area, and may radiate to the neck, back, abdomen or side. It is usually worse with deep breathing and swallowing, and the person often finds relief when sitting up and leaning forward.

The nurse is caring for a client with leukemia that is having difficulty with blood clotting after having several transfusions with packed red blood cells. What does the nurse anticipate infusing for this client to assist with controlling the bleeding? a) Platelets b) Albumin c) Ringers Lactate solution d) Whole blood

a) Platelets Thrombocytes, or platelets, are circulating cell fragments of the large megakaryocytes that are derived from the myeloid stem cell. They function to form the platelet plug to help control bleeding after injury to a vessel wall. Their cytoplasmic granules release mediators required for the blood coagulation process. Thrombocytes have a membrane but no nucleus, cannot replicate, and, if not used, last approximately 10 days in the circulation before the phagocytic cells of the spleen remove them.

Which of the following occurs during repolarization? a) Positively charged K+ moves outward across the cell membrane b) Positively charged K+ moves into the cell membrane c) Positively charged Na+ moves into the cell membrane d) Positively charged Na+ moves outward across the cell membrane

a) Positively charged K+ moves outward across the cell membrane During rapid repolarization period, the slow Ca++ channels close and the influx of Ca++ and Na+ ceases. There is a sharp rise in K+ permeability, contributing to the rapid outward movement of K+ and reestablishing the resting membrane potential.

Which of the following factors affect cardiac performance? Select all that apply. a) Preload b) Heart rhythm c) Afterload d) Myocardial contractility

a) Preload c) Afterload d) Myocardial contractility The four factors that affect cardiac performance include preload, afterload, myocardial contractility, and heart rate; not heart rhythm.

Which of the following statements regarding circadian variations in blood pressure are correct? Select all that apply. a) Pressure falls during the day. b) "Dippers" are persons with normal circadian pattern of blood pressure changes. c) Pressure is lowest upon awakening. d) Pressure remains steady during the day. e) Pressure is highest upon awakening.

a) Pressure falls during the day. b) "Dippers" are persons with normal circadian pattern of blood pressure changes. e) Pressure is highest upon awakening. Blood pressure is highest upon awakening and gradually falls during the waking period to the lowest point after several hours of sleep. "Dippers" are persons who exhibit this normal circadian pattern of blood pressure.

An elderly client presents to the clinic just not "feeling well." Which of the following bone marrow results would confirm the diagnosis of chronic lymphocytic leukemia? a) Proliferation of well-differentiated blood cells in the marrow b) Uncontrolled growth of immature blood cells in the marrow c) Replication of pluripotent precursor blood cells in the marrow d) Production of undifferentiated blood cells in the marrow

a) Proliferation of well-differentiated blood cells in the marrow Chronic leukemia involves abnormal proliferation and growth of well-differentiated blood cell types. Acute leukemia is associated with proliferation of undifferentiated, immature, and precursor blood cells.

The nurse is interpreting an electrocardiogram of a 65-year-old woman. Which should the nurse recognize as representing ventricular depolarization? a) QRS complex b) T wave c) ST segment d) P wave

a) QRS complex The QRS complex is representative of ventricular depolarization. The P wave is atrial depolarization, the T wave is ventricular repolarization, and the ST segment is the time to ventricular repolarization.

A client is suspected of having the onset of alcoholic liver disease. The nurse should be assessing for which of the following manifestations related to the necrosis of liver cells? a) Rapid onset of jaundice b) Development of multiple skin nodules c) Tremors of the hands d) Long muscle group atrophy

a) Rapid onset of jaundice Alcoholic hepatitis is the intermediate stage between fatty changes and cirrhosis and is characterized by inflammation and necrosis of liver cells. The condition is always serious and sometimes fatal. The cardinal sign of alcoholic hepatitis is rapid onset of jaundice. Hand tremors are not specific to alcoholic hepatitis. Long muscle group atrophy can occur but is not the primary sign. Development of nodules is not caused by alcoholic hepatitis.

Which of the following would be tested by a complete blood count (CBC)? a) Red blood cells and platelets b) Size and shape of blood cells c) Lymphocytes and neutrophils d) Sodium and potassium

a) Red blood cells and platelets A CBC is a commonly performed screening test that determines the number of red blood cells, white blood cells, and platelets per unit of blood. Lymphocytes and neutrophils may be included in the CBC if a differential of white blood cells was also ordered. Electrolytes and size/shape of blood cells are not part of a CBC.

A patient is prescribed an angiotensin converting enzyme inhibitor (ACEI) for hypertension. The nurse knows that ACEIs are contraindicated by which clinical condition? a) Renal artery stenosis b) Heart failure c) Coronary artery disease d) Diabetes

a) Renal artery stenosis ACEI are contraindicated in renal artery stenosis, a condition that relies on the renin-angiotensis system to maintain adequate renal perfusion. ACEI are used clinically in the management of heart failure. Coronary artery disease and diabetes do not contraindicate ACEI.

The nurse is assessing a client who has just been admitted to the unit with a diagnosis of cholelithiasis. The nurse is aware that the client may manifest: a) Right upper quadrant pain b) Left lower quadrant pain c) Left upper quadrant pain d) Right lower quadrant pain

a) Right upper quadrant pain The pain is usually located in the upper right quadrant or epigastric area and may be referred to the upper back, right shoulder, or midscapular region. Typically, the pain is abrupt in onset, increases steadily in intensity, persists for 2 to 8 hours, and is followed by soreness in the upper right quadrant.

A client who is experiencing angina at rest that has been increasing in intensity should be instructed to do which of the following? a) See the doctor for evaluation immediately. b) Give it 5-10 minutes more to see if there is relief. c) Not worry about it as this is common for someone who has already had a myocardial infarction. d) Take a second nitroglycerine.

a) See the doctor for evaluation immediately. Angina that occurs at rest, is of new onset, or is increasing in intensity or duration denotes an increased risk for myocardial infarction and should be seen immediately using the criteria for acute coronary syndrome (ACS).

The nurse is reviewing hospital discharge instructions with a patient who has been diagnosed with secondary thrombocytosis and who also suffers from chronic ulcerative colitis, making anticoagulation inadvisable. The nurse stresses to the patient that the patient should avoid which of the following? Select all that apply. a) Smoking b) Low-fat diet c) Immobilization d) Oral contraceptives e) Folate supplements

a) Smoking c) Immobilization d) Oral contraceptives The goal with these patients would be to decrease thrombotic effects from other causes such as immobolization, smoking, oral contraceptives,steroid therapy, vitamin B, and folate deficiencies and to decrease the need for anticoagulation therapy.

Which of the following is the difference between the end-diastolic and end-systolic volumes? a) Stroke volume b) Ejection fraction c) Cardiac output d) Cardiac reserve

a) Stroke volume Stroke volume is determined by the difference between end-diastolic and end-systolic volumes. Cardiac output is determined by stroke volume and heart rate. Ejection fraction is the volume ejected from the left ventricle. Cardiac reserve is determined by cardiac output.

The health care provider is concerned that a client has developed a decrease in cardiac output. Cardiac output may be calculated as: a) Stroke volume × heart rate b) Blood pressure × heart rate c) Stroke volume × respiratory rate d) Blood pressure × stroke volume

a) Stroke volume × heart rate The cardiac output (CO) is a product of the stroke volume and the heart rate (HR) and can be expressed as CO = SV × HR.

A patient is diagnosed with stage 2 hypertension. The nurse knows that which of the following is characteristic of stage 2 hypertension? a) Sustained systolic pressure equal to or greater that 160 mm Hg b) Sustained diastolic pressure of 90-99 mm Hg c) Sustained systolic pressure of 140-159 mm Hg d) Sustained diastolic pressure of 81-89 mm Hg

a) Sustained systolic pressure equal to or greater that 160 mm Hg Stage 2 hypertension is systolic pressure greater than 159 mm Hg or diastolic pressure greater than 99 mm Hg. Systolic pressure of 140-159 mm Hg or diastolic pressure of 90-99 mm Hg are characteristic of stage 1 hypertension.

The nurse is reviewing the circulatory system. Which statements are correct about the functional organization of the circulatory system? Select all that apply. a) The arterial system distributes oxygenated blood to the tissues. b) The heart exchanges gases, nutrients and wastes. c) The capillaries pump blood. d) The venous system collects deoxygenated blood from the tissues .

a) The arterial system distributes oxygenated blood to the tissues. d) The venous system collects deoxygenated blood from the tissues . The circulatory system consists of the heart which pumps blood, the arterial system which distributes oxygenated blood to the tissues, the venous system which collects deoxygenated blood from the tissues and returns it to the heart, and the capillaries where exchange of gases, nutrients and waste takes place.

A client has been diagnosed with aortic stenosis and asks the nurse what this means. The most appropriate response would be: a) The valve opening is narrowed and produces increased resistance to blood flow out of the left ventricle and into the aorta. b) The valve opening is incompetent, thereby allowing blood to flow back from the pulmonary artery and into the left atrium. c) The valve opening permits backward flow to occur when the valve should be closed. d) The valve opens backward permitting blood to flow from the right ventricle into the right atrium.

a) The valve opening is narrowed and produces increased resistance to blood flow out of the left ventricle and into the aorta. Stenosis refers to a narrowing of the valve orifice and failure of the valve leaflets to open normally. This increases the work and volume of the chamber emptying through the narrowed valve-the left atrium in the case of mitral stenosis and the left ventricle in aortic stenosis. An incompetent or regurgitant valve permits backward flow to occur when the valve should be closed-flowing back into the left ventricle during diastole when the aortic valve is affected and back into the left atrium during systole when the mitral valve is diseased.

The community health nurse is teaching about prevention of hepatitis A. Which of these groups does the nurse suggest will benefit from this vaccine? a) Those traveling to third world countries b) Those who have been recently exposed to hepatitis A c) Those working for the Centers for Disease Control and Prevention d) Homosexual women

a) Those traveling to third world countries International hepatitis A vaccine is suggested for travelers to regions where sanitation is poor and endemic HAV infections are high, children living in communities with high rates of HAV infection, homosexually active men, and users of illicit drugs.

The nurse is providing education for a patient diagnosed with essential hypertension. The nurse will state that the cause of this disorder is which of the following? a) Unknown b) Hyperaldosteronism c) Decreased renal blood flow d) Pheochromocytoma

a) Unknown Essential or primary hypertension has no identifiable cause, although there are many risk factors. The other options are causes of secondary hypertension, for which a cause can be identified and often treated.

A nurse is reviewing the laboratory data for a hospitalized client. The nurse would be most concerned about which finding? a) White blood cell (WBC) count of 2800/µL b) Eosinophils 1.5/ µL c) Platelet count of 200,000/µL d) Red blood cells 4.0/µL

a) White blood cell (WBC) count of 2800/µL Leukocytes are crucial to our defense against disease. The low WBC count in this patient would indicate that the patient's immune function may be compromised and the underlying cause of the problem needs to be investigated.

A nurse is caring for a client in the intensive care unit who has sustained severe trauma and now has developed disseminated intravascular coagulation (DIC). The nurse is aware that the client is experiencing: a) Widespread coagulation and bleeding in the vascular compartment b) Impaired platelet function due to vitamin K deficiency c) Bleeding due to structurally weak vessels resulting from vitamin C deficiency d) Thrombocytosis as a result of widespread infection

a) Widespread coagulation and bleeding in the vascular compartment DIC begins with massive activation of the coagulation sequence, which leads to fibrin deposition and formation of thrombi in the microcirculation of the body. DIC and its acute manifestations are related to the bleeding problems that occur.

Anaphylactic shock is directly associated with: a) type I hypersensitivity response. b) failure of the heart as a pump. c) loss of blood volume. d) bacterial blood infection.

a) type I hypersensitivity response. Anaphylaxis, a type of distributive shock, is a clinical syndrome that represents the most severe systemic allergic reaction. It results from an immunologic-mediated reaction in which vasodilator substances such as histamine are released into the blood. Bacterial blood infection is the cause of sepsis. Circulatory shock may be due to blood volume loss (hypovolemic shock) or myocardial infarction/ heart failure (cardiogenic shock).

A neonate has been diagnosed with hyperbilirubinemia and begun phototherapy. The infant's risk of encephalopathy will increase if: a) unconjugated bilirubin levels are not successfully reduced. b) conjugated bilirubin crosses the infant's blood-brain barrier. c) serum bilirubin cannot bind with RBC's for excretion. d) conjugated bilirubin reverts to unconjugated bilirubin.

a) unconjugated bilirubin levels are not successfully reduced. Hyperbilirubinemia reflects an elevated level of unconjugated bilirubin. Conjugated bilirubin cannot change to an unconjugated state. Bilirubin does not bind with RBC's in order to be excreted. Unconjugated bilirubin poses no risk of crossing the blood-brain barrier because it is not lipid soluble

The nurse is caring for a female client with cholelithiasis. When teaching the client about the disease, the nurse includes which of these points? a) "You have an inflammation of your gallbladder caused by an autoimmune process." b) "Gallstones have developed, which are typically composed of cholesterol." c) "You are unable to store bile in your gallbladder, which inhibits digestion of the foods you eat." d) "Stones in the gallbladder are most common in men. Your case is unusual."

b) "Gallstones have developed, which are typically composed of cholesterol." Cholelithiasis or gallstones is caused by precipitation of substances contained in bile, mainly cholesterol and bilirubin. It is most common in women, multiple pregnancies, those taking oral contraceptives or those who are obese.

The mother of a newborn infant questions why her baby needs a vitamin K injection immediately after birth. The best response by the nurse would be: a) "It is hospital policy to administer the injection to newborns." b) "Infants are not born with the normal intestinal bacteria that synthesize vitamin K for clotting." c) "Infants have a higher body fat content, which prohibits the absorption of vitamin K." d) "The infant was exposed to high levels of heparin upon birth."

b) "Infants are not born with the normal intestinal bacteria that synthesize vitamin K for clotting." Vitamin K is needed by newborns to begin the synthesis of blood clotting prior to their establishing the normal intestinal flora. While it may be hospital policy to administer vitamin K, this does not explain to the concerned mother why her newborn is getting an injection.

A patient who will be undergoing a Holter monitor examination would be given which of the following instructions? a) "Lay very still during the procedure for an accurate reading." b) "Keep a diary of your activities and symptoms throughout the examination." c) "You will need to have an intravenous injection prior to the start of the test." d) "Wear clothing that you can exercise in during the test."

b) "Keep a diary of your activities and symptoms throughout the examination." During the period of a patient wearing a Holter monitor (usually 48 hours), the patient will be instructed to maintain a diary of activities and symptoms so that the recorded electrocardiographic reading can be correlated with the diary events. The other options are not instructions appropriate for a Holter monitor examination.

The nurse knows that the main objective of the management of hypertension is to achieve a sustainable level of blood pressure below which of the following? a) 100/60 b) 140/90 c) 100/90 d) 140/60

b) 140/90 The main objective of hypertension management is to achieve a pressure of 140/90, although persons with renal disease or diabetes are advised to aim for 130/80 or lower. Persons with blood pressure between 140/90 and 120/80 are considered to be prehypertensive and should be monitored at regular intervals to detect any rise in pressure.

In hypovolemic shock, renal perfusion and urinary output decline. The nurse will monitor urinary output and knows that output below which of the following levels indicates inadequate renal perfusion? a) 40—20 mL/hour b) 20 mL/hour c) 60—40 mL/hour d) 80—60 mL/hour

b) 20 mL/hour In hypovolemia, renal vasculature is constricted in a compensatory attempt to circulate blood to more vital organs. Urinary output at 20 mL/hour or below indicates that renal perfusion is too inadequate.

The nurse is monitoring hourly urine output of a client diagnosed with hypovolemic shock. The nurse is most concerned if the client's output is: a) 80 mL/hour b) 20 mL/hour c) 60 mL/hour d) 40 mL/hour

b) 20 mL/hour Urine output decreases very quickly in hypovolemic shock. Compensatory mechanisms decrease renal blood flow as a means of diverting blood flow to the heart and brain. Oliguria of 20 mL/hour or less indicates inadequate renal perfusion.

A client with heart disease has the left ventricular ejection fraction measured. What is the normal left ventricular ejection when determined by angiocardiography? a) 35% - 55% b) 55% - 75% c) 65% - 85% d) 45% - 65%

b) 55% - 75% The normal left ventricular ejection fraction is usually 55 percent to 75 percent when determined by angiocardiography.

Which of the following is the most abundant type of plasma proteins? a) Globulins b) Albumin c) Fibrinogens d) Beta globulin

b) Albumin Albumin makes up approximately 54% of the plasma proteins. Although the others are made up plasma proteins, they are in the plasma in smaller numbers than is albumin.

When assessing the client with acute pancreatitis, which of these diagnostic tests, consistent with the disease, does the nurse anticipate will be altered? a) Creatine kinase b) Amylase and lipase c) The transaminases d) Glucose values

b) Amylase and lipase Serum amylase and lipase are the laboratory markers most commonly used to establish a diagnosis of acute pancreatitis.

Which of the following patients should most likely be assessed for orthostatic hypotension? a) A patient who has a history of poorly controlled type 1 diabetes. b) An elderly patient who has experienced two falls since admission while attempting to ambulate to the bathroom. c) A patient whose vision has become much less acute in recent months and who has noticed swelling in her ankles. d) A 78-year-old woman who has begun complaining of frequent headaches unrelieved by over-the-counter analgesics.

b) An elderly patient who has experienced two falls since admission while attempting to ambulate to the bathroom. Dizziness and syncope are characteristic signs and symptoms of orthostatic hypotension, and both predispose an individual to falls; this is especially the case among older adults. Headaches, edema, diabetes, and vision changes are not associated with orthostatic hypotension.

The heart consists of four valves. Which are the semilunar values? Select all that apply. a) Mitral b) Aortic c) Tricuspid d) Pulmonary

b) Aortic d) Pulmonary The heart's atrioventricular valves are the tricupsid and the mitral. The heart's semilunar valves are the pulmonary and aortic values.

Which of the following types of white blood cells are related to the connective tissue mast cells and respond in allergic and hypersensitivity reactions? a) B lymphocytes b) Basophils c) Neutrophils d) Granulocytes

b) Basophils The basophil is related to connective tissue mast cell because they contain similar granules and are believed to be involved in allergic and hypersensitivity reactions. Neutrophils, granulocytes, and B lymphocytes are not identified as being similar to mast cells.

A patient has developed disseminated intravascular coagulation (DIC). The nurse knows that which statements regarding DIC are true? Select all that apply. a) Fibrin removal increases. b) Bleeding may accompany coagulation. c) Generation of thrombin increases. d) Endogenous anticoagulation mechanisms are suppressed. e) Thrombotic occlusion occurs in small and midsized blood vessels.

b) Bleeding may accompany coagulation. c) Generation of thrombin increases. d) Endogenous anticoagulation mechanisms are suppressed. e) Thrombotic occlusion occurs in small and midsized blood vessels. In disseminated intravascular coagulation, generation of thrombin activates formation of fibrin clots which occlude small and midsized blood vessels. Platelets and coagulation factors are depleted as they form clots, and simultaneous bleeding can occur. Endogenous anticoagulant factors are suppressed, decreasing removal of fibrin.

The efficiency of the heart as a pump often is measured in the amount of blood the heart pumps per minute. Which is the correct formula to figure out the cardiac output? a) HR = CO + SV b) CO = SV x HR c) HR = SV x CO d) SV = HR x CO

b) CO = SV x HR Cardiac output = stroke volume x heart rate

The nurse's review of a client's laboratory results indicates that inflammation is absent and platelet levels are low. Which test was performed to provide the platelet count? a) Bone marrow aspiration b) Complete blood count c) Hematocrit d) Sedimentation rate

b) Complete blood count The complete blood count provides the number and characteristics of red blood cells, leukocytes, and platelets. Hematocrit is simply the percentage of RBCs and sedimentation rate indicates inflammation. Bone marrow aspiration is used to examine the stem cells.

While studying the physiology of the heart, the nursing students have learned that which of the following influence the blood flow in the coronary vessels that supply the myocardium? Select all that apply. a) Hypothalamus b) Compression of the intramyocardial vessels c) Autoregulatory mechanisms d) The aortic pressure e) Thyroid gland

b) Compression of the intramyocardial vessels c) Autoregulatory mechanisms d) The aortic pressure Blood flow in the coronary vessels that supply the myocardium is influenced by the aortic pressure, the autoregulatory mechanisms, and compression of the intramyocardial vessels by the contracting heart muscle.

A client has been admitted after being resuscitated from a cardiac arrest. The client is stated to be in Class I for electrocardiogram monitoring. What type of monitoring will the nurse perform for this client? a) Monitoring only with chest pain b) Continuous cardiac monitoring c) Intermittent cardiac monitoring d) No monitoring is indicated

b) Continuous cardiac monitoring The American Heart Association has published practice standards for electrocardiogram monitoring in hospital settings. The rating system has three categories. Class I clients require cardiac monitoring consistently. Class II clients may need monitoring but it is not essential. Class III clients do not have monitoring indicated.

When an acute MI occurs, many physiologic changes occur very rapidly. What causes the loss of contractile function of the heart within seconds of the onset of an MI? a) Inadequate production of glycogen with mitochondrial shrinkage b) Conversion from aerobic to anaerobic metabolism c) Conversion from anaerobic to aerobic metabolism d) Overproduction of energy capable of sustaining normal myocardial function

b) Conversion from aerobic to anaerobic metabolism The principal biochemical consequence of MI is the conversion from aerobic to anaerobic metabolism with inadequate production of energy to sustain normal myocardial function. As a result, a striking loss of contractile function occurs within 60 seconds of onset. None of the other answers occur.

Thrombocytosis is used to describe elevations in the platelet count above 1,000,000/μL. It is either a primary or a secondary thrombocytosis. Secondary thrombocytosis can occur as a reactive process due to what? a) Hirschsprung disease b) Crohn disease c) Megacolon d) Lyme disease

b) Crohn disease The common underlying causes of secondary thrombocytosis include tissue damage due to surgery, infection, cancer, and chronic inflammatory conditions such as rheumatoid arthritis and Crohn disease. Lyme disease, caused by a tick bite, does not cause thrombocytosis. Hirschsprung disease and megacolon are the same thing, and they are not inflammatory conditions.

To have stem cells for transplantation, clients are given specific agents to increase the quantity and migration of the cells from the bone marrow. What is the agent used to accomplish this? a) Human leukocyte antigen growth factor b) Cytokine growth factor c) Human growth factor d) Platelet growth factor

b) Cytokine growth factor Peripheral blood stem cells are harvested from the blood after the administration of a cytokine growth factor that increases the quantity and migration of the cells from the bone marrow. HLA is the abbreviation for human leukocyte antigen and is what is matched in umbilical cord blood. It does not increase either the quantity or the migration of peripheral blood stem cells. Platelets are a rich source of growth factors, but they do not, by themselves, increase either the quantity or the migration of peripheral blood stem cells. There is a human growth hormone but not a human growth factor.

A client has been prescribed a thiazide diuretic, hydrochlorothiazide (HCTZ), for the initial treatment of hypertension. What effect does the nurse know this drug will have to decrease blood pressure? a) Decrease renin release b) Decrease vascular volume c) Increase vasoconstriction d) Decrease heart rate

b) Decrease vascular volume Diuretics, such as the thiazides, loop diuretics, and the aldosterone antagonist (potassium-sparing) diuretics, lower blood pressure initially by decreasing vascular volume (by suppressing renal reabsorption of sodium and increasing salt and water excretion) and cardiac output.

A patient's electrocardiogram monitor begins to sound an alarm and shows sustained ventricular fibrillation. The patient is unconscious and without a pulse. Which of the following priority interventions should the nurse take? a) Administer IV push atropine b) Defibrillate the patient c) Notify the patient's attending physician d) Perform synchronized cardioversion

b) Defibrillate the patient Immediate defibrillation using a nonsynchronized, direct-current electrical shock is mandatory for ventricular fibrillation and for ventricular flutter that has caused loss of consciousness.

A client has developed constrictive pericarditis and myocardial hypertrophy. Select the most likely cause. a) Systolic dysfunction b) Diastolic dysfunction c) Backward failure d) Frontward failure

b) Diastolic dysfunction The conditions that cause diastolic dysfunction are those that impede expansion of the ventricles (i.e., pericarditis), those that increase ventricular wall thickness and reduce chamber size (i.e., myocardial hypertrophy), and those that delay diastolic relaxation (i.e., aging).

A client is admitted to the hospital with a diagnosis of deep vein thrombosis and started on intravenous heparin therapy. Seven days later, the client's lab values identify a rapid decrease in platelets. The health care provider recognizes this as: a) Immune thrombocytopenic purpura (ITP) b) Drug-induced thrombocytopenia c) Hemolytic-uremic syndrome (HUS) d) Thrombotic thrombocytopenic purpura (TTP)

b) Drug-induced thrombocytopenia Drug-associated thrombocytopenia presents with a rapid fall in the platelet count within 2 to 3 days of resuming a drug or 7 or more days after initiating a drug for the first time. The platelet count rises rapidly after the drug is discontinued. Heparin is one of the drugs that can cause this problem. Immune thrombocytopenic purpura (ITP) is an autoimmune disorder that results in platelet antibody formation and excess destruction of platelets. Thrombotic thrombocytopenic purpura (TTP) and hemolytic-uremic syndrome (HUS) are thrombotic microangiopathies.

A patient has been placed on Bactrim as treatment for community-acquired methicillin-resistant Staphylococcus aureus MRSA for 10 days. After taking the medication for 8 day, the patient comes to the doctor's office complaining of multiple nosebleeds over the past day. Laboratory work shows a platelet count of 80,000/μL. The nurse practitioner suspects which of the following conditions? a) Hypersplenic thrombocytopenia b) Drug-induced thrombocytopenia c) Drug-induced thrombocytosis d) Antiphospholipid syndrome

b) Drug-induced thrombocytopenia Many drugs have the potential to induced an antigen-antibody response including the formation of immune complexes that target and destroy platelets causing thrombocytopenia, which leads to increased bleeding. Bactrim is a sulfa-containing antibiotic that can cause this reaction. The reaction typically occurs around day 7 with people who have never taken the drug before and on days 2 to 3 for people who have taken the drug before. Thrombocytopenia resolves quickly once the drug is discontinued.

What are the components of blood? (Select all that apply.) a) Bile b) Electrolytes c) Ascites d) Enzymes

b) Electrolytes d) Enzymes Blood is made up of plasma, plasma proteins, fixed elements or blood cells, and substances such as hormones, enzymes, electrolytes, and by-products of cellular waste. Ascites is an accumulation of intraperitoneal fluid containing large amounts of protein and electrolytes. Bile is a secretion of the liver that is stored in the gallbladder.

An adult client with good overall health has reported headaches and dizziness in recent weeks. The nurse's assessment reveals blood pressure of 158/99 mm Hg, which is unprecedented for the client. On observation, the client's skin appears reddened, though he denies feeling warm. The nurse should anticipate what further diagnostic finding? a) Abnormal RBC indices coupled with low platelets. b) Elevated RBC, hemoglobin, and hematocrit. c) Normal RBC, hemoglobin and hematocrit with alterations in morphology. d) Further signs and symptoms of internal hemorrhage.

b) Elevated RBC, hemoglobin, and hematocrit. The client's presentation is characteristic of polycythemia. RBC, hemoglobin and hematocrit are likely to be elevated. Platelets are not likely to be low. Polycythemia is not linked to hemorrhage and RBC morphology is not directly affected

In the days following a tooth cleaning and root canal, a client has developed an infection of the thin, three-layered membrane that lines the heart and covers the valves. What is this client's most likely diagnosis? a) Myocarditis b) Endocarditis c) Vasculitis d) Pericarditis

b) Endocarditis The endocardium is a thin, three-layered membrane that lines the heart and covers the valves; infection of this part of the heart is consequently referred to as endocarditis.

A 16-year-old male client who has been diagnosed with infectious mononucleosis asks the health care provider what caused the condition. The best response would be: a) Abnormal cell nucleus development b) Epstein-Barr virus (EBV) c) Human immunodeficiency virus (HIV) d) Non-Hodgkin lymphoma

b) Epstein-Barr virus (EBV) EBV is the usual causative factor that results in the development of infectious mononucleosis. People with HIV may be at more risk for contacting mononucleosis. EBV may contribute to lymphoproliferative disorders, such as non-Hodgkin lymphoma.

The Rh-negative mother, who has become sensitized during a previous birth, gives birth to a Rh-positive infant. Which method will the practitioner chose to prevent kernicterus in the infant? a) Administration of erythropoietin b) Exchange transfusions c) Injection of Rh immune globulin d) Transfusion of fresh frozen plasma

b) Exchange transfusions After an Rh-negative mother has been sensitized, the Rh antibodies from her blood are transferred to subsequent infants through placental circulation. These antibodies react with the red cell antigens of the Rh-positive infant causing agglutination and hemolysis. There is a danger of kernicterus developing in the infant, resulting in severe brain damange or death. Exchange transfusions are administered after birth by removing and replacing the infant's blood volume with type O Rh-negative blood. The injection of Rh immune globulin is given to the Rh-negative mother post delivery to prevent sensitization. Erythropoietin and plasma administration do not prevent kernicterus.

There are two pathways that can be activated by the coagulation process. One pathway begins when factor XII is activated. The other pathway begins when there is trauma to a blood vessel. What are these pathways? a) Clotting and bleeding pathways b) Extrinsic and intrinsic pathways c) Factor and trauma pathways d) Inner and outer pathways

b) Extrinsic and intrinsic pathways The coagulation process results from the activation of what has traditionally been designated the intrinsic or the extrinsic pathways. The intrinsic pathway, which is a relatively slow process, begins in the circulation with the activation of factor XII. The extrinsic pathway, which is a much faster process, begins with trauma to the blood vessel or surrounding tissues and the release of tissue factor, an adhesive lipoprotein released from the subendothelial cells. The terminal steps in both pathways are the same: the activation of factor X and the conversion of prothrombin to thrombin. All other answers do not exist in the formation of clots.

The physician suspects a client may have developed pancreatitis, and the physician has ordered laboratory blood work. Diagnosis-confirming results would identify: a) Change in platelet count and prothrombin level b) High serum amylase and lipase c) Chymotrypsin level and fibrinogen level d) Altered alkaline phosphatase and red blood cell count

b) High serum amylase and lipase Laboratory criteria for the diagnosis of pancreatitis are serum amylase or lipase greater than three times the upper limit of normal. Altered alkaline phosphatase and prothrombin level may indicate liver disease. Chymotrypsin digests proteins in the intestine.

When caring for the client with acute pancreatitis, which of these alterations does the nurse recognize is consistent with the disease? a) Leukopenia b) Hyperglycemia c) Hypertension d) Polyuria

b) Hyperglycemia Serum amylase and lipase are the laboratory markers most commonly used to establish a diagnosis of acute pancreatitis. The white blood cell count may be increased, and hyperglycemia and an elevated serum bilirubin level may be present.

A 20-year-old college student has a pelvic fracture and a severed leg from a motorcycle accident. She lost several units of blood. When the student arrived in the emergency department, her blood pressure was very low, her pulse was high, and her skin was pale. The nurse knows that this patient has developed which of the following types of shock? a) Obstructive b) Hypovolemic c) Cardiogenic d) Distributive

b) Hypovolemic Hypovolemic shock occurs when there has been extensive blood or body fluid loss. Cardiogenic shock occurs when the heart loses its ability to pump sufficient blood to meet oxygenation needs of body tissues. Distributive shock is loss of blood vessel tone, resulting in expansion of the vascular compartment. Obstructive shock is an inability of the heart to fill or empty properly.

A client has just returned from his surgical procedure. During initial vital sign measurements, the nurse notes that the client's heart rate is 111 beats/minute and the BP is 100/78 (borderline low). In this early postoperative period, the nurse should be diligently monitoring the client for the development of: a) Renal failure due to an overdose of medication b) Hypovolemic shock due to acute intravascular volume loss c) Pulmonary embolism due to development of deep vein thrombosis d) Side effects from versed administration causing excessive vasoconstriction

b) Hypovolemic shock due to acute intravascular volume loss Hypovolemic shock is characterized by diminished blood volume such that there is inadequate filling of the vascular compartment. Hypovolemic shock also can result from an internal hemorrhage or from third-space losses, when extracellular fluid is shifted from the vascular compartment to the interstitial space or compartment, without fluid movement in/out of the cells. Within seconds after the onset of hemorrhage or the loss of blood volume, compensatory manifestations of tachycardia, vasoconstriction, and other signs of sympathetic and adrenal medullary activity appear. There is no indication that this client has developed a pulmonary embolism, is having side effects from versed administration, or is going into renal failure due to an overdose of medication.

The health care provider has completed the assessment of a client who presented to the emergency department with jaundice. The provider determines that the jaundice is a result of: a) Increased conjugation of bilirubin b) Impaired uptake of bilirubin by the liver c) Increased red blood cell development d) Decreased oxygen demands at birth

b) Impaired uptake of bilirubin by the liver The four major causes of jaundice are excessive destruction of RBCs, impaired uptake of bilirubin by the liver cells, decreased conjugation of bilirubin, and obstruction of the bowel flow in the canaliculi of the hepatic lobules or in the intrahepatic or extrahepatic bile ducts. Jaundice would not occur as a result of decreased oxygen. Jaundice in newborns is the result of the breakdown of fetal hemoglobin.

The health care team is developing a plan of care for a client diagnosed with congestive heart failure (CHF). The primary treatment goal would be: a) Placing a stent for fluid drainage from the heart b) Improving quality of life by relieving symptoms c) Maintaining higher oxygen levels to decrease the work of breathing d) Eliminating CHF through curing the disease

b) Improving quality of life by relieving symptoms A primary treatment goal for a client with CHF is to improve the quality of life through symptom management. CHF will not be cured, and maintaining a higher oxygen level will assist with dyspnea associated with CHF. A stent is not an option for treatment of CHF.

The nurse is reviewing lab results of a client who has liver failure. The nurse determines that the client is at an increased risk for bleeding when the results include: a) Decreased number of red blood cells b) Increased prothrombin time c) Increased levels of vitamin K d) Increased platelet count

b) Increased prothrombin time Clients with liver failure have malabsorption of vitamin K (decrease), which impairs the synthesis of clotting factors. An increased prothrombin time places the client at risk for bleeding. Factors V, VII, IX, and X, prothrombin, and fibrinogen are synthesized by the liver; their decline in liver disease contribute to bleeding disorders. A decrease in RBC will not cause an increase in bleeding. An increased platelet count will cause the blood to clot.

Nursing students who are studying for their upcoming cardiac exam are discussing how the heart could possibly continue to beat once removed from the body. One of the students explains that this phenomenon is directly related to automaticity. What is automaticity? a) Inherent discharge rate of 60-80 b) Inherent spontaneous action-potential c) Inherent low-impulse conductivity d) Inherent discharge rate of 40-60

b) Inherent spontaneous action-potential The heart has four inherent properties essential in the development and conduction of cardiac rhythms. The property of automaticity is the ability of certain cells in the myocardium to automatically or spontaneously initiate an electrical impulse called an action potential. In a normally functioning heart the rate is controlled by the sinoatrial (SA) node.

A nurse is teaching a patient with newly diagnosed hypertension about antihypertensive drug therapy. The nurse determines that the patient understands when the patient correctly describes which of the following as the mechanism of action of an angiotensin-converting enzyme (ACE) inhibitor? a) Decreases vascular volume by suppressing renal reabsorption of sodium b) Inhibition of the conversion of angiotensin I to angiotensin II, decreasing angiotensin II levels and reducing its effect on vasoconstriction c) Decreases the heart rate, which will cause a decrease in cardiac output d) Inhibit the movement of calcium into cardiac and vascular smooth muscle

b) Inhibition of the conversion of angiotensin I to angiotensin II, decreasing angiotensin II levels and reducing its effect on vasoconstriction The ACE inhibitors act by inhibiting the conversion of angiotensin I to angiotensin II, thus decreasing angiotensin II levels and reducing its effect on vasoconstriction.

The nurse is assessing a client diagnosed with anemia and notes that the client's skin and mucous membranes are pale. The nurse interprets this as: a) Presence of systolic murmur b) Insufficient hemoglobin c) Tissue hypoxia to the brain d) Changes in blood viscosity

b) Insufficient hemoglobin The redistribution of the blood from cutaneous tissues or a lack of hemoglobin causes pallor of the skin, mucous membranes, conjunctiva, and nail beds. Tissue hypoxia to the brain causes headache, faintness, and dim vision but is not the direct cause of pallor. Changes in blood viscosity may lead to a systolic murmur.

Some cytokines stimulate the growth and production of new blood cells. Other cytokines support the proliferation of stem cells in the human body. Which cytokines support the proliferation of stem cells in the human body? a) Transforming growth factor, interferons, and tumor necrosis factor b) Interleukins, interferons, and tumor necrosis factor c) Granulocytes, B-cell growth factor, and interferons d) Interleukins, T-cell growth factor, and colony-stimulating factors

b) Interleukins, interferons, and tumor necrosis factor Cytokines, such as the interleukins, interferons, and tumor necrosis factor, support the proliferation of stem cells and the development of lymphocytes, and act synergistically to aid the multiple functions of the colony-stimulating factors. B-cell growth factor, T-cell growth factor, colony-stimulating factors, and transforming growth factor are all cytokines that stimulate the growth and production of new blood cells.

A female patient comes to the clinic with symptoms of fatigue and heavy menses over the last 6 months. Laboratory tests reveal a microcytic hemochromic anemia. Based on these results, the nurse anticipates teaching the patient about which type of anemia? a) Aplastic anemia b) Iron deficiency anemia c) Anemia related to kidney disease d) Sickle cell anemia

b) Iron deficiency anemia The red blood cell indices identified that it was a microcytic hemochromic anemia, specifically iron deficiency.

A nursing instructor is explaining arterial circulation to a group of nursing students. Which of the following is the most appropriate information for the nurse to provide? a) The vascular pressure is lower than the venous circulation. b) It contains one sixth of the total blood volume. c) Arteries are more distensible than veins. d) It contains approximately two thirds of the total blood volume.

b) It contains one sixth of the total blood volume. The arterial side of the circulation contains approximately one sixth of the blood volume. Arterial circulation exerts much greater than the pressure on the venous side of the circulation. The venous side contains approximately two thirds of the blood. Because of their thicker, muscular walls arteries are less distensible than veins.

Which of the following is the greatest diagnostic limitation of an electrocardiogram (ECG)? a) It is an expensive diagnostic tool. b) It documents only current cardiac function. c) It is an invasive procedure. d) It can interfere with the heart's conduction system.

b) It documents only current cardiac function. The resting ECG is the first approach to the clinical diagnosis of disorders of cardiac rhythm and conduction, but it is limited to events that occur during the period the ECG is being monitored. The other options are not accurate statements regarding an ECG.

Which of the following is true regarding pulmonary circulation? a) The system functions with an increased arterial pressure to circulate through the distal parts of the body. b) It is a low-pressure system that allows for improved gas exchange. c) It is the larger of the two circulatory systems. d) It consists of the left side of the heart, the aorta, and its branches.

b) It is a low-pressure system that allows for improved gas exchange. The pulmonary circulation consists of the right heart and the pulmonary artery, capillaries, and veins. It is the smaller of the systems and functions at a lower pressure to assist with gas exchange.

What laboratory markers are most commonly used to diagnose acute pancreatitis? a) Cholesterol and triglycerides b) Lipase and amylase c) Amylase and cholesterol d) Lipase and triglycerides

b) Lipase and amylase Serum amylase and lipase are the laboratory markers most commonly used to establish a diagnosis of acute pancreatitis. Cholesterol and triglycerides are not used as laboratory markers for acute pancreatitis.

A 30-year-old male brought to the emergency department has the following admission data: Blood pressure 50/30, pulse 100, respiratory rate 12, temperature 101°F. The nurse does not know the exact cause but does know that patient is in shock because of which of the following readings? a) Slow respiratory rate b) Low blood pressure c) Elevated pulse rate d) Elevated temperature

b) Low blood pressure Low blood pressure is characteristic of most occurrences of shock. Elevated pulse rate may be a compensatory effort to support blood pressure. Elevated temperature can occur with septic shock. Decreased respiratory rate is not characteristic of shock.

Which of the following related circulatory complications can result from surgical treatment for metastatic breast cancer? a) Hypotension upon standing b) Lymphedema in the affected arm c) Tachycardia when at rest d) Irregular heart rate

b) Lymphedema in the affected arm Involvement of lymphatic structures by malignant tumors and removal of lymph nodes at the time of cancer surgery are common causes of lymphedema. The other options are not related to the surgery that would have removed any affected lympth nodes.

A client is admitted for observation due to abnormal heart sounds, pulmonary congestion, nocturnal paroxysmal dyspnea, and orthopnea. Upon auscultation a low-pitched, rumbling murmur, best heard at the apex of the heart, is also heard. Which condition does the client likely have? a) Aortic valve stenosis b) Mitral valve stenosis c) Mitral valve prolapse d) Aortic valve prolapse

b) Mitral valve stenosis Mitral valve stenosis represents the incomplete opening of the mitral valve during diastole, with left atrial distention and impaired filling of the left ventricle with associated symptoms. Mitral prolapse and aortic valve disorders will lead to the development of cardiomyopathies.

The nurse working in the emergency room triages a client who comes in with complaints of chest pain, shortness of breath, sweating and elevated anxiety. The physician suspects a myodardial infarction. The client is given a nitrate, which does nothing for his pain. Which of the following medications should the nurse suspect the doctor will order next for the pain? a) Codeine b) Morphine c) Demerol d) Fentanyl

b) Morphine Although a number of analgesic agents have been used to treat pain of myocardial infarction, morphine is the drug of choice and is usually indicated if chest pain is unrelieved with oxygen and nitrates.

ECG monitoring has been found to be more sensitive than a client's report of symptoms when identifying transient ongoing myocardial ischemia. Why is this? a) ECG monitoring is reliable only when the client remains still. b) Most ECG-detected ischemic events are clinically silent. c) The ECG can look at ischemic events from different directions. d) Most ECG-detected ischemic events cause a great deal of pain.

b) Most ECG-detected ischemic events are clinically silent. Persons with acute coronary syndrome are at risk for developing extension of an infarcted area, ongoing myocardial ischemia, and life-threatening arrhythmias. Research has revealed that 80% to 90% of ECG-detected ischemic events are clinically silent. Thus, ECG monitoring is more sensitive than a client's report of symptoms for identifying transient ongoing myocardial ischemia. Other answers are incorrect.

A teenager is diagnosed with hypertension. The nurse knows that risk factors for hypertension in children and adolescents include which of these? Select all that apply. a) Being female b) Obesity c) High salt consumption d) Inactive lifestyle e) Poor performance in school

b) Obesity c) High salt consumption d) Inactive lifestyle The prevalence of hypertension in children and adolescents in increasing due to increases in obesity, and lifestyle factors such as lack of exercise and consumption of high-calorie, high-salt foods. Gender is not a factor nor is academic performance.

A client with a long history of stable angina suddenly experiences substernal pain that radiates to the left arm, neck, and jaw. He describes the pain as severe and feels as if he is suffocating. He has taken nitroglycerin and not experienced any relief. The client is most likely experiencing: a) Pneumonia b) Onset of STEMI c) Acute respiratory distress syndrome (ARDS) d) Gastroesophageal reflux disease (GERD)

b) Onset of STEMI The onset of STEMI involves abrupt and significant chest pain. The pain typically is severe, often described as being constricting, suffocating, and crushing. Substernal pain that radiates to the left arm, neck, or jaw is common, although it may be experienced in other areas of the chest and back. Unlike that of angina, the pain associated with MI is more prolonged and not relieved by rest or nitroglycerin.

A monitored hospitalized patient with a pulmonary embolism has been in atrial fibrillation (AF) for 4 days. The nurse observes the rhythm spontaneously convert to a normal sinus rhythm. Which of the following forms of AF is this? a) Persistent b) Paroxysmal c) Chronic d) Permanent

b) Paroxysmal AF is characterized as rapid disorganized atrial activation and uncoordinated contraction by the atria. It is classified into three categories: paroxysmal, persistent, and permanent. Paroxysmal AF self-terminates and lasts no longer than 7 days, whereas persistent lasts greater than 7 days and usually requires intervention such as a cardioversion. AF is classified as permanent when attempts to terminate are failed and the the person remains in AF. The symptoms of chronic AF vary. Some people have minimal symptoms, and others have severe symptoms, particularly at the onset of the arrhythmia.

Which of these assessments does the nurse recognize is consistent with signs and symptoms of acute pancreatitis? a) Hypertension b) Periumbilical pain c) Pulse of 58 beats per minute d) Groin pain

b) Periumbilical pain Abdominal pain is a cardinal manifestation of acute pancreatitis. The pain is usually located in the epigastric or periumbilical region and may radiate to the back, chest, or flank areas. Physical examination findings include fever, tachycardia, hypotension, severe abdominal tenderness, respiratory distress, and abdominal distention.

A nurse is reviewing a client's complete blood count (CBC) which indicates thrombocytopenia. Based on this result which action should the nurse include in the plan of care? a) Increase intake of iron-rich foods. b) Place the client on bleeding precautions. c) Place the client on contact isolation. d) Encourage increased oral fluids.

b) Place the client on bleeding precautions. Thrombocytopenia is a decreased number of platelets, which places the patient at high risk for bleeding.

A 69-year-old client who is obese and has a diagnosis of angina pectoris has been prescribed clopidogrel (Plavix) by his primary care provider. The client asks, "Why do I need this medication? It won't help my chest pain." The best response would be this medication: a) Inhibits the intrinsic clotting pathway to keep RBCs from clumping together in the heart vessels b) Prevents the blood cells from forming a clot in your heart vessels c) Activates plasminogen, which converts to plasmin to digest clots in your heart vessels d) Inactivates calcium ions, thereby preventing blood clotting

b) Prevents the blood cells from forming a clot in your heart vessels Clopidogrel (Plavix) is a common platelet aggregation inhibitor. It does not achieve a therapeutic effect by activating plasminogen, inhibiting the intrinsic pathway, or inactivating calcium ions.

A patient experiencing a sinus arrest would demonstrate which of the following symptoms or findings? a) P-wave that occurs more frequently than expected b) Prolonged periods of asystole demonstrated on an electrocardiogram c) Spontaneous persistent sinus bradycardia d) Heart rate of greater than 100 beats per minute

b) Prolonged periods of asystole demonstrated on an electrocardiogram The patient with sinus arrest refers to failure of the sinoatrial node to discharge and results in an irregular pulse, prolonged periods of asystole, and predisposition to other arrthythmias. The other options do not demonstrate the symptoms demonstrated during a sinus arrest.

The nurse knows that vasopressin (antidiuretic hormone, ADH) plays which role in blood pressure control? a) Decreases blood volume b) Promotes retention of water c) Increases osmolality of body fluids d) Produces prolonged increase in blood pressure

b) Promotes retention of water Vasopressin increases the permeability to water of the collecting ducts of the kidney, promoting renal reabsorption of water. Vasopressin is released in response to decreased blood volume or increased osmolality. Vasopressin also constricts some vascular beds. However, it cannot maintain an increase in blood pressure.

The nurse is caring for a client diagnosed with sickle cell disease. Select the most important factor for the nurse to be aware of that may cause the cells to sickle. a) Rapid administration of intravenous fluids b) Reduced oxygen tension while the client sleeps c) Acute chest syndrome d) Presence of pain in the client's joints

b) Reduced oxygen tension while the client sleeps Factors associated with sickling include cold, stress, physical exertion, infection, dehydration and illnesses that cause hypoxia, dehydration, or acidosis. Even such trivial incidents as reduced oxygen tension induced by sleep may contribute to the sickling process. Pain and acute chest syndrome are complications of sickle cell disease

A client has had an acute myocardial infarction. The brother of the client has a history of angina. The client asks how they will know if the brother's pain is angina or if the brother is actually having an MI. Which statement is correct? a) Chest pain with angina only occurs during the day; MI pain is more likely at night. b) Rest and intake of nitroglycerin relieve chest pain with angina; they do not relieve chest pain with an MI. c) Chest pain with angina only occurs at rest; MI pain occurs during a stressful time. d) Pain is more severe and lasts longer with angina than with an MI.

b) Rest and intake of nitroglycerin relieve chest pain with angina; they do not relieve chest pain with an MI. Rest and intake of nitroglycerin relieve chest pain with angina but not with an MI. Pain with angina and MI is an subjective symptom for each client. Pain with angina and MI can occur at a variety of times.

A 20-year-old college student being treated for a kidney infection developed a temperature of 104ºF in spite of treatment with antibiotics. Her pulse was high, her blood pressure was low, and her skin was hot, dry, and flushed. The nurse knows that this patient most likely is experiencing which of the following types of shock? a) Anaphylactic b) Septic c) Cardiogenic d) Neurogenic

b) Septic Septic shock can result with the body's response to a severe infection. Neurogenic shock is a loss of sympathetic (adrenergic) control of systemic blood vessel tone. Cardiogenic shock is a loss of cardiac efficiency, and anaphylactic shock is a severe allergic reaction.

Which of the following does the nurse know can lead to right-sided heart failure? a) Myocardial infarction b) Severe pneumonia c) Chronic kidney disease d) Uncontrolled hypertension

b) Severe pneumonia Severe pneumonia can induce pulmonary hypertension, which would impede blood flow from the right ventricle. Myocardial infarction and uncontrolled hypertension cause left-sided failure. Renal dysfunction can result from heart failure or hypertension.

A client who has been diagnosed with acute symptomatic viral hepatitis is now in the icteric period. The nurse would expect the client to manifest: a) Severe anorexia b) Severe pruritus and liver tenderness c) Disappearance of jaundice d) Chills and fever

b) Severe pruritus and liver tenderness Severe pruritus and liver tenderness are common during the icterus period. Chills, fever, and severe anorexia occur during the prodromal period. The disappearance of jaundice occurs in the convalescent phase.

What is the primary cause of heart failure in infants and children? a) Idiopathic heart disease b) Structural heart defects c) Reactions to medications d) Hyperkalemia

b) Structural heart defects Structural (congenital) heart defects are the most common cause of heart failure in children. The other answers are not correct.

Which of the following patients is at the greatest risk of developing rheumatic heart disease? a) Young adult with viral meningitis b) Teenager with untreated strep throat c) Child with impetigo on the face d) Older adult with shingles

b) Teenager with untreated strep throat Rheumatic fever is caused by group A (beta-hemolytic) streptococcal throat infection. Although the same bacteria cause the skin infection called impetigo, it is not known to cause rheumatic heart disease. Viral infections such as meningitis and shingles (herpes zoster) do not cause rheumatic heart disease.

A client has just been admitted to the cardiac intensive care unit with a diagnosis of infective endocarditis. His wife appears distraught and asks the nurse what caused this to happen to her husband. What would be the nurse's best response? a) Sometimes a parasite is involved. b) The most common cause is a staph infection. c) It can be attributed to drug abuse. d) Let's not worry about the cause as we need to focus on getting him better.

b) The most common cause is a staph infection. Staphylococcal infections have now emerged as the leading cause of infective endocarditis, with streptococci & enterococci as the other two most common causes. Informing the client's wife about the drug abuse connection infers that you think he is a drug addict and is not therapeutic. Parasites are never the cause and telling the patient to focus on her husband also is not appropriate as this is a genuine concern for her.

A 16-year-old girl has been admitted to the emergency department after ingesting 20 g of acetaminophen (Tylenol) in a suicide attempt. The care team would recognize that this patient faces risk for which of these complications? a) Portal hypertension b) Toxic hepatitis c) Hepatitis D virus infection d) Secondary biliary cirrhosis

b) Toxic hepatitis Among the manifestations of the direct, predictable liver injuries that accompany overdoses of acetaminophen is toxic hepatitis with tissue necrosis. HDV infection and secondary biliary cirrhosis do not typically result from hepatotoxic drugs. While portal hypertension may develop, it would result from the toxicicity.

Stem cell transplantation has been shown to provide potential cures for diseases such as aplastic anemia and the leukemias. What are the sources of stem cells used for transplant? a) Peripheral blood cells and immature embryonic cells b) Umbilical cord blood and bone marrow c) Peripheral blood and yellow bone marrow d) Bone marrow and immature neural cells

b) Umbilical cord blood and bone marrow Sources of the stem cells include bone marrow and umbilical cord blood. All replenish the recipient with a normal population of pluripotent stem cells. Immature embryonic cells do not necessarily contain stem cells. Immature neural cells are not stem cells. Yellow bone marrow does not make blood cells, so it would not contain stem cells.

The nurse is reviewing the complete blood count (CBC) and white blood cell (WBC) differential of a client admitted with lower right abdominal pain. Which laboratory results are the most important for the nurse to communicate to the health care provider? a) Monocytes 4% b) White blood cells (WBCs) 18,500/µL c) Platelet count 168,000/µL d) Basophils 2/µL

b) White blood cells (WBCs) 18,500/µL Leukocytes are crucial to our defense against disease. The high WBC count in this would indicate that the patient has an inflammatory or infectious process ongoing, which may be the cause of the patient's pain, and that further diagnostic testing is needed.

Cardiac tamponade and pericardial effusion can be life-threatening when the pericardial sac _______ and ______ the heart. a) thickens; stretches b) fills rapidly; compresses c) ruptures; releases d) contracts; friction rubs

b) fills rapidly; compresses Rapid accumulation of effusion fluid or blood in the pericardial sac causes cardiac tamponade, resulting in compression of the heart. This leads to cardiac standstill or failure. Pericardial sac thickening due to inflammation can restrict the heart, rather than allow stretching. Rupture of the sac is pathologic, resulting in heart expansion. A friction rub sound (rubbing between the inflamed pericardial surfaces) is characteristic of acute pericarditis. Constrictive pericarditis causes scar tissue formation that contracts and interferes with filling.

A client has been diagnosed with alcohol-induced liver disease. He admits to the nurse, "I know what the lungs do, and I know what the heart does, but honestly, I have no idea what the liver does in the body." The nurse should tell the client that the liver: a) is responsible for the absorption of most dietary nutrients, as well as the production of growth hormones. b) metabolizes most components of food and also cleans the blood of bacteria and drugs. c) maintains a balanced level of electrolytes and pH in the body and stores glucose, minerals, and vitamins. d) contributes to the metabolism of ingested food and provides the fluids that the GI tract requires.

b) metabolizes most components of food and also cleans the blood of bacteria and drugs. Protein, carbohydrate, and fat metabolism are performed by the liver. As well, it metabolizes drugs and removes bacteria by Kupffer cells. Absorption of nutrients takes place in the intestines and the liver does not produce the bulk of fluids secreted in the GI tract. The liver does not have a primary role in the maintenance of acid--base or electrolyte balance.

An intensive care unit nurse is caring for a client who suffered a myocardial infarction involving the anterior wall, and notes a change in the cardiac rhythm. The rhythm has a PR interval that does not change, but there are twice as many P waves as there are R waves. The nurse prepares for a temporary pacemaker insertion because the client has developed: a) a benign second-degree AV block. b) second-degree, type 2 AV block. c) complete, third-degree heart block. d) second-degree, Mobitz type I.

b) second-degree, type 2 AV block. Second-degree, type 2 AV blocks are frequently seen following anterior wall myocardial infarctions, and because this condition can easily progress to a complete heart block a temporary or permanent pacemaker is often necessary. Second-degree, type 2 AV block is also called a Mobitz II.

A client with a history of cancer that metastasized to the liver has arrived at the outpatient clinic to have a paracentesis performed. The physician anticipates that the client will have more than 5 L of fluid removed. The physician has prescribed intravenous albumin following the procedure. The client asks why she needs, "more fluids in my vein?" The nurse responds: a) "Albumin will stay in your blood vessels a long time so that you will not seep out more fluid in your belly for at least a few weeks." b) "After the albumin, your potassium level will stay steady and you should keep excess water weight off for several weeks." c) "Albumin is a volume expander. Since a lot of fluid was removed, you have a decrease in your vascular volume, so without this albumin, your kidneys will try to reabsorb and hold onto water." d) "Albumin works like your diuretics to help you get rid of excess fluid thru your kidneys, it's just more potent than your home water pills."

c) "Albumin is a volume expander. Since a lot of fluid was removed, you have a decrease in your vascular volume, so without this albumin, your kidneys will try to reabsorb and hold onto water." Large-volume paracentesis (removal of 5 L or more of ascitic fluid) may be done in persons with massive ascites and pulmonary compromise. Because the removal of fluid produces a decrease in vascular volume along with increased plasma renin activity and aldosterone-mediated sodium and water reabsorption by the kidneys, a volume expander such as albumin usually is administered to maintain the effective circulating volume. The other distractors are all incorrect.

The father of a 2-year-old boy recently diagnosed with hemophilia A asks the nurse how to prevent complications for his son. The best response would be: a) "Do not allow the child to play outside or with other children." b) "Hemophilia A will no longer be a concern once treated with a blood transfusion." c) "Avoid administering aspirin and nonsteroidal anti-inflammatory drugs." d) "Small bleeding in the joints is to be expected and can be treated at home."

c) "Avoid administering aspirin and nonsteroidal anti-inflammatory drugs." The prevention of trauma is important in persons with hemophilia, and aspirin and nonsteroidal anti-inflammatory drugs should be avoided. Children with hemophilia should not be prevented from interaction with other children, and hemophilia will not be resolved with a blood transfusion. All bleeds require medical attention.

A nurse is observing a patient's cardiac status by telemetry monitoring. On the monitor, the P wave changes shape and an impulse frequently occurs before the next expected sinoatrial (SA) node impulse. The nurse interprets this rhythm by stating which of the following?? a) "The patient is in normal sinus rhythm." b) "The patient is experiencing a junctional rhythm." c) "The patient is experiencing premature atrial contractions (PACs)." d) "The patient is experiencing premature ventricular contractions."

c) "The patient is experiencing premature atrial contractions (PACs)." PACs are contractions that originate in the atrial conduction pathways or atrial muscle cells and occur before the next expected SA node impulse. This impulse to contract usually is transmitted to the ventricle and back to the SA node. The location of the ectopic focus determines the configuration of the P wave. In general, the closer the ectopic focus is to the SA node, the more the ectopic complex resembles a normal sinus complex. Retrograde transmission to the SA node often interrupts the timing of the next sinus beat, such that a pause occurs between the two normally conducted beats.

A client tells the nurse that he has recently begun to take over-the-counter (OTC) calcium supplements to ensure that his blood will clot. The best response by the nurse would be: a) "It is a good idea; you will prevent bleeding." b) "Take the supplement twice a day with milk to increase absorption." c) "This is not necessary, unless it has been prescribed by your health care provider." d) "It will not cause problems, and it will keep your bones strong."

c) "This is not necessary, unless it has been prescribed by your health care provider." The body usually has sufficient amounts of calcium for these reactions. Most of the coagulation factors are proteins synthesized in the liver. Vitamin K is necessary for the synthesis of factors VII, IX, and X; prothrombin (factor II); and proteins C and S. Calcium (factor IV) is required in all but the first two steps of the clotting process. Unless prescribed, excessive calcium can contribute to electrolyte imbalance and complications.

A heart failure client has an echocardiogram performed revealing an ejection fraction (EF) of 40%. The nurse knows this EF is below normal and explains to the client: a) "You need to increase the amount of exercise you do to get your heart muscle back in shape." b) "This means you have a lot of pressure built-up inside your heart." c) "This means your heart is not pumping as much blood out of the heart with each beat." d) "Your ventricular muscle is getting too stiff to beat normally."

c) "This means your heart is not pumping as much blood out of the heart with each beat." Ejection fraction is the percentage of diastolic volume ejected from the heart [left ventricle] during systole. Stroke volume is determined by the difference between end-diastolic and end-systolic volumes. Cardiac output is determined by stroke volume and heart rate. Cardiac reserve refers to the maximum percentage of increase in cardiac output that can be achieved above the normal resting level.

A 38-year-old female is considering the use of oral contraceptives as a method of birth control. She asks her health care provider if any risks would be involved. The best response would be: a) "Have you talked about this with your partner?" b) "There are no risks involved" c) "Yes, risks increase with age." d) "Why do you want the pill?"

c) "Yes, risks increase with age." The incidence of stroke, thromboemboli, and myocardial infarction is greater in women who use oral contraceptives, particularly those older than 35 years of age and those who smoke tobacco. The other options do not address the client's question of risk.

A client asks why he has not had major heart damage since his cardiac catheterization revealed he has 98% blockage of the right coronary artery. The nurse's best response is: a) "You are just a lucky person since most people would have had a massive heart attack by now." b) "You must have been taking a blood thinner for a long time." c) "You have small channels between some of your arteries, so you can get blood from a patent artery to one severely blocked." d) "With this amount of blockage, your red blood cells get through the vessel one-by-one and supply oxygen to the muscle."

c) "You have small channels between some of your arteries, so you can get blood from a patent artery to one severely blocked." Collateral circulation is a mechanism for the long-term regulation of local blood flow. In the heart, anastomotic channels exist between some of the smaller arteries. These channels permit perfusion of an area by more than one artery. When one artery becomes occluded, these anastomotic channels increase in size, allowing blood from a patent artery to perfuse the area supplied by the occluded vessel. For example, persons with extensive obstruction of a coronary blood vessel may rely on collateral circulation to meet the oxygen needs of the myocardial tissue normally supplied by that vessel. There is no indication that the client is on a blood thinner.

A patient with persistent primary hypertension remains apathetic about his high blood pressure, stating, "I don't feel sick, and it doesn't seem to be causing me any problems that I can tell." How would the nurse best respond to this patient's statement? a) "That's true, but it's an indicator that you're not taking very good care of yourself." b) "Actually, high blood pressure makes you very susceptible to getting diabetes in the future." c) "You may not sense any problems, but it really increases your risk of heart disease and stroke." d) "You're right, but it's still worthwhile to monitor it in case you do develop problems."

c) "You may not sense any problems, but it really increases your risk of heart disease and stroke." Hypertension is a highly significant risk factor for heart disease and stroke. It would be inappropriate to promote monitoring without promoting lifestyle modifications or other interventions to lower the patient's blood pressure, or teaching the patient about the deleterious effects of hypertension. It is likely unproductive to simply characterize the patient's hypertension as demonstrating that he does not "take care" of himself. Hypertension is not a risk factor for the development of diabetes mellitus.

A client arrives at the emergency room with dizziness and a near syncopal episode. Vital signs include a heart rate of 46 and blood pressure of 86/50. The cardiac monitors show regular rhythm as above. The client states his physician has been running blood work to rule out hypothyroidism. Based on the rhythm what does the nurse report the client has? a) A symptomatic third degree AV block b) A symptomatic tachyarrhythmia c) A symptomatic bradyarrhythmia d) Wolff-Parkinson White syndrome

c) A symptomatic bradyarrhythmia This client is suffering from a symptomatic bradyarrhythmia as evidenced by a heart rate of 46 with normal P waves before every QRS, normal PR interval and symptoms of dizziness and syncope. Bradyarrhythmias can decrease oxygen delivery to the brain, along with other vital organs, causing symptoms such as dizziness, lightheadedness, fatigue and syncope.

The leukemia most commonly found in children and adolescents is recognized as: a) Chronic lymphocytic leukemia (CLL) b) Chronic myelocytic leukemia (CML) c) Acute lymphocytic leukemia (ALL) d) Acute myeloid leukemia (AML)

c) Acute lymphocytic leukemia (ALL) Among children and adolescents, ALL is the most common type, accounting for 75% of leukemia cases. In adults 20 years of age and older, the most common types are CLL (38%) and AML (30%)

A nurse is caring for four patients. Which of the following patients is most at risk for atrial fibrillation (AF)? a) A middle-aged female with shortness of breath who takes an aspirin daily b) A male adult postcarotid endarterectomy c) An elderly male who is 2 days postcoronary artery bypass surgery d) A female adult with diabetes mellitus and hypertension

c) An elderly male who is 2 days postcoronary artery bypass surgery AF can be seen in people without any apparent disease, or it may occur in people with coronary artery disease, mitral valve disease, ischemic heart disease, hypertension, myocardial infarction, pericarditis, congestive heart failure, digitalis toxicity, and hyperthyroidism. AF is the most common chronic arrhythmia, with an incidence and prevalence that increase with age. The incidence of AF increases with age. For example, it occurs in less than 0.5% of the population aged less than 50 years and increases by 2% at ages 60-69 years old. The prevalence is also greater in men than in women.

After birth, red blood cells are normally made in which of the following locations? a) Kidneys b) Liver c) Bone marrow d) Spleen

c) Bone marrow After birth, red cells are produced in the red bone marrow. Until a child is 5 years old, almost all bones produce red cells to meet the growth needs of a child, after which bone marrow activity gradually declines.

Polycythemia develops in clients with lung disease as a result of: a) Decreased blood viscosity b) Excessive respiratory fluid loss c) Chronic hypoxia d) Hyperventilation

c) Chronic hypoxia Secondary polycythemia results from a physiologic increase in the level of erythropoietin, commonly as a compensatory response to hypoxia. Conditions causing hypoxia include living at high altitudes, chronic heart and lung disease, and smoking.

A patient who experienced an ST elevation myocardial infarction (STEMI) received fibrinolytic therapy with streptokinase. Which of the following manifestations alerts the nurse to a developing complication? a) Diarrhea b) Symmetrical joint pain c) Decreased level of consciousness d) Hypoglycemia

c) Decreased level of consciousness Fibrinolytic therapy is most effective in treating STEMI when administered within 30 minutes after the onset of symptoms. It can still be beneficial up to 12 hours after the onset of ischemic pain. Patients who should not receive fibrinolytic therapy are those with a history of intracranial hemorrhage or significant trauma within the preceding 3 months. The primary complication of fibrinolytic treatment is intracranial bleeding that usually occurs within the first 24 hours following treatment. This would be evident with a change in mental status.

A nurse is caring for a newborn that has developed a low platelet count. Which of the following could be the cause? a) Increased pluripotent stem cell division b) Decreased spleen function c) Decreased thrombopoietin (TPO) d) Increased megakaryocytes

c) Decreased thrombopoietin (TPO) TPO stimulates the differentiation of platelets.

A newborn has a clotting disorder that results in the body being unable to produce fibrin. Which of the following could be the cause? a) Increase in the number of megakaryocytes b) Iron deficiency c) Deficiency of fibrinogen d) Decreased spleen function

c) Deficiency of fibrinogen Fibrinogen is a soluble protein that polymerizes to form the insoluble protein fibrin during blood clotting.

The nurse knows that persons with atrial fibrillation are at risk of developing which of the following? a) Hypovolemic shock b) Cardiogenic shock c) Diastolic dysfunction d) Systolic dysfunction

c) Diastolic dysfunction In atrial fibrillation, the lack of coordinated atrial contraction will decrease ventricular filling and thus decrease the volume of blood available for cardiac output. Systolic dysfunction, a decrease in myocardial contractility, is not affected by atrial fibrillation. Cardiogenic shock, a decrease in the pumping ability of the heart, and hypovolemic shock, a decrease in blood volume, also are not affected by atrial fibrillation.

The nurse knows that which group of antihypertensive drugs is usually the least expensive and is well tolerated? a) Beta adrenergic blockers b) Angiotensin converting enzyme inhibitors c) Diuretics d) Calcium channel blockers

c) Diuretics Diuretics are usually the least expensive, and are usually well tolerated by most patients.

A nurse is evaluating laboratory results of a patient diagnosed with a parasitic infection. The tests reveal a large group of cells that are membrane bound with granules in their cytoplasm that are aiding in the destruction of the parasite. Which of the following cells is the nurse evaluating? a) Monocyte b) Macrophage c) Eosinophil d) Lymphocyte

c) Eosinophil People who have helminthic parasites will have an increase in eosinophils in their complete blood count. The specific cytoplasmic granules of the eosinophils stain red with the acidic dye eosin. In parasitic infections, the eosinophils use surface markers to attach themselves to the parasite and then release hydrolytic enzymes that kill it.

A client with hemophilia Type A comes to the emergency department with severe pain and swelling in the right knee. The nurse anticipates the administration of which of the following to reduce musculoskeletal damage? a) Corticosteroids b) Heat to the knee c) Factor VIII replacement therapy d) Nonsteroidal anti-inflammatory drug

c) Factor VIII replacement therapy Factor VIII replacement therapy administered at home has reduced the typical musculoskeletal damage. It is initiated when bleeding occurs or as prophylaxis with repeated bleeding episodes for clients with Hemophilia A

A patient being treated for polymyalgia rheumatica (PMR) comes to the clinic for a follow-up visit. The current erythrocyte sedimentation rate (ESR) rate has decreased from 60 mm/hour to 30 mm/hour and the patient is afebrile, denies any headaches, and reports a decrease in joint pain and fatigue. Based on these findings, the nurse anticipates which of the following changes to the treatment plan? a) No change of treatment; continue to monitor the patient with a follow-up visit in 1 month. b) The patient is cured, and no further treatment is necessary. c) Gradually decrease the current dosage of prednisone and repeat ESR in 2 weeks. d) Discontinue the current prednisone prescription and start an anti-inflammatory drug such as naproxen. Repeat ESR in 2weeks.

c) Gradually decrease the current dosage of prednisone and repeat ESR in 2 weeks. The ESR is a screening test for monitoring the fluctuations in the clinical course of a disease such as PMR. Symptoms of this inflammatory disorder include fatigue, fever, pain, and headache, which are all a result of the systemic effects of cytokine release. An individual who is diagnosed with PMR will have an initial ESR drawn. If the ESR is >60, the person is diagnosed with a severe PMR. Generally, the person is prescribed prednisone, and then at follow-up visits, the dose of prednisone will be gradually decreased depending on the ESR and the person's symptoms.

The client is an average-sized adult and has abnormal microcytic hypochromic red blood cells due to a long-term, chronic disease. Which of the following complete blood count (CBC) results is characteristic of her type of anemia? a) Reticulocytes 1.5% b) Hematocrit 44% c) Hemoglobin 8 g/dL d) Band cells 3000/mL

c) Hemoglobin 8 g/dL Anemia of chronic disease is characterized by a low hemoglobin level, low hematocrit, and low reticulocyte count. The quantity of band cells, immature neutrophils released from the marrow, is unrelated to the anemia.

When an Rh-negative mother gives birth to an Rh-positive infant, the mother usually produces antibodies that will attack any subsequent pregnancies in which the fetus is Rh positive. When subsequent babies are Rh positive, erythroblastosis fetalis occurs. What is another name for erythroblastosis fetalis? a) Microcytic disease of the newborn b) Macrocytic disease of the newborn c) Hemolytic disease of the newborn d) Hemolytic iron deficiency anemia

c) Hemolytic disease of the newborn Erythroblastosis fetalis, or hemolytic disease of the newborn, occurs in Rh-positive infants of Rh-negative mothers who have been sensitized. The Rh-negative mother usually becomes sensitized during the first few days after delivery, when fetal Rh-positive red cells from the placental site are released into the maternal circulation. Because the antibodies take several weeks to develop, the first Rh-positive infant of an Rh-negative mother usually is not affected. There is no such thing as microcytic or macrocytic disease of the newborn, nor is there a hemolytic iron deficiency anemia

The emergency department nurse is concerned that her patient's snake bite may trigger disseminated intravascular coagulation (DIC). If this should occur, which of the following clinical manifestations would be seen? a) Hypertension b) Headaches c) Hemorrhage d) Platelet loss

c) Hemorrhage DIC is hemorrhage in the presence of excessive coagulation. Late-stage DIC is characterized by uncontrolled hemorrhaging. Platelet deficit is not present; blood loss causes hypotension and rapid loss of consciousness.

Which of the following enzymes has a powerful vasodilator effect on arterioles and increases capillary permeability? a) Arachidonic acid b) Serotonin c) Histamine d) Prostaglandins

c) Histamine Histamine has a powerful vasodilator effect on arterioles and has the ability to increase capillary permeability, allowing leakage of both fluid and plasma proteins into the tissues. Serotonin causes vasoconstriction and plays a major role in control of bleeding. Prostaglandins produce either vasoconstriction or vasodilation.

The health care provider is discussing major risk factors for coronary artery disease (CAD) with a client. The most important information for the provider to include would be: a) High serum high-density lipoprotein and diabetes b) Physical inactivity and high serum high-density lipoprotein cholesterol c) History of cigarette smoking and elevated blood pressure d) Advanced age and low serum total and low-density lipoprotein cholesterol

c) History of cigarette smoking and elevated blood pressure The major risk factors for CAD include cigarette smoking, elevated blood pressure, elevated LDL cholesterol, low HDL cholesterol, diabetes, advancing age, abdominal obesity, and physical inactivity.

Increased cardiac workload with left heart failure can result in which of the following change to the myocardial cells? a) Dysplasia b) Hyperplasia c) Hypertrophy d) Atrophy

c) Hypertrophy Myocardial hypertrophy is a compensatory mechanism in heart failure as the heart attempts to maintain adequate pumping ability. Paradoxically, hypertrophy can gradually decrease cardiac efficiency.

A client has been admitted for immune thrombocytopenic purpura. The client has not responded to corticosteroid treatment. The priority nursing intervention for this client would include which of the following treatment measures? a) Place the client in isolation, so the skin rashes will not spread to other clients. b) Insert a Foley catheter to monitor hourly urine output. c) Insert an intravenous catheter, so immune globulin can be administered in a timely manner. d) Prepare a surgical permit for an emergency splenectomy.

c) Insert an intravenous catheter, so immune globulin can be administered in a timely manner. The decision to treat ITP is based on the platelet count and the degree of bleeding. Corticosteroids are used as initial therapy; other effective initial treatment includes intravenous immune globulin. However, this treatment is expensive, and the beneficial effect may last only 1 to 2 weeks. Because the spleen is the major site of antibody formation and platelet destruction, splenectomy is the traditional second-line treatment for persons who relapse or do not respond to medications. Isolation is not required; it is not contagious. The client may require a Foley if he does not respond to the immune globulin and must undergo surgery.

A client who has suffered a myocardial infarction is being treated in the emergency room. His pain remains severe even though he was given nitrates and oxygen. The physician now orders morphine for the pain. What method should the nurse to administer the morphine? a) By mouth in pill form b) By mouth in a liquid c) Intravenous d) Subcutaneous

c) Intravenous Morphine is given intravenously when a patient is in the emergency room suffering a myocardial infarction. It is given intravenously because of the rapid onset of action, and it does not elevate enzyme levels.

The nurse is educating a pregnant client about the importance of folic acid. Select the food that has the highest levels of folic acid. a) Apples b) Carrots c) Kale d) Eggs

c) Kale Folic acid is readily absorbed from the intestine. It is found in vegetables (particularly the green leafy types), fruits, cereals, and meats. Much of the vitamin, however, is lost in cooking. The most common causes of folic acid deficiency are malnutrition or dietary lack, especially in the elderly or in association with alcoholism.

The liver has many jobs. One of the most important functions of the liver is to cleanse the portal blood of old and defective blood cells, bacteria in the bloodstream, and any foreign material. Which cells in the liver are capable of removing bacteria and foreign material from the portal blood? a) Epstein cells b) Langerhans cells c) Kupffer cells d) Davidoff cells

c) Kupffer cells Kupffer cells are reticuloendothelial cells that are capable of removing and phagocytizing old and defective blood cells, bacteria, and other foreign material from the portal blood as it flows through the sinusoid. Langerhans cells are stellate dendritic cells found mostly in the stratum spinosum of the epidermis. Epstein cells do not exist. Davidoff cells are large granular epithelial cells found in intestinal glands.

Anaphylactic shock is the most severe form of systemic allergic reaction. Immunologically medicated substances are released into the blood, causing vasodilation and an increase in capillary permeability. What physiologic response often accompanies the vascular response in anaphylaxis? a) Gastrointestinal relaxation b) Uterine smooth muscle relaxation c) Laryngeal edema d) Bronchodilation

c) Laryngeal edema Anaphylaxis is a clinical syndrome that represents the most severe form of systemic allergic reaction. Anaphylactic shock results from an immunologically mediated reaction in which vasodilator substances such as histamine are released into the blood. The vascular response in anaphylaxis is often accompanied by life-threatening laryngeal edema and bronchospasm, circulatory collapse, contraction of gastrointestinal and uterine smooth muscle, and urticaria (hives) or angioedema.

A client comes to the emergency room exhibiting signs and symptoms of right-sided heart failure. Upon X-ray it is determined that he has 250ML of fluid in the pericardial cavity. Which disease should the nurse suspect this client to be suffering? a) Myocardial infarction b) COPD c) Pericardial effusion d) Pericarditis

c) Pericardial effusion Pericardial effusion refers to the accumulation of fluid in the pericardial vacuity, usually as a result of an inflammatory or infectious process. A sudden accumulation of even 200ml of fluid may raise intracardiac pressure to levels that will cause symptoms similar to right-sided heart failure. Pericarditis is inflammation of the pericardium while COPD is a respiratory disease.

On a holiday trip home, the nurse's mother states that the nurse's father was diagnosed with right heart failure. Which of the following manifestations exhibited by the father does the nurse know might have preceded this diagnosis? a) Vertigo, headache b) Dyspnea, cough c) Peripheral edema weight gain d) Weakness, palpitations

c) Peripheral edema weight gain In right failure. blood backs up into the venous side of the circulatory system causing increased hydrostatic pressure in capillaries and leakage of plasma which forms peripheral edema and becomes apparent as weight gain. The other manifestations listed are not characteristic of right-sided failure.

A patient presents to the clinic with symptoms of elevated blood pressure, dizziness, red face, pain in fingers and toes, headache, and difficulty concentrating. A blood smear reveals an increased number of erythrocytes. Based on these findings, the nurse anticipates which of the following diagnoses? a) Hemolytic anemia b) Leukemia c) Polycythemia vera d) Hyperbilirubinemia

c) Polycythemia vera Polycythemia vera is a neoplastic disease of the pluripotent cells of the bone marrow characterized by an absolute increase in total red blood cell mass accompanied by elevated white cell and platelet counts. In polycythemia vera, the clinical manifestations are hypertension, headache, dizziness, inability to concentrate, and some difficulty with hearing and vision because of decreased cerebral blood flow. Venous stasis gives rise to a plethoric appearance or dusky redness, even cyanosis, particularly of the lips, fingernails, and mucous membranes. Because of the increased concentration of blood cells, the person may experience itching and pain in the fingers or toes, and the hypermetabolism may induce night sweats and weight loss.

The nurse is preparing a client with suspected leukemia for a bone marrow and biopsy. What preferred site will the nurse be sure is accessible for the physician? a) Cervical spinous process b) Anterior femur c) Posterior iliac crest d) Clavicle

c) Posterior iliac crest Usually, the posterior iliac crest is used in all people older than 12 to 18 months of age. Other sites include the anterior iliac crest, sternum, and spinous processes T10 through L4. The sternum is not commonly used in children because the cavity is too shallow and there is danger of mediastinal and cardiac perforation.

A patient's blood pressure is persistently in the range of 130-135 mm Hg systolic and 85-88 mm Hg diastolic. The nurse knows that which of the following conditions correctly describes this patient's blood pressure? a) Normal blood pressure b) Stage 2 Hypertension c) Prehypertension d) Stage 1 Hypertension

c) Prehypertension Normal blood pressure is considered to be systolic below 120 mm Hg or diastolic below 80 mm Hg. Prehypertension is systolic 120-139 mm Hg and diastolic 80-89 mm Hg. This condition should be checked at least once yearly. Stage 1 hypertension is 140-159 mm Hg systolic or 90-99 mm Hg diastolic and should be confirmed within two months. Stage 2 is systolic equal to or greater than 160 mm Hg or diastolic equal to or greater than 100 mm Hg. Antihypertensive medication should be considered for Stage 1 and 2; lifestyle changes should be recommended as appropriate for any blood pressure higher than normal.

A 35-year-old client arrives for an appointment with his primary care physician because he has been experiencing "this funny fluttering in my chest" and is concerned that there is something seriously wrong. The physician evaluates the client and discovers that the client has many stressors along with suspected alcohol and tobacco abuse. The physician notes on the cardiac monitor that the client has a heart rate of 90 with a slightly irregular rhythm with P waves before the QRS although not all the P waves are the same shape. Which of the following rhythms fits this description? a) Atrial tachycardia b) Premature ventricular contractions c) Premature atrial contractions d) Atrial fibrillation

c) Premature atrial contractions The P waves and heart rate are the clues to the answer for this question. Premature atrial contractions (PACs) originate in the atrium before the expected impulse from the SA node. The PACs travel the normal conduction pathway to the ventricles and then back to the SA node causing an interruption of the next SA node impulse. The interruption of the normal SA node impulse causes a pause in the rhythm and also gives the ventricles more filling time for the next normal impulse, which may be felt due to the increased volume. The shape of the P waves is determined by the location in atria that the ectopic impulse originates; however, they will be different than the SA nodal P waves. The rate of 70 rules out a tachycardia and the presence of P waves before the QRS complexes would rule out both atrial fibrillation and premature ventricular contractions. Alcohol and tobacco abuse along with stress are contributing factors to the development of premature atrial contractions in healthy individuals.

A teenager is seen in the emergency room with complaints of a sore throat, headache, fever, abdominal pain, and swollen glands. His mother tells the nurse that he was seen three weeks before in the clinic and treated with antibiotics for a strep throat. He was better for a few days but now he seems to have gotten worse in the last two days. What should the nurse suspect is wrong with this client? a) Meningitis b) Flu c) Rheumatic fever d) Mononucleosis

c) Rheumatic fever Rheumatic fever is an immune mediated inflammatory disease that occurs a few weeks after a group A strep (sore throat). It can manifest as an acute, recurrent or chronic disorder.

A nurse witnesses an elderly woman sitting outside who suddenly faints and is helped to the ground by family. She is awake, although confused, heart rate 130, RR 28. The woman states that all of a sudden she felt dizzy, but denies other symptoms. She says that her husband died last week and she has been very upset. The nurse determines that which of the following is the most likely reason for the syncopal episode? a) Blood loss b) Severe pain c) Severe anxiety d) Congestive heart failure

c) Severe anxiety This person is most likely suffering from severe anxiety following her husband's death, which would be a normal physiologic explanation for the tachyarrhythmia that may have contributed to the syncope. Although the other options are all potential causes of tachyarrhythmias, there is nothing in the question that supports that they would be the cause.

A patient is seen in the emergency department complaining of chest discomfort, productive cough, and a fever of over 101°F for 3 days. The nurse performs an electrocardiogram and observes a rate of 110 beats per minute (bpm) with a normal P wave and a PR interval of 0.12 sec preceding each QRS complex. Which of the following does the nurse determine the rhythm to be? a) Sick sinus syndrome b) Atrial flutter c) Sinus tachycardia d) Third-degree heart block

c) Sinus tachycardia Sinus tachycardia is a heart rate >100 bpm that has its origin in the sinoatrial node. A normal P wave and PR interval should precede each QRS complex. The mechanism of sinus tachycardia is enhanced automaticity, related to sympathetic stimulation or withdrawal of vagal tone. Sinus tachycardia is a normal response during any increase in metabolic activity such as fever, stress, anxiety, and the like.

The most recent blood work of a client with a diagnosis of acute myelogenous leukemia (AML) reveals thrombocytopenia. Where is the client most likely to experience abnormal bleeding as a result of low platelets? a) Inside the brain b) Sclerae of the eyes c) Skin and mucous membranes d) Nephrons and ureters

c) Skin and mucous membranes Common sites for spontaneous bleeding from platelet disorders are the skin and mucous membranes of the nose, mouth, gastrointestinal tract, and uterine cavity. Intracranial hemorrhage is rare, and neither the sclerae nor the kidneys are typical sites of spontaneous bleeding.

The nurse teaches the client that which of these contributed to his developing acute cholelithiasis? a) Chronic pancreatitis b) Excessive alcohol intake c) Stasis of bile d) Rapid elimination of bile

c) Stasis of bile Three factors contribute to the formation of gallstones: abnormalities in the composition of bile, stasis of bile (rather than rapid elimination), and inflammation of the gallbladder. Inflammation of the gallbladder alters the absorptive characteristics of the mucosal layer, allowing excessive absorption of water and bile salts. Although a number of factors are associated with the development of acute pancreatitis, most cases result from gallstones (rather than cause gallstone formation) or alcohol abuse. Alcohol is known to be a potent stimulator of pancreatic secretions, and it also is known to cause partial obstruction of the sphincter of the pancreatic duct; alcohol intake is not a factor in the development of cholesterol or bilirubin stones.

A 20-year-old college student, with no past medical history, arrives at the emergency room complaining of severe palpitations and dizziness that started this morning following a night of studying. The student is very upset that this is happening because the final exams are the following day. The cardiac monitor shows a heart rate of 110, regular rhythm with occasional premature ventricular complexes. The nurse explains to the student that this can happen in healthy hearts and is usually caused by stimulation of which of the following? a) Parasympathetic nervous system b) Vagal nerve c) Sympathetic nervous system d) Atrial ectopic foci

c) Sympathetic nervous system premature ventricular complexes can occur in healthy hearts in response to stimulation of the sympathetic nervous system. This client states nighttime studying (possibly with coffee intake) and stress over upcoming exams, both of which can stimulate the sympathetic nervous system.

The nurse is assessing the ECG of a client who is experiencing unstable angina. The nurse observes: a) Peaked T waves b) Deep Q waves c) T-wave changes d) Significant ST-segment elevation

c) T-wave changes Unstable angina/non-ST elevation myocardial infarction (UA/NSTEMI) is a clinical syndrome of myocardial ischemia ranging from angina to myocardial infarction. The ECG pattern associated with in NSTEMI may display normal or ST segment depression (or transient ST segment elevation) and T wave changes. The degree of ST segment deviation from baseline is an important measure of ischemia and indicator of prognosis. Abnormal Q waves occurs with ACS.

In second-degree atrioventricular block, there is a relationship between the P waves and the QRS complex resulting in recurring PR intervals. What does this mean? a) The relationship between the P waves and the QRS complexes is a narrowing PR interval. b) The association of P waves and QRS complexes is random. c) The association of P waves and QRS complexes is not random. d) The relationship between the P waves and the QRS complexes is a widening PR interval.

c) The association of P waves and QRS complexes is not random. A distinguishing feature of second-degree atrioventricular block is that conducted P waves relate to QRS complexes with recurring PR intervals; that is, the association of P waves with QRS complexes is not random. The other answers are not correct.

Which information obtained by the nurse caring for a client with antiphospholipid syndrome should be immediately communicated to the health care provider? a) The client has bruises on the arms and legs. b) The client's platelet count is 1,000,000/µL c) The client is difficult to arouse. d) The client has a family history of Leiden mutation.

c) The client is difficult to arouse. Antiphospholipid syndrome common features are venous and arterial thrombi. People with the disorder present with a variety of clinical manifestations, typically those characterized by recurrent venous and arterial thrombi. Venous thrombosis, especially in the deep leg veins, occurs in up to 50% of people with the syndrome, half of whom develop pulmonary emboli. Arterial thrombosis involves the brain in up to 50% of cases, causing transient ischemic attacks or strokes. Difficulty in arousing the patient may indicate a cerebral hemorrhage.

Which client will the nurse prioritize to assess first? a) The client with sinus bradycardia b) The client with a sinus arrhythmia c) The client with sinus arrest d) The client with tachycardia

c) The client with sinus arrest The client with sinus arrest refers to failure of the SA node to discharge and results in an irregular pulse. An escape rhythm develops as other pacemakers take over, but it may result in prolonged periods of asystole and other abnormal rhythms. The client may need further interventions such as a pacemaker. The other clients need monitoring; however, their rhythm is not placing them in need of immediate assistance.

The nurse is caring for a 2-day-old newborn infant who appears lethargic and has a yellowish tint to the skin. Select the most likely cause of this newborn's signs and symptoms. a) A decrease in the breakdown of red blood cells b) The inability of a newborn to produce bilirubin c) The inability of the immature liver to conjugate bilirubin d) Cirrhosis of the liver

c) The inability of the immature liver to conjugate bilirubin Jaundice is a sign of increased bilirubin related to the increased red blood cell breakdown and the inability of the immature liver to conjugate bilirubin. Jaundice occurs from a large production of bilirubin, and a newborn would not have cirrhosis of the liver.

The nurse knows that the primary long term regulation of blood pressure is exerted by which of the following? a) Hormonal activity b) Humoral influence c) The kidneys d) Neural mechanisms

c) The kidneys The kidneys exert long-term control of blood pressure by modulating sodium content and extracellular fluid volume. An increase in extracellular fluid will yield an increase in blood volume and an increase in cardiac output. If this results in too great an increase in blood pressure, the kidneys will excrete sodium and water. Neural (autonomic nervous system) and humoral mechanisms (renin-angiotensin-aldosterone system and vasopressin) exert short-term control of blood pressure. Hormonal influence involves release of epinephrine (which works through the sympathetic nervous system) from the adrenal glands.

All cells of the body age and are replaced in a natural order. When RBCs age, they are destroyed in the spleen. During this process, the iron from their hemoglobin is released into the circulation and returned where? a) To the muscles to be stored for strength. b) To the lungs to bind with oxygen. c) To the bone marrow for incorporation into new RBCs. d) To the liver to bind with oxygen.

c) To the bone marrow for incorporation into new RBCs. When RBCs age and are destroyed in the spleen, the iron from their hemoglobin is released into the circulation and returned to the bone marrow for incorporation into new RBCs or to the liver and other tissues for storage. Iron is not bound to RBCs in the liver. Iron does not bind with oxygen in the lung without first being incorporated into an RBC. Iron is stored in tissues of the body, but not for strength, only for its oxygen-binding capacity.

Which type of pacing involves the placement of large patch electrodes on the anterior and posterior chest wall that can be connected by a cable to an external pulse generator? a) Epicardial b) Transvenous c) Transcutaneous d) Permanent

c) Transcutaneous The type of pacing described is transcutaneous because it is only form that is accessible externally.

The heart consists of four valves. Which are the heart's atrioventricular valves? Select all that apply. a) Aortic b) Pulmonary c) Tricuspid d) Mitral

c) Tricuspid d) Mitral The heart's atrioventricular valves are the tricupsid and the mitral. The semilunar valves are the pulmonary and aortic.

Which serum biomarker is highly specific for myocardial tissue? a) C-reactive protein b) White blood cells c) Troponin d) Creatine kinase

c) Troponin The troponin assays have high specificity for myocardial tissue and have become the primary biomarker tests for the diagnosis of myocardial infarction. Creatine kinase is specific for muscle injury but is not as focused as is troponin. White blood cells and C-reactive protein are associated with inflammation.

The nurse identifies the blood vessel layer that constricts to regulate and control diameter as which of the following? a) Tunica intima b) Tunica externa c) Tunica media d) Tunica adventitia

c) Tunica media The tunica media, the middle layer, is largely a smooth muscle layer that constricts to regulate and control the diameter of the vessel. The tunica adventitia and tunica externa refer to the outermost layer of a vessel. The tunica intima refers to the innermost layer that prevents platelet adherence and blood clotting.

Preload represents the volume work of the heart and is largely determined by: a) Ventricular emptying b) Vascular resistance c) Venous blood return d) Force of contraction

c) Venous blood return Preload represents the amount of blood the heart must pump with each beat and represents the volume of blood stretching the ventricular muscle fibers at the end of diastole. Pressure (resistance), contraction, and ventricular emptying relate to afterload.

Which of the following arrhythmias is considered to be the most fatal and requires immediate treatment? a) Premature ventricular contractions b) Premature atrial contractions c) Ventricular fibrillation d) Atrial flutter

c) Ventricular fibrillation Ventricular fibrillation represents severe derangements of cardiac rhythms that terminate fatally within minutes unless corrective measures are taken promptly. All of the other arrhythmias need to have further investigation into etiology, but are not immediately fatal.

A patient is diagnosed with right-sided heart failure. The nurse knows that a frequent sign of this type of failure is peripheral edema, evidenced by which of the following? a) Decreased blood pressure b) Shortness of breath c) Weight gain d) Copious urination

c) Weight gain In right-sided heart failure, blood backs up into the venous system and increased capillary hydrostatic pressure forces plasma out of the circulatory system, resulting in edema. The accumulation of fluid is evidenced by rapid weight gain. Shortness of breath and decreased renal perfusion and output are characteristic of left-sided failure.

To form a platelet plug, platelets are attracted to the damaged vessel, and then platelet ___________ occurs. a) thromboxane A2 b) fibrinolysis c) adhesion d) thrombosis

c) adhesion There are two processes involved in platelet plug formation, the first of which is platelet adhesion to the vessel subendothelial layer. Fibrinolysis is the process of clot dissolution. Thrombosis is inappropriate formation of clots within the vascular system. Thromboxane A2 (TXA2) is released during platelet aggregation, the step after adhesion.

A 10-month-old infant has begun to take his first steps and his mother has brought him for assessment because of swelling in his ankles and knees. The mother also states that he was eager to walk but has now regressed and cries when she tries to encourage it. The clinician should: a) administer vitamin K STAT. b) prescribe a trial of desmopressin acetate (DDAVP) to rule out von Willebrand disease. c) assess the child for signs and symptoms of hemophilia A. d) order testing to rule out disseminated intravascular coagulation (DIC).

c) assess the child for signs and symptoms of hemophilia A. The child's presentation is characteristic of hemophilia A, not DIC or von Willebrand disease. Vitamin K does not affect the physiology of hemophilia.

An elderly client has been diagnosed with chronic heart failure. He is prescribed an ACE inhibitor to treat the symptoms and improve his quality of life. This drug will alleviate the client's symptoms of heart failure by: a) promoting cardiac output through a reduction in afterload. b) selectively blocking the synthesis of renin in the kidneys. c) blocking the conversion of angiotensin I to angiotensin II. d) enhancing inotropy by maximizing calcium channel function.

c) blocking the conversion of angiotensin I to angiotensin II. ACE inhibitors block the conversion of angiotensin I to angiotensin II. They do not directly affect renin synthesis, calcium channel function, or afterload.

A nurse on an oncology floor is treating a client who is anemic following chemotherapy. The drugs being administered are designed to increase the number of red blood cells in circulation. These drugs are likely agonists of: a) lymphokines. b) trophic hormones. c) cytokines. d) secondary hormones.

c) cytokines. RBC production is directed by cytokines and growth factors. Lymphokines are used for communication between lymphocytes, and trophic and secondary hormones are too broad an answer to this question.

A 23-year-old female client has been diagnosed with von Willebrand disease following a long history of "heavy periods" and occasional nosebleeds. Which of the client's following statements demonstrates a sound understanding of her new diagnosis? a) "I read on a website that I might have to get blood transfusions from time to time." b) "I hope my insurance covers the injections that I'll need to help my blood clot." c) "I'm really disappointed that I won't be able to do sports anymore." d) "I'll make sure to take Tylenol instead of aspirin when I get aches and pains."

d) "I'll make sure to take Tylenol instead of aspirin when I get aches and pains." Most clients with von Willebrand disease, especially less serious variants, require only monitoring and aspirin avoidance. It is likely unnecessary for the client to avoid sports. Drugs, clotting factors, and transfusions are likely not required, given the moderate bleeding that the client has experienced to this point.

A nurse is performing client health education with a 68-year-old man who has recently been diagnosed with heart failure. Which of the following statements demonstrates an accurate understanding of his new diagnosis? a) "I know it's healthy to drink a lot of water, and I'm going to make sure I do this from now on." b) "I'm going to avoid as much physical activity as I can so that I preserve my strength." c) "I'll be sure to take my beta blocker whenever I feel short of breath." d) "I'm trying to think of ways that I can cut down the amount of salt that I usually eat."

d) "I'm trying to think of ways that I can cut down the amount of salt that I usually eat." Salt and fluid restrictions are indicated for most clients with heart failure (HF). Beta blockers do not address shortness of breath, and cardiac medications are not normally taken in response to acute symptoms. Clients should be encouraged to maintain, and increase, physical activity within the limits of their condition.

A client who had a pulmonary embolism is receiving IV heparin and has just begun taking his first dose of warfarin (Coumadin). The client asks the nurse, "How long will this pill take in order to prevent me from developing more clots. I would like to go home soon." The nurse responds: a) "That's a good question. If warfarin has a long half-life, it could take many days. I will have to ask the pharmacist this question." b) "I don't really know. Right now you are on two blood thinners, so we won't be able to tell if it's working until the doctor discontinues your heparin drip." c) "First, we have to have your health care provider switch you from IV heparin to a low molecular weight heparin, which you need to inject in your belly." d) "It usually takes 2 to 3 days for warfarin to become therapeutic, meaning your blood will be thin enough to prevent further clot formation."

d) "It usually takes 2 to 3 days for warfarin to become therapeutic, meaning your blood will be thin enough to prevent further clot formation." The anticoagulant drugs warfarin and heparin are used to prevent thromboembolic disorders, such as deep vein thrombosis and pulmonary embolism. Warfarin acts by decreasing prothrombin. It alters vitamin K in a manner that reduces its ability to participate in the synthesis of the vitamin K-dependent coagulation factors in the liver. Warfarin's maximum effect takes 36 to 72 hours because of the varying half-lives of different clotting factors that remain in the circulation. Heparin ultimately suppresses the formation of fibrin and therefore inhibits coagulation. The low molecular weight heparins are given by subcutaneous injection and require less frequent administration and monitoring compared with the standard (unfractionated) heparin. Clients usually are not sent home with an IV medication. Once warfarin is in therapeutic range, heparin or low molecular weight heparins are no longer required.

A client refuses to take the 81 mg of aspirin ordered by the physician, stating, "I do not have any pain." The best response by the nurse would be: a) "The doctor wants you to take the medication to prevent you from experiencing pain." b) "Low-dose aspirin will help prevent you from having increased bleeding after surgery." c) "This dose of aspirin will break apart the blood clot that you have in your leg." d) "The 81 mg of aspirin daily will help protect you from a stroke or a heart attack."

d) "The 81 mg of aspirin daily will help protect you from a stroke or a heart attack." A low dose of aspirin (81 mg) can be used to prevent platelet aggregation and clot formation in persons who are at risk for myocardial infarction, stroke, or peripheral artery disease. This dose of aspirin will not be therapeutic for preventing pain in most clients and will not prevent increased bleeding. Aspirin will not dissolve a blood clot.

The nurse is caring for a client who is a carrier of hepatitis B. Which of these does the nurse teach the family? a) "Something in your home causes him to 'carry' the virus to his body." b) "The tests your loved one took show he has had hepatitis B in the past." c) "A carrier state means the individual is at risk for sudden death." d) "Your loved one may not look ill, but the virus is present in his blood."

d) "Your loved one may not look ill, but the virus is present in his blood." Infection with HBV and HCV can produce a carrier state in which the person does not have symptoms but harbors the virus and can therefore transmit the disease.

A client has suffered chest injuries following a automobile accident. The chest injury has resulted in an increase in intrathoracic pressure. There can be a transient shift from the pulmonary to the systemic circulation of how much blood? a) 200 mL b) 300 mL c) 150 mL d) 250 mL

d) 250 mL An increase in intrathoracic pressure, which impedes venous return to the right heart, can produce a transient shift from the pulmonary to the systemic circulation of as much as 250 mL of blood.

Four clients were admitted to the emergency department with severe chest pain. All were given preliminary treatment with aspirin, morphine, oxygen, and nitrates and were monitored by ECG. Which client most likely experienced myocardial infarction? a) 33-year-old man whose pain started at 7 AM during moderate exercise and was relieved by nitrates; ECG was normal; cardiac markers remained stable b) 67-year-old woman whose pain started at 2 AM while she was asleep and that responded to nitrates; the ECG showed arrhythmias and ST-segment elevation; cardiac markers remained stable c) 61-year-old man whose pain started at 9 AM during a short walk and responded to nitrates, but not to rest; ECG and cardiac markers remained stable, but anginal pattern worsened d) 80-year-old woman whose pain was not relieved by nitrates or rest; the ECG showed ST-segment elevation with inverted T waves and abnormal Q waves; levels of cardiac markers subsequently rose

d) 80-year-old woman whose pain was not relieved by nitrates or rest; the ECG showed ST-segment elevation with inverted T waves and abnormal Q waves; levels of cardiac markers subsequently rose The chest pain of myocardial infarction does not respond to rest or to nitrates. Ischemic injury to the myocardium alters the ECG patterns, often elevating the ST segment and inverting T waves. Abnormal Q waves indicates necrosis. Cardiac markers are released in response to myocardial injury; rising levels indicate damage to the heart.. The other clients have angina of varying severity.

What is considered the normal amount of serum bilirubin found in the blood? a) >1.8mg/dL b) 2mg/dL c) 1.7mg/dL d) <1.5mg/dL

d) <1.5mg/dL Usually, only a small amount of bilirubin is found in the blood; the normal level of total serum bilirubin is

In which situation is blood most likely to be rapidly relocated from central circulation to the lower extremities? a) A client reclines from a sitting to supine position b) A client undergoes a stress test on a treadmill c) A client does isotonic exercises in a wheelchair d) A client is helped out of bed and stands up

d) A client is helped out of bed and stands up During a change in body position, blood is rapidly relocated from the central circulation (when the client is recumbent) to the lower extremities (when the client stands up). This results in a temporary drop in blood pressure known as postural hypotension and reflects the redistribution of blood in the body.

A client is admitted to the cardiac unit with a diagnosis of pericarditis. The nurse is teaching the patient about the anatomical location of the infection. The nurse evaluates the effectiveness of the teaching when the client correctly identifies which of the following as the location of the pericardium? a) The electrical conduction system of the heart b) The outer muscular layer of the heart c) The innermost lining of the heart chambers d) A membranous sac that encloses the heart

d) A membranous sac that encloses the heart The pericardium forms a fibrous covering around the heart, holding it in a fixed position in the thorax and providing physical protection and a barrier to infection. The pericardium is a tri-layer sac consisting of a tough, outer fibrous layer and a thin, inner serous layer.

The nurse observes a client care technician obtain a blood pressure on an obese client using a BP cuff that is too small. The nurse is aware that this will result in: a) A reading that is too low b) An accurate BP assessment c) A decreased pulse pressure reading d) A reading that is too high

d) A reading that is too high The width of the bladder should be at least 40% of arm circumference and the length at least 80% of arm circumference. Undercuffing (using a cuff with a bladder that is too small) can cause an overestimation of blood pressure. This is because a cuff that is too small results in an uneven distribution of pressure across the arm, such that a greater cuff pressure is needed to occlude blood flow. Likewise, overcuffing (using a cuff with a bladder that is too large) can cause an underestimation of blood pressure.

The nurse is teaching a group of nursing students about alcohol-induced liver disease and the cytochrome P450 system (CYP P450). Which of these is correct for the nurse to include in the lesson? a) Alcohol is a source of calories, but cannot be stored as protein, fat or carbohydrates. b) This system prevents rapid metabolism of ethanol and subsequent intoxication. c) Impairment of the CYP P450 system may cause severe hypoglycemia. d) Alcohol consumption enhances susceptibility to effects of drugs and toxins.

d) Alcohol consumption enhances susceptibility to effects of drugs and toxins. Increased activity of the CYPp P450 system enhances the susceptibility of persons with heavy alcohol consumption to the hepatotoxic effects of other substances.

A client has been diagnosed with atrial flutter. What assessment finding does the nurse expect? a) An increased blood pressure b) A slow atrial rate c) A ventricular heart rate above 240 beats per minute d) An atrial heart rate above 240 beats per minute

d) An atrial heart rate above 240 beats per minute Atrial flutter is a rapid atrial ectopic tachycardia and has an atrial rate from 240 to 450 beats per minute. The ventricular rate would not be expected to be as high as 240 beats per minute, and the blood pressure would not be expected to increase.

A 20-year-old male client is experiencing a severe immunologically mediated reaction in which histamines have been released into the blood. Select the type of reaction most likely occurring with this client. a) Septic shock b) Distributive shock c) Neurogenic shock d) Anaphylatic shock

d) Anaphylatic shock Anaphylactic shock is initiated through an immunologically mediated reaction to an irritant. Neurogenic shock is caused by decreased sympathetic control of blood vessel tone due to a defect in the vasomotor center in the brain stem or the sympathetic outflow to the blood vessels. Distributive or vasodilatory shock is characterized by loss of blood vessel tone, enlargement of the peripheral vascular compartment, and displacement of the vascular volume away from the heart and central circulation. Septic shock, which is the most common type of distributive shock, is associated with the systemic immune response to severe infection.

A child's history of a recurrent sore throat followed by severe knee and ankle pain has resulted in a diagnostic workup and a diagnosis of rheumatic fever. What are the treatment priorities for this child? a) Pain control and oxygen therapy b) Cardiac catheterization and corticosteroid therapy c) Implanted pacemaker and β-adrenergic blockers d) Antibiotics and anti-inflammatory drugs

d) Antibiotics and anti-inflammatory drugs A diagnosis of rheumatic fever (RF) necessitates the use of antibiotics (usually penicillin) and antiinflammatory drugs. These measures supersede the importance of pain control and oxygen therapy. Cardiac catheterization, corticosteroid therapy, pacemakers, and B-adrenergic blockers are not common treatment modalities for RF.

The globulins that make up part of the plasma of the blood have three distinct purposes. What are the gamma globulins? a) Transporters of bilirubin and steroids b) Autoantibodies of the immune system c) Transporters of iron and copper d) Antibodies of the immune system

d) Antibodies of the immune system There are three types of globulins: the alpha globulins that transport bilirubin and steroids, the beta globulins that transport iron and copper, and the gamma globulins that constitute the antibodies of the immune system. Alpha globulins transport bilirubin and steroids. Beta globulins transport iron and copper. Autoantibodies are immunoglobulins that recognize an antigen on that person's own tissue.

The cloning of the genes for most of the hematopoietic growth factors has been accomplished. The recombinant proteins that are produced are used in a wide range of clinical problems. What diseases have these proteins been used to fight? a) Aplastic anemia and the anemia of Huntington disease b) Anemia of cancer and Parkinson disease c) AIDS and autoimmune disorders d) Aplastic anemia and the anemia of kidney failure

d) Aplastic anemia and the anemia of kidney failure The genes for most hematopoietic growth factors have been cloned, and their recombinant proteins have been generated for use in a wide range of clinical problems. They are used to treat bone marrow failure caused by chemotherapy or aplastic anemia, the anemia of kidney failure and cancer, hematopoietic neoplasms, infectious diseases such as AIDS, and congenital and myeloproliferative disorders. Autoimmune disorders, Parkinson disease, and Huntington disease are not anemic disorders, so the recombinant proteins have not been used in the treatment of these diseases.

Coronary artery bypass grafting (CABG) is a treatment modality for which of the following disorders of cardiac function? a) Aortic valve regurgitation and aortic stenosis b) Pericardial effusion and cardiac tamponade c) Dilated cardiomyopathies d) Atherosclerosis and unstable angina

d) Atherosclerosis and unstable angina Coronary artery bypass grafting (CABG) may be the treatment of choice for people with significant coronary artery disease (CAD) who do not respond to medical treatment and who are not suitable candidates for percutaneous coronary intervention. CABG does not address valve disorders, pericardial effusion, or cardiomyopathies.

A patient comes to the clinic following exposure to chicken pox. The patient states he had chicken pox as a child but was worried about getting sick again. Which of the following statements best explains humoral-mediated immunity to the patient? a) The body has natural immunity to prevent active diseases from occurring following expose to a pathogen. b) T cells in the body are natural killer cells that quickly attack and kill antigens in order to prevent infections. c) Monocytes in the body are activated in the presence of pathogens. They quickly engulf and destroy invading organisms, preventing infections from occurring. d) B lymphocytes (B cells) in the body produce antibodies to fight infections. Once exposed, the cells retain memory and are able to quickly fight off pathogens during re-exposure and prevent the disease from reoccurring.

d) B lymphocytes (B cells) in the body produce antibodies to fight infections. Once exposed, the cells retain memory and are able to quickly fight off pathogens during re-exposure and prevent the disease from reoccurring. The B lymphocytes (B cells) differentiate to form antibody-producing plasma cells that retain memory and are involved in humoral-mediated immunity.

Which of the following cardiac drug classifications decreases sympathetic outflow to the heart and is the is the cornerstone of therapy for catecholaminergic polymorphic ventricular tachycardia (CPVT)? a) Potassium blockers b) Sodium channel blockers c) Calcium channel blockers d) Beta-adrenergic blockers

d) Beta-adrenergic blockers Antiarrhythmic drugs act by modifying disordered formation and conduction of impulses that induce cardiac muscle contraction. Beta-adrenergic-blocking drugs decrease sympathetic outflow to the heart. Antiadrenergic treatment with beta-blockers is the cornerstone of therapy for CPVT. The remaining options act by blocking specific electrolytes, thus altering electrical impulses affecting the heart.

A nurse is participating in a health fair and is addressing many of the varied factors that can contribute to hypertension. The nurse should be cognizant of the higher incidence and prevalence of hypertension in which groups? a) Aboriginals and South Asians b) East Asians and blacks c) Aboriginals and Caucasians d) Blacks and South Asians

d) Blacks and South Asians A 2006 Canadian study showed that the ethnic groups consisting of South Asians (30%) and blacks (31%) in Canada are more likely to have hypertension in comparison to the primarily Caucasian population (21%) and East Asians are the least likely to have hypertension (19%).

A pregnant female client is at risk for the development of preeclampsia-eclampsia. Select the most important data to assess. a) Blood pressure 90/60 and proteinuria in the second trimester b) Blood pressure 130/88 and hyperglycemia occurring during the 22nd week of pregnancy c) Blood pressure 140/90 and glucose in the urine during the 36th week of pregnancy d) Blood pressure 160/100 mm Hg and proteinuria during the 30th week of pregnancy

d) Blood pressure 160/100 mm Hg and proteinuria during the 30th week of pregnancy Preeclampsia-eclampsia is defined as an elevation in blood pressure (systolic >140 mm Hg or diastolic >90 mm Hg) and proteinuria (300 or greater in 24 hours) developing after 20 weeks of gestation.

The nurse is caring for a client who has just experienced an acute myocardial infarction and is diagnosed with "pump failure." The nurse is aware that the client is experiencing which type of shock? a) Septic b) Neurogenic c) Hypovolemic d) Cardiogenic

d) Cardiogenic Cardiogenic shock occurs when the heart fails to pump blood sufficiently to meet the body's demands. Clinically, it is defined as decreased cardiac output, hypotension, hypoperfusion, and indications of tissue hypoxia despite an adequate intravascular volume. Cardiogenic shock most commonly occurs from an acute myocardial infarction.

The practitioner notes the client with hemolytic anemia has Raynaud phenomenon. Which of the following causes this type of anemia? a) Prosthetic heart valve b) Deficiency of glucose-6-phosphate dehydrogenase (G6PD) c) Warm-reacting antibodies d) Cold-reacting antibodies

d) Cold-reacting antibodies The hemolytic process caused by cold-reacting antibodies occurs in distal body parts, where the temperature may fall below 30 degrees C. Vascular obstruction of red cells results in pallor, cyanosis of of the body parts exposed to cold temperatures, and Raynaud phenomenon. The other options cause hemolytic anemia, but not Raynaud phenomenon.

The heart controls the direction of blood flow. What is the role of the aortic valve? a) Controls the direction of blood flow from the atria to the ventricles b) Controls the direction of blood flow from the left side of the heart to the lungs c) Controls the direction of blood flow from the ventricles to the artia d) Controls the direction of blood flow from the left side of the heart to the systemic circulation

d) Controls the direction of blood flow from the left side of the heart to the systemic circulation The heart valves control the direction of blood flow from the atria to the ventricles (the AV valves), from the right side of the heart to the lungs (pulmonic valves) and from the left side of the heart to the systemic circulation (aortic valve)

Ascites is an accumulation of fluid in the peritoneal cavity and usually occurs in advanced cirrhosis. What is the treatment of choice for ascites? a) Thoracentesis b) Paracentesis c) DDAVP d) Diuretics

d) Diuretics Because of the many limitations in sodium restriction, the use of diuretics has become the mainstay of treatment for ascites. A paracentesis may be done if the diuretics do not correct the problem. A thoracentesis would never be done for ascites. DDAVP is given to decrease urine output, not increase it.

Leukocytes consist of three categories of cells that have different roles in the inflammatory and immune responses. Which of the following leukocytes is correctly matched with its function? a) Monocytes—release heparin b) Lymphocyte—phagocytosis c) Basophils—engulf antigens d) Eosinophils—allergic reactions

d) Eosinophils—allergic reactions Eosinophils, a type of granulocyte, increase in number during allergic reactions. Lymphocytes (agranulocytes) consist of three cell types that are not phagocytes but do have an important role in the immune response. Basophils and mast cells release heparin and histamine in response to allergens. Monocytes and macrophages are phagocytes that engulf antigens.

The school nurse is doing a health class on the functional organization of the circulatory system. What is the function of the capillaries in the circulatory system? a) Distribute oxygenated blood to the tissues b) Pump blood c) Collect deoxygenated blood from the tissues d) Exchange gases, nutrients and wastes

d) Exchange gases, nutrients and wastes The circulatory system consists of the heart which pumps blood, the arterial system which distributes oxygenated blood to the tissues, the venous system which collects deoxygenated blood from the tissues and returns it to the heart, and the capillaries where exchange of gases, nutrients and waste takes place.

Hemophilia A is a hereditary blood disorder caused by inadequate activity or absence of: a) Intrinsic factor b) von Willebrand complex c) Prothrombin d) Factor VIII

d) Factor VIII Factor VIII gene deficiency or absence is the cause of hemophilia A. Intrinsic factor and prothrombin are part of the coagulation process and unrelated to hemophilia A. Von Willebrand factor (vWF) disease is caused by a lack of vWF complex, which binds and stabilizes factor VIII

Anemia refers to a deficiency of: a) Blood plasma b) Platelets c) Folic acid d) Hemoglobin

d) Hemoglobin Anemia is a condition of an abnormally low number of circulating red blood cells or hemoglobin level, or both. It is not a disease but a manifestation of a disease process or alteration in body function.

Which of the following is the main job of lymphocytes? a) Degranulation b) Apoptosis c) Phagocytosis d) Immune reaction

d) Immune reaction Immune reaction is the main job of lymphocytes.

When will the nurse plan to assess a patient's blood pressure to confirm the possible diagnosis of orthostatic hypotension?. a) In the evening just before sleep b) Within a half hour after the heaviest meal of the day c) Midmorning right after taking a short walk d) In the morning before arising from bed

d) In the morning before arising from bed To confirm orthostatic hypotension, blood pressure should be assessed while the patient is supine and then after standing for one minute and 3 minutes. A fall of 20 mm Hg or more in systolic pressure, or 10 mm Hg or more in diastolic pressure are considered orthostasis.

A patient is admitted to the emergency department with a diagnosis of polycythemia. The nurse plans to assess for symptoms related to which of the following problems? a) Decreased ability to carry oxygen to the cells b) Increased risk of infection c) Inability to form blood clots d) Increased blood viscosity

d) Increased blood viscosity Unregulated overproduction of the red cell mass is termed polycythemia, which causes a thickening of the blood and an increased risk of blood clots.

A client has experienced sympathetic nervous stimulation of the heart. The nurse is aware that the client may manifest which of the following? a) Decreased rate and force of contraction b) Increased heart rate and decreased contractility c) Decreased contractility and decreased heart rate d) Increased heart rate and increased contractility

d) Increased heart rate and increased contractility The sympathetic nervous system has an excitatory influence on heart rate and contractility, and it serves as the final common pathway for controlling the smooth muscle tone of the blood vessels. With stimulation, both heart rate and contractility would increase.

When caring for the client with hepatic failure, the nurse recognizes which of these problems places the client at increased risk for bleeding? a) Increased vitamin K b) Increased platelet count c) Decreased red blood cells d) Increased prothrombin time

d) Increased prothrombin time An increased prothrombin time would increase the risk for bleeding. Other factors that contribute to increased bleeding risk in patients with liver failure is malabsorption of vitamin K (decrease), which further impairs the synthesis of clotting factors. Increased platelets would not cause increased bleeding but rather increase the risk of clotting, while red blood cell count is not relevant.

Which of the following blood flow patterns reduces friction, allowing the blood layers to slide smoothly over one another? a) Crosswise b) Axially c) Turbulent d) Laminar

d) Laminar Laminar blood flow reduces friction by allowing the blood layers to slide smoothly over one another, with the axial layer having the most rapid rate of flow. Axially, crosswise, and turbulent blood flow would result in increased friction. In turbulent flow, the laminar stream is disrupted and the flow becomes mixed, moving radially (crosswise) and axially (lengthwise).

A nurse sends a blood sample to the lab for analysis. Assuming the sample is normal, the nurse anticipates which of the following white blood cells (WBCs) will account for the highest percentage? a) Monocytes b) Eosinophils c) Lymphocytes d) Neutrophils

d) Neutrophils Neutrophils constitute 55-65% of the total WBCs.

The pericardium is a tri-layer sac. Which layer prevents acute dilation of the heart chambers and exerts a restraining effect on the left ventricle? a) Visceral layer b) Inner serous layer c) Parietal layer d) Outer fibrous layer

d) Outer fibrous layer The outer fibrous layer prevents acute dilation of the heart chambers. The inner serous layer consists of a visceral layer and a parietal layer.

A 16-year-old girl has been brought to her primary care provider by her mother due to the daughter's recent malaise and lethargy. Which of the following assessments should the clinician perform in an effort to confirm or rule out infectious mononucleosis? a) Auscultating the client's lungs b) Assessing the client for bone pain c) Assessing the client's cranial nerve reflexes d) Palpating the client's lymph nodes

d) Palpating the client's lymph nodes In cases of infectious mononucleosis, the lymph nodes are typically enlarged throughout the body, particularly in the cervical, axillary, and groin areas. Palpation of these nodes is a priority assessment in cases of suspected mononucleosis. Bone pain, adventitious lungs sounds, and abnormal cranial nerve reflexes do not accompany mononucleosis.

A 75-year-old patient with a history of heart valve replacement arrives at the outpatient clinic with multiple red pinpoint lesions. The nurse identifies the lesions as being which of the following? a) Purpura b) Erythema c) Ecchymoses d) Petechiae

d) Petechiae Petechiae are pinpoint red lesions caused by bleeding under the skin and can be noted in the presence of many bleeding disorders, including structural weakening of blood vessels that comes with age.

A mother has brought her 2-week-old infant to the emergency department due to the baby's persistent and increasing jaundice. Blood testing reveals that the infant's unconjugated bilirubin level is 28 mg/dL, and assessment does not reveal neurologic deficits. The infant's weight is normal, and the mother claims to have had no significant difficulty feeding the infant. The most likely treatment for this infant will be: a) Phlebotomy b) Intravenous antibiotics c) Packed red blood cell transfusion d) Phototherapy

d) Phototherapy Phototherapy is the standard treatment for mild to moderate hyperbilirubinemia, with exchange transfusion an option for greater risks of kernicterus. Blood transfusions, phlebotomy, and antibiotics are not indicated in hyperbilirubinemia.

A nurse is caring for a patient who recently received GPIIb/IIIa inhibitors and is concerned that the patient has developed thrombocytopenia as demonstrated by which of the following? a) Platelet count greater than 200,000/μL b) Platelet count greater than 150,000/μL c) Red blood cell count less than 5.4 million per microliters d) Platelet count less than 50,000/μL

d) Platelet count less than 50,000/μL Thrombocytopenia is defined as a platelet count below the normal level of 150,000/μL and the GPIIb/IIIa medications are indicated in causing acute and delayed thrombocytopenia.

A client tells the nurse that the doctor told her she has too many red blood cells accompanied by elevated white cells and platelet counts. The nurse recognizes this as: a) Aplastic anemia b) Hemolytic anemia c) Pernicious anemia d) Polycythemia vera

d) Polycythemia vera Polycythemia vera is a neoplastic disease of the pluripotent cells of the bone marrow characterized by an absolute increase in total red blood cell (RBC) mass accompanied by elevated white cell and platelet counts. In pernicious anemia, the RBCs are not high in number but are larger in size. In aplastic and hemolytic anemia, there is a small number of RBCs

Which goal is a priority for a nurse caring for a patient diagnosed with orthostatic hypotension? a) Supporting an exercise program b) Monitoring for signs of diabetes mellitus c) Encouraging a low-fat diet d) Preventing falls

d) Preventing falls Orthostatic hypotension is an abnormal decrease in systolic and diastolic blood pressure that occurs upon standing. Dizziness and syncope are frequent consequences. Treatment includes correcting possible causes, and assisting the patient to compensate for the disorder and prevent falls and injuries, and to avoid circumstances that promote vasodilation or loss of body fluids. Peripheral neuropathy of diabetes can impede autonomic reflex responses to decreases in blood pressure. Vigorous exercise in a warm environment is discouraged.

When an Rh-negative mother has been sensitized and is pregnant with an Rh-positive fetus, what happens to the fetus? a) Nothing, this is normal b) Plasma volume depletion c) Bilirubin deficiency d) Profound red cell hemolysis

d) Profound red cell hemolysis This situation is totally detrimental to the health of the fetus. Rh incompatibility and production of antibodies by the mother will result in life-threatening fetal red cell hemolysis. The bilirubin will be sufficiently elevated to cause brain damage. The infant will have massive edema from a lack of albumin production.

A client is at high risk for the development of rheumatic heart disease. The most important information for the nurse to provide would be: a) Yearly electrocardiography after the age of 50 b) Avoiding frequent dental examinations c) Annual blood specimen assessed for rheumatoid factor d) Prompt diagnosis and treatment of streptococcal infections

d) Prompt diagnosis and treatment of streptococcal infections Rheumatic heart disease is normally caused by streptococcal infections; therefore, early diagnosis of these would decrease the risk for the disease. Frequent EKG may be required after a client has a history of rheumatic heart disease. Rheumatoid factor is not related to rheumatic heart disease, and the client should have regular dental examinations.

A nurse preceptor is evaluating the skills of a new registered nurse (RN) caring for clients experiencing shock. Which action by the new RN indicates a need for more education? a) Inserting an IV to begin a normal saline infusion b) Administration of 2L of oxygen by nasal cannula c) Placing a pulse oximeter on the client to monitor oxygenation status d) Raising the head of the bed to a high Fowler's position

d) Raising the head of the bed to a high Fowler's position. Treatment measures include close monitoring of cardiovascular and respiratory function; maintenance of respiratory gas exchange, cardiac output, and tissue perfusion; and the administration of oxygen, antihistamine drugs, and corticosteroids. The person should be placed in a supine position. This is extremely important because venous return can be severely compromised in the sitting position. This in turn produces a pulseless mechanical contraction of the heart and predisposes to arrhythmias. In several cases, death has occurred immediately after assuming the sitting position.

The nurse is reviewing the anatomy and physiology of the heart. What is the function of the right atrium? a) Pumps blood into the systemic circulation b) Pumps blood to the lungs c) Receives oxygenated blood from the lungs d) Receives blood returning to the heart from the systemic circulation

d) Receives blood returning to the heart from the systemic circulation The right atrium receives blood returning to the heart from the system circulation. The left atrium receives oxygenated blood from the lungs. The right ventricle pumps blood to the lungs. The left ventricle pumps blood into the systemic circulation.

In infants and children, secondary hypertension is the most common form of hypertension. What is the most common cause of hypertension in an infant? a) Cerebral vascular bleed b) Pheochromocytoma c) Coarctation of the aorta d) Renal artery thrombosis

d) Renal artery thrombosis Hypertension in infants is associated most commonly with high umbilical catheterization and renal artery obstruction caused by thrombosis. Cerebral vascular bleeds, coarctation of the aorta, and pheochromocytoma all can raise blood pressure; they are not the most common cause of hypertension in an infant.

A 60-year-old client is being evaluated by the nurse practitioner for complaints of palpitations, dizziness, and near syncope. The client states that the symptoms seem to appear out of nowhere and go as quickly as they come. The nurse practitioner then attaches the client to a cardiac monitor and notes intermittent changes in heart rate from the mid-50s to the mid-120s. P waves are present although not before all QRS complexes. What does the nurse practitioner suspect? a) Atrial tachycardia b) Respiratory sinus arrhythmia c) Atrial fibrillation d) Sick sinus syndrome

d) Sick sinus syndrome This client most likely has sick sinus syndrome (SSS), also known as bradycardia-tachycardia syndrome, due to the episodes of bradycardia and tachycardia not appropriate for the physiologic situation. SSS includes intra-atrial and AV conduction abnormalities and most often involves destruction of the SA node, occlusion of the sinus node artery, or inflammation of the nerves that surround the node. The symptoms that accompany SSS are most often caused by the bradycardia and include lightheadedness, dizziness, and syncope.

The practitioner carefully monitors his client who exhibits hemoglobin S (HbS) genes. The practitioner know that the client is predisposed to life-threatening infection due to damage by HbS to which of the following organs? a) Heart b) Lungs c) Pancreas d) Spleen

d) Spleen Sickle cell disease is an inherited disorder in which the person has abnormal hemoglobin S (HbS). The spleen is especially susceptible to damage by HbS. The congestion of the spleen predisposes the person to life-threatening infections caused by encapsulated organisms

The nursing instructor is teaching the students about rheumatic fever. She tells the students that it is an important cause of heart disease and is very serious mainly for which reason? a) That there is no definitive test used for diagnoses b) The fact that it affects young and old c) The cost associated with treating the disease d) The disabling effects that result from involvement of heart valves

d) The disabling effects that result from involvement of heart valves Rheumatic fever is a very important cause of heart disease and its most serious and disabling effects result from involvement of the heart valves.

A 6-year-old child is having a stem cell transplant using umbilical cord blood. What benefit does the nurse understand that this type of transplant will have for the child? a) The child will not have to take any form of chemotherapy. b) This type of transplant does not require an inpatient stay in the hospital. c) There are no side effects to this type of transplant. d) This type of transplant creates less risk of graft-versus host disease

d) This type of transplant creates less risk of graft-versus host disease Umbilical cord blood from HLA-matched donors is a transplant option for children and carries less risk of graft-versus-host disease. Stem cell transplants focus on correcting bone marrow failure, immunodeficiencies, hematologic defects and malignancies, and inherited errors of metabolism. Sources of the stem cells include bone marrow and umbilical cord blood, which replenish the recipient with a normal population of pluripotent stem cells. Stem cell transplants may be derived from the client (autologous) or from a histocompatible donor (allogeneic). Autologous transplants are often used to replenish stem cells after high-dose chemotherapy or irradiation.

A client with a 2-week history of hyperthyroidism due to a virus is admitted for non-emergent cardioversion. The client states that she has been experiencing intermittent episodes of palpitations over the past week along with shortness of breath and fatigue. The nurse attaches the client to a cardiac monitor and notes that the client is in normal sinus rhythm with frequent episodes of atrial fibrillation. The nurse anticipates which of the following to be ordered prior to the cardioversion to avoid a possible stroke? a) Ultrasound of the thorax b) Pulmonary function studies c) Chest x-ray d) Transesophageal echocardiography

d) Transesophageal echocardiography Cardioversion to normal sinus rhythm involves a significant risk of thrombus development with subsequent emboli breaking off and causing a stroke or other embolic complications. Transesophageal echocardiography (TEE) uses sound waves to create pictures of the inside of the heart and can help to insure that there are no thrombi in the atria prior to the cardioversion and may also continue to be used during the procedure. Transesophageal echocardiography provides views of the upper heart chambers that are much clearer than the views provided by standard echocardiography.

An infant from parents of Mediterranean decent has been diagnosed with a severe form of β-thalassemia anemia. The nurse caring for this infant knows that the infant will likely receive which of the following medical treatments? a) Warfarin, a blood thinner to decrease clot formation b) Iron sulfate supplements c) Stem cell transplant d) Transfusion therapy

d) Transfusion therapy Persons who are homozygous for the trait (thalassemia major) have severe, transfusion-dependent anemia that is evident at 6 to 9 months of age when the hemoglobin switches from HbF to HbA. If transfusion therapy is not started early in life, severe growth retardation occurs in children with the disorder. Iron and blood thinners will not be therapeutic for this client. Stem cell transplantation is a potential cure for low-risk clients, particularly in younger persons with no complications of the disease or its treatment, and has excellent results.

A client with a suspected MI is brought to the emergency department by ambulance. The nurse caring for this client would expect to receive an order for which laboratory test to confirm a diagnosis of MI? a) Complete blood components b) Creatine kinase marker c) Calcium level d) Troponin level

d) Troponin level The troponin assays have high specificity for myocardial tissue and have become the primary biomarker for the diagnosis of myocardial infarction (MI). The troponin complex, which is part of the actin filament consists of three subunits (i.e., TnC, TnT, and TnI) that regulate calcium mediated actin myosin contractile process in striated muscle (see Chapter 1, Fig. 1-19). TnI and TnT, which are present in cardiac muscle, begin to rise within 3 hours after the onset of MI and may remain elevated for 7 to 10 days after the event. This is especially adventitious in the late diagnosis of MI. The other blood work may be ordered, but not to confirm the diagnosis of MI.

Which of the following signs and symptoms is most suggestive of acute cholecystitis? a) Fever and sudden abdominal distention b) Appearance of undigested fat in feces c) Nausea resulting in greenish vomitus d) Upper right quadrant or epigastric pain

d) Upper right quadrant or epigastric pain Persons with acute cholecystitis usually experience an acute onset of upper right quadrant or epigastric pain. Nausea and vomiting are also common, although these are not specific to cholecystitis. Abdominal distention and steatorrhea are not key signs of acute cholecystitis.

A patient is told that she has cardiac valve leaflets, or cusps, that are floppy and fail to shut completely, permitting blood flow even when the valve should be completely closed. The nurse knows that this condition can lead to heart failure and is referred to as which of the following? a) Valvular stenosis b) Infective endocarditis c) Pericardial effusion d) Valvular regurgitation

d) Valvular regurgitation When cardiac valves such as the aortic or mitral fail to close properly, blood does not efficiently exit from the left ventricle. This condition is valvular regurgitation. Valvular stenosis is a narrowing of the lumen of the valve. Pericardial effusion, an accumulation of fluid in the pericardial sac causes obstructive shock, as it impedes ventricular filling. Infective endocarditis could trigger septic shock.

Select the first stage of hemostasis. a) Clot dissolution b) Formation of the platelet plug c) Clot retraction d) Vessel spasm

d) Vessel spasm Hemostasis is divided into five stages: (1) vessel spasm, (2) formation of the platelet plug, (3) blood coagulation or development of an insoluble fibrin clot, (4) clot retraction, and (5) clot dissolution.

The nurse is caring for a client who is a strict vegetarian; the client is at greatest risk for the development of: a) Folic acid deficiency anemia b) Blood loss anemia c) Microcytic anemia d) Vitamin B12 deficiency anemia

d) Vitamin B12 deficiency anemia Vitamin B12 is found in all foods of animal origin. Dietary deficiency is rare and usually found only in strict vegetarians who avoid all dairy products as well as meat and fish. The hallmark of vitamin B12 deficiency is megaloblastic anemia. The other options are not affected by the client being a vegetarian.

To form a platelet plug, platelets must adhere to the vessel inner layer. For this to occur, which protein molecule is required? a) Thromboxane A2 b) Plasminogen c) Lipoprotein d) Von Willebrand factor

d) Von Willebrand factor Platelet adhesion requires a protein molecule called von Willebrand factor (vWF). This factor is produced by both megakaryocytes and endothelial cells and circulates in the blood as a carrier protein for coagulation factor VIII. The plasma also contains a plasma protein called plasminogen, which gets activated and converted to plasmin, an enzyme capable of digesting the fibrin strands of the clot. The extrinsic pathway of coagulation, which is a much faster process, begins with trauma to the blood vessel or surrounding tissues and with the release of an adhesive lipoprotein called tissue factor (also known as thromboplastin or factor III) from the subendothelial cells. Thromboxane A2 (TXA2) is released during platelet aggregation, the step after adhesion.

When assessing the client during the icteric phase of viral hepatitis, which of these findings does the nurse anticipate observing? a) Yellow-tinged blood b) Increased energy c) Blood in the stool d) Yellow-tinged sclera

d) Yellow-tinged sclera The icterus phase is reflected by development of jaundice of skin and sclera, liver tenderness and worsened prodromal symptoms.

Severe shock can be followed by acute lung injury/acute respiratory distress syndrome (ALI/ARDS) characterized by: a) excessive surfactant. b) hyperventilation. c) hyperinflated alveolar sacs. d) ventilation-perfusion mismatch.

d) ventilation-perfusion mismatch. Despite the delivery of high levels of oxygen using high-pressure mechanical ventilatory support and positive end-expiratory pressure, many persons with ALI/ARDS remain hypoxic, often with a fatal outcome. Arterial blood gas analysis establishes the presence of profound hypoxemia with hypercapnia, resulting from impaired matching of ventilation and perfusion and from the greatly reduced diffusion of blood gases across the thickened alveolar membranes. Abnormalities in the production, composition, and function of surfactant may contribute to alveolar collapse and gas exchange abnormalities.

Many different proteins, enzymes, and hormones are involved in maintaining hemostasis. Which protein is required for platelet adhesion? a) Growth factors b) Ionized calcium c) Platelet factor 4 d) von Willebrand factor

d) von Willebrand factor Platelet adhesion requires a protein molecule called von Willebrand factor. This factor is produced by the endothelial cells of blood vessels and circulates in the blood as a carrier protein for coagulation factor VIII. The release of growth factors results in the proliferation and growth of vascular endothelial cells, smooth muscle cells, and fibroblasts and is important in vessel repair. Ionized calcium contributes to vasoconstriction. Platelet factor 4 is a heparin-binding chemokine.


Conjuntos de estudio relacionados

Chapter 1: Human Growth and Development

View Set

Info Sec Chapter 10 Implementing Security

View Set

Unit 2 Quiz 7 - Blood Vessels of the Upper Limb

View Set

Reproduction & Development FINISH AND Switch to NOTION

View Set

Question 8, Question 15, Question 16

View Set

Iggy Chapter 62: Care of Patients With Pituitary and Adrenal Gland Problems

View Set